SlideShare una empresa de Scribd logo
1 de 55
Descargar para leer sin conexión
1
1. EJERCICIOS SOBRE CONCEPTOS BÁSICOS
1. En la figura, el ángulo COB mide 120º y el ángulo COD mide la mitad del ángulo BOA. Entonces, la
medida del  BOA es:
Sea mBOA = x, luego mCOD = x/2. Se tiene
x 3x
120 x 180 60 x 40
2 2
          
2. Si dos planos diferentes se intersecan, su intersección es
A. Un punto. B. Dos puntos C. Una única recta
D. Dos rectas diferentes E. Falta información
Esto es uno de los axiomas de la Geometría Euclidiana.
3. En la figura, 3241 mmmm

 , ¿cuál de las siguientes expresiones es siempre verdadera?
Con la información dada las parejas de rectas perpendiculares están
“libres”, luego pueden ser giradas y seguirían satisfaciendo los
datos dados.
Por tanto no puede afirmarse ni A, ni B, ni C, ni D.
4. R, S y T son tres puntos colineales como se muestran en la figura. Si ST = 4x + 4 y RS es la mitad
de ST, entonces la longitud de RT es
A. 3x – 4 B. 3x – 6 C. 3x + 2 D. 6x – 12 E. 6x + 6
Dado que los puntos son colineales, se tiene RT = RS + ST =
1 3 3
ST ST ST
2 2 2
   (4x +4) = 6x + 6
C B
D O A
A. 20º B. 30º C. 40º D. 60º E. 80º
120
x/2 x
1m

2m

3m

4m

R S T
  
A. 21 mm

|| B. 31 mm

 C. 43 mm

||
D. 42 mm

 E. NDLA
1m

2m

3m

4m

PROHIBIDA
LA
VENTA
GEOMETRÍA
2
Sean A, B y C los puntos indicados en la figura, y sean mBAC = , mACB = .
Se tiene  = 50, por ser opuesto por el vértice con el ángulo que mide 50 y  = 180 – 130 = 50.
El ángulo que mide y es un ángulo exterior con respecto al ABC, luego su medida equivale a la suma
de los ángulos internos no adyacentes, es decir y =  +  = 50 + 50 = 100
Dado que las rectas son paralelas, x = mFCE, por ser ángulos correspondientes. A su vez este ángulo
por ser externo al ECD, es la suma de las medidas de los ángulos CED y EDC.
Se tiene mCED = 90 y mEDC = 180 – 140 = 40, luego x = 90 + 40 = 130
Las marcas en el ángulo A, indican que AD es bisectriz de dicho ángulo, luego x = 40 y mA = 80.
Al considerar el ABC, se tiene z = 180 – 80 – 70 = 30
Dado que
__ __
EB ||DC , se tiene x = 180 – 130 = 50 por ser ángulos internos al mismo lado, entre
paralelas y como
__ __
AD AC, el ADC es triangulo rectángulo y por tanto “y” es el complemento de “x”,
luego y = 90 – 50 = 40
x
y
130º
A B C
D
E
En la figura,
________
DC||EB,ACAD , entonces el valor de y es:
A. 30º B. 40º C. 45º D. 50º E. 60º
8.
50º 120º
130º
xº
yº
A partir de la información indicada en la figura,
el valor de y es:
A. 170º B. 130º C. 120º D. 100º E. 50º
5.
A
B C


90º
140º
xº
En la figura, si
____
CD||AB , el valor de x es:
A. 50º 70º C. 130º D. 140º E. 230º
A B
F C D
6.
E
xº
A partir de la información brindada en la figura, el valor de z
resulta:
A. 30º B. 40º C. 70º D. 80º E. 110º
x
z
40
70º
7.
A
B D C
PROHIBIDA
LA
VENTA
GEOMETRÍA
3
Se tiene mABC = 180 – 140 = 40, x = 115 – 40 = 75 ya que el ACD es externo al ABC
Se tiene mEDB = 180 – 150 = 30, ABC  DBE por ser opuestos por el vértice y por el teorema de
semejanza AA, ABC  DBE, luego mBAC = x = mEDB = 30.
11. A – B – C – D; E y F son puntos medios de AB y CD respectivamente; Si AC = 10 y BD = 12,
entonces EF = ?
A. 5 B. 6 C. 9 D. 11 E. 22
Sean AE = EB = x, CF = FD = y (E y F son puntos medios de AB y CD respectivamente)
Se tiene BC = AC – AB = 10 – 2x (1)
y también BC = BD – CD = 12 – 2y (2)
Igualando (1) y (2): 10 – 2x = 12 – 2y Al simplificar se obtiene y – x = 1 (3).
Por otro lado se tiene EF = EB + BC + CF = x + (10 – 2x) + y = 10 – x + y = 10 + (y – x)
Al introducir (3) resulta EF + 10 + 1 = 11
x
140º
115º
9.
En la figura, el valor de x es
A. 25º B. 40º C. 45º D. 65 E. 75º
A
B C D
xº
150º
10.
En la figura, el valor de x es
A. 30º B. 40º C. 45º D. 50º E. 60º
A
B
C
D E
A E B C F D
x x yy
10
12
PROHIBIDA
LA
VENTA
GEOMETRÍA
4
En el ABC, tenemos que mA = 180 – mABC – mACB = 180 – (mABC – mACB) (1)
Se tiene que mABC = 180 –  y mACB = 180 – , luego mABC + mACB = 360 – ( + )
Como  +  = 255, resulta mABC + mACB = 360 – 255 = 105
Sustituyendo en (1) obtenemos mA = 180 – 105 = 75
13. ¿Para qué valor de x, los segmentos AB y CD son paralelos?
Como el ángulo a la izquierda de C es congruente con el ángulo a la derecha, también mide 25. Luego
mACD = 180 – 2  25 = 130.
Como el APC, es recto en P, mPAC = 90 – 25 = 65
Para que AB y CD sean paralelos, el ángulo CAB debe ser el suplemento del ángulo ACD ya que serían
ángulos internos al mismo lado entre paralelas o sea mCAB = 180 – 130 = 50.
Se tiene entonces x + 50 + 65 = 180  x = 180 – 50 – 65 = 65
14. Si
____
CD||AB , ¿Cuál es el valor de x?
A. 170º B. 150º C. 120º D. 100º E. 80º
Trazamos EF, paralela a las rectas AB y CD, luego mAEF = 180º – 120º = 60º
y mFEC = 180º – xº . Además se tiene mAEF + mFEC = mAEC = 90 , luego
60 + (180º – xº) = 90. Al despejar x, resulta x = 150
 
A
B C
En la figura º + º = 255º, entonces ¿m A = ?
A. 75º B. 105º C. 127.5º D. 30º E. 45º
12.
25º
xº
A. 25 B. 50 C. 65 D. 75 E. 130
A
B
C
D
25º
130º
P
65
120º
xº
A B
C D
E F
PROHIBIDA
LA
VENTA
GEOMETRÍA
5
15. Si la medida de un ángulo es tres veces la medida de su suplemento, ¿cuál es la medida de dicho
ángulo?
A. 30º B. 60º C. 90º D. 120º E. 135º
Sean  la medida del ángulo buscado y  la medida de su suplemento, luego
 +  = 180   = 180 – .
El ejercicio indica que  = 3, luego  = 3 (180 – ) = 540 – 3   4 = 540   = 135.
16. Dos veces la medida de un ángulo es 30° menos que cinco veces la medida de su complemento,
¿cuál es la medida de dicho ángulo?
A. 30º B. 60º C. 90º D. 120º E. 135º
Sean  la medida del ángulo buscado y  la medida de su complemento, luego
 +  = 90   = 90 – .
Al interpretar la información del ejercicio se tiene
2 = 5 – 30  2 = 5 (90 – ) – 30 = 450 – 5  – 30  7 = 420   = 60
Sean A, B, C y D los puntos indicados en la figura. Al trazar por B una paralela a

1m y

2m , se forman
ángulos alternos – internos entre paralelas, y por tanto congruentes con los ángulos indicados
inicialmente, luego
x + 60 = 110  x = 110 – 60 = 50.
OTRA FORMA:
Al prolongar CB, sea D el punto donde corta a la recta

1m
Se tiene mADB = x, por ser alterno – interno con el ángulo que se
forma
en C.
El angulo ABC es externo al ABD, luego 60 + x = 110  x = 50

1m y

2m son paralelas.18. En la figura las rectas
Entonces el valor de x es
A. 170 B. 50 C. 85 D. 25 E. 20
Como las rectas son paralelas se tiene (3x + 10) + (x – 6) = 84  4x + 4 = 84  4x = 80  x = 20
60º
110º
xº 
2m

1m17.
En la figura las rectas

1m y

2m son paralelas. Entonces el valor
de x es
A. 170 B. 50 C. 85 D. 25 E. 30
A
B
C
xº
60º
60º
110º
xº 
2m

1m
A D
B
C
xº
84°
(x – 6)°
(3x + 10)°

1m

2m
PROHIBIDA
LA
VENTA
GEOMETRÍA
6
Sean  y  las medidas de los ángulos indicados en la figura. Se tiene  +  = 90.
Como SQR = 2 y  1  2, se tiene 2mSQR = 180 – . (1)
Similarmente se obtiene que 2mQSR = 180 – . (2)
Al sumar (1) y (2) resulta 2mSQR + 2mQSR = (180 – ) + (180 – ) = 360 – ( + )
Como  +  = 90, 2(mSQR + mQSR) = 360 – 90 = 270  mSQR + mQSR) = 135
Luego mR = 180 – (mSQR + mQSR) = 180 – 135 = 45
.
20. En una recta se toman los puntos A, B y C, de manera que B es punto medio de
__
AC . Se toma otro
punto O, tal que B – O – C. Encuentre el valor numérico de
OB
OCAO 
.
A. 2 B. 1 C.
2
1
D.
2
3
E. Falta información.
Se tiene AB = BC = x, por ser B punto medio de AC. Sea OB = y, luego OC = x – y, AO = x + y.
Al sustituir estos valores en la expresión dada se tiene:
AO – OC = (x – y) – (x + y) = 2y, OB = y, luego
AO OC
OB


2y
2
y

Nota: en ejercicios de este tipo no se admite asignar valores arbitrarios, ya que se estaría resolviendo
para valores específicos. El planteamiento es general. Cuando se afirma que B – O – C, está indicando
que O es un punto cualquiera que se encuentra entre B y C, y el valor numérico encontrado es valido
para cualquier punto O que esté entre B y C.
P Q
S
R
1
2
3
4
Si m  P = 90º,  1  2,  3   4, entonces m  R es
A. 30º B. 45º C. 60º
D. 90º E. Falta información.
19.


A B O C
PROHIBIDA
LA
VENTA
GEOMETRÍA
7
2. EJERCICIOS SOBRE TRIÁNGULOS Y CUADRILÁTEROS
1. Un poste cercano a un árbol mide 2 m y su sombra en un momento dado mide 1.8 m, entonces si la
sombra del árbol en ese momento mide 11 m, la altura del árbol es
A. 11 m B. 11.22 m C. 12. D. 12.22 E. 13 m
Dado que los rayos del sol prácticamente caen
paralelos y que el poste y el tronco del árbol
son perpendiculares al piso, el árbol y su
sombra y la línea que une sus extremos forman
un triángulo semejante al formado por el poste
su sombra y la línea que une sus extremos,
tenemos
h 2 11 2
h 12.22
11 1.8 1.8

   
2. Una varilla clavada en el piso y cercana a un árbol mide 3 m y su sombra mide 1.5 m, entonces si el
árbol mide 36 m, su sombra mide
A. 36 m B. 30 m C. 18 m D. 15 m E. 9 m
El problema es similar al anterior, en este
caso se tiene
x 1.5 36 1.5
x 18
36 3 3

   
3. El perímetro de un triángulo rectángulo isósceles con hipotenusa igual a 10 redondeado a dos
decimales es
A. 7.07 B. 14.14 C. 24.14 D. 24.99 E. 50
En un triángulo rectángulo isósceles, la hipotenusa mide 2 x , siendo x la longitud de sus catetos, luego
10
2 x 10 x 5 2
2
   
Su perímetro será P 10 2 5 2 10 10 2 24.14     
2
1.8 11
h
3
1.5 x
36
PROHIBIDA
LA
VENTA
GEOMETRÍA
8
Por el teorema de la altura se tiene 2
4 x 8 64 x 16     y la hipotenusa mide 4 + x = 20
Por el teorema de los catetos se tiene 2
y 4 20 80 y 80 4 5 8.94      
5. Un método para encontrar la altura de un edificio es colocar un espejo en el suelo y después situarse
de manera que la parte más alta del edificio pueda verse en el espejo ¿qué altura tiene un edificio si una
persona cuyos ojos están a 1.5 m del piso observa la parte superior del edificio cuando el espejo está a
120 m del edificio y la persona está a 6 m del espejo?
A. 20 m B. 30 m C. 31.5 m D. 120 m E. 126 m
Dado que las leyes de la óptica
indican que en un espejo plano, el
ángulo de incidencia es igual al
ángulo de reflexión, se forman dos
triángulos rectángulos
semejantes, luego
h 1.5
h 30
120 6
   metros.
6. La altura respecto a la hipotenusa de un triángulo rectángulo mide 10 m y los segmentos que
determina sobre la hipotenusa son entre sí como 7 es a 14. Entonces la longitud del cateto menor es
A. 4 m B. 7.07 m C. 12.25 m D. 14 m E. 15.5 m
Sean m y n los segmentos determinados por la altura sobre la hipotenusa,
con m < n, luego
m 7
n 2m
n 14
   .
Por el teorema de la altura
2 2
m n 10 m 2m 100 m 50 m 5 2 y n 10 2        
La hipotenusa mide c = m + n = 15 2
Por el teorema de los catetos 2
a = m(m + n) = 5 2 15 2 150  a 150 5 6 12.25  
8
y
x4
4. En el triángulo rectángulo de la figura, los valores de x e
y, respectivamente son
A. 11 y 13 B. 15 y 16 C. 9 y 8
D. 16 y 8.94 E. 12 y 8.94
h
6 120
1.5
10
a
m n
PROHIBIDA
LA
VENTA
GEOMETRÍA
9
7. El perímetro de un rectángulo es 85 m y su diagonal mide 32.5 m. Por lo tanto los lados del
rectángulo miden:
A. 15 m y 27.5 m B. 20 m y 22.5 m C. 7.5 m y 25 m D. 30m y 12.5 m E. 40m y 2.5 m
Sean a y b los lados del rectángulo. Se tiene
P = 2 (a + b) = 85  a + b = 42.5 (1)
2 2 2
a b 32.5 1056.25   (2)
Despejando b de (1), e introduciendo en (2)
2
a + 2
[42.5 a] = 1056.25  2
2a 85a 750 0    a = 12.5  a = 30
Al sustituir en (1) se obtiene b = 30  b = 12.5.
8. El perímetro de un triángulo mide 50 y sus lados son proporcionales a 4, 6 y 8. Entonces su lado
mayor mide
A. 50/3 B. 25/9 C. 100/9 D. 25 E. 200/9
Sean a, b y c las longitudes de los lados, con a < b < c, luego P = a + b + c =50 y
a b c
4 6 8
 
Por las propiedades de las proporciones
a b c a b c 50 8 50 200
c
4 6 8 4 6 8 18 18 9
  
      
 
9. En un triángulo rectángulo, un lado mide 2 106 , otro 5 15 . Si el lado desconocido es el menor,
¿cuánto mide?
A. 7 B. 8 C. 9 D. 10 E. 11
Como 2 106 5 15 , la hipotenusa de este triángulo es 2 106 , luego el cateto menor es
2 2
a [2 106 ] [5 15 ]   424 375 49 7  
10. El área del triángulo de la figura, redondeada al entero más cercano, mide:
A. 21 B. 22 C. 27 D. 31 E. 54
Aplicamos la fórmula de Herón:      A s s a s b s c       , donde s es el semiperímetro.
Se tiene
6 7 9
s 11
2
 
  , luego      A 11 11 6 11 7 11 9 11 5 4 2 440 20.97            
a
b
32.5
6
7
9
PROHIBIDA
LA
VENTA
GEOMETRÍA
10
Dado que es un triángulo rectángulo su área es la mitad del producto de sus catetos.
El cateto desconocido mide 2 2
b 10 6 100 36 64 8     
Por tanto
1
A
2
 (6) (8) = 24
12. Si un rectángulo de 3 m de ancho y 10 m de largo tiene la misma área que un triángulo rectángulo
isósceles, entonces la longitud de cada cateto del triángulo es
A. 7.5 B. 2 15 C. 15 D. 15 3 E. 10
El área de un triángulo rectángulo isósceles está dada por 21
A x
2
 , donde x es la longitud de sus
catetos, luego tenemos que el área del rectángulo es 30, por tanto 21
x 30 x 60 2 15
2
   
13. El área de un trapecio isósceles de bases 22 m y 10 m y cuyos lados congruentes miden 10 es
A. 2220 m
2
B. 160 m
2
C. 128 m
2
D. 80 m
2
E. 64 m
2
Por ser un trapecio isósceles, al proyectar la base menor sobre la base
mayor, la base mayor queda dividida en tres segmentos de 6, 10 y 6
metros.
Aplicando el teorema de Pitágoras, se tiene 2 2
h 10 6 64 8   
Aplicando la fórmula para el área de un trapecio:
 B b h
A
2
 
 resulta
  2
22 10 8
A 128 m
2
 
 
6
10
¿Cuál es el área del triángulo de la figura?
A. 20 B. 24 C. 30 D. 48 E. 60
11. b
3
10
x
x
10
22
10
10
6 10 6
h
PROHIBIDA
LA
VENTA
GEOMETRÍA
11
14. La siguiente figura consta de siete cuadrados congruentes. El área total de esta figura es 63 cm
2
.
Entonces el perímetro de la figura es:
Observamos que el perímetro está formado por 16 veces el lado de cada cuadrado. Como hay siete
cuadrados congruentes, cada uno tiene un área de 2 63
x 9 x 3
7
    . Por tanto el perímetro de la
figura es P = 16  3 = 48 cm.
La figura indica que B, D, F y H son puntos medios de los lados del cuadrado ACEG, luego su área es el
doble del área del cuadrado BDFH, es decir [ACEG] = 2  162 = 324 luego AC 324 18 
16. Se tiene un trapecio ABCD donde
__
BC es la base menor. BC = 10 cm. y CD = 20 cm. Las medidas
de los ángulos A, B y C son 30°, 150° y 120° respectivamente, entonces AD = ?
A. 60 cm. B. 50 cm. C. 40 cm. D. 30 cm. E. 20 cm.
Sean B’ y C’ las proyecciones de B y C sobre la base mayor y sean
AB’ = x, C’D = y.
Por ser BC paralela a AD, mD = 180 – mC = 180 – 120 = 60
El CC’D es un triángulo 30 – 60, luego h = CC’ = 10 3 y
y = C’D = 10
También el AB’B resulta ser un triángulo 30 – 60, con su cateto
menor BB’ = h = 10 3 , luego AB’ = 10 3 3 30 
Tenemos entonces que la base mayor mide AD = x + 10 + y = 30 + 10 +10 = 50.
A B C
G F E
H D
Si ACEG es un cuadrado y el área del cuadrilátero BDFH
mide 162 ¿cuánto mide AC? (las marcas iguales representan
partes congruentes)
A. 9 B. 12.72 C. 18 D. 25.44 E. 81
15.
A. 16 cm B. 21 cm C. 24 cm D. 48 cm E. 84 cm
B 10 C
20
A x B’ 10 C’ y D
h
30 60
PROHIBIDA
LA
VENTA
GEOMETRÍA
12
17. Si las medianas en un triángulo rectángulo, trazadas a partir de los vértices de los ángulos agudos
miden 5 cm y 40 cm, entonces la medida de la hipotenusa del triángulo rectángulo es
A.
2
405 
cm B. 2 13 cm C. 45 cm D. 11.32 cm E. 5.66 cm
Sean M y N los puntos medios de BC y AB respectivamente.
Sean AM = 5 y CN = 40 , BC = a, AB = c, luego
a c
BM y NB
2 2
  .
Sea la hipotenusa AC = b
Aplicando el teorema de Pitágoras en los ABM y BCN
2
2 2 2 2 a
AM AB BM c 25
4
     (1)
2
2 2 2 2 c
CN NB BC a 40
4
     (2)
Al sumar (1) y (2) resulta
2 2
2 2 25c 5a 4
65 a c 65 52 b b 52 2 13
4 4 5
          
18. En la figura, los cuadrados ABCD y EFGH son congruentes. AB = 10 cm y G es el centro del
cuadrado ABCD. Entonces el área total cubierta por el polígono AHEFBCDA es
Dado que los cuadrados son congruentes sus áreas son iguales y como el lado AB mide 10, cada uno
tiene un área de 100 cm
2
, pero ellos comparten el ABG de manera que para el área total del polígono a
la suma de las áreas de los cuadrados debemos restarle el área de este triángulo para que sea
considerada solo una vez.
Dado que G es el centro del cuadrado ABCD, el área del triángulo es la cuarta parte del área del
cuadrado o sea 25 cm
2
.
Luego el área buscada es A = [ABCD] + [EFGH] – [ABG] = 100 + 100 – 25 = 175 cm
2
D C
G
A B
H F
E
A. 100 cm
2
B. 120 cm
2
C. 150 cm
2
D. 175 cm
2
E. 200 cm
2
N
A
B M C
PROHIBIDA
LA
VENTA
GEOMETRÍA
13
19. ABCD es un cuadrado, el  ABE es isósceles, CF = FB. Entonces, la medida del ángulo EFB es
igual a
Como el ABE es isósceles, AE = BE y por ser ABCD un cuadrado, E es el punto medio de DC y por
tanto EC = CF, ya que por ser CF = FB, F es punto medio de BC. Luego el ECF resulta ser triangulo
rectángulo isósceles y como consecuencia mCFE = 45. El ángulo buscado es el suplemento del
CFE, luego mEFB = 180 – mCFE = 180 – 45 = 135
Se tiene que CF = AB = 18, ya que ABCF es un paralelogramo. CE = CF – FE = 18 – 12 = 6.
Por otro lado BD y AF son paralelas, luego FAE  CDE, ya que son alternos internos entre paralelas y
FEA  CED, ya que son opuestos por el vértice. Como consecuencia se tiene FEA  CED.
Sea AE = x, luego ED = AD – AE = 30 – x. Por la semejanza anterior,
CE ED 6 30 x
6x 360 12x 18x 360 x 20
FE EA 12 x

         
21. En un trapecio isósceles, la diferencia de las bases es de 10 m. La altura mide 12 m. y el perímetro
76 m. Entonces su área es:
A. 86 m
2
B. 176 m
2
C. 226 m
2
D. 288 m
2
E. 300 m
2
Como B – b = 10, al proyectar la base menor sobre la base mayor se
forman tres segmentos de longitudes 5, b y 5 como se muestra en la
figura. Luego como la altura es 12, en los extremos del trapecio se
forman triángulos rectángulos de catetos 5 y 12. Aplicando el teorema
de Pitágoras hallamos que la hipotenusa mide 13 lo cual corresponde a
la longitud de los lados no paralelos del trapecio. Considerando que el
perímetro mide 76 m. se tiene:
2b + 2 (13) + 2 (5) = 76  b = 20
Al considerar que B – b = 10, resulta B = 30
Aplicando la fórmula para el área de un trapecio, el área buscada resulta
    2
B b h 20 30 12
A 300 m
2 2
   
  
D E C
F
A B
A. 150° B. 135° C. 90°
D. 60° E. 45°
B C D
A F
E
En la figura,  ABCF es un paralelogramo. B, C y D son
colineales. Si AB = 18, AD = 30 y FE = 12. ¿Cuánto mide AE?
A. 10 B. 12 C. 15 D. 20 E. 25
20.
13
B
b
13
5 b 5
12
PROHIBIDA
LA
VENTA
GEOMETRÍA
14
22. En la figura ABCD es un cuadrado de lado 1 cm. y CE = 2 cm., entonces el área del triángulo ADF
en cm
2
es igual a
Dado que ABCD es un cuadrado AD y CE son paralelas, resultando que ADF  ECF por el teorema de
semejanza AA, ya que DAF  CEF por ser alternos internos entre paralelas y DFA  CFE por ser
opuestos por el vértice.
De la semejanza resulta que
AD DF 1 DF
CF 2DF
CE CF 2 CF
     (1)
Como CD = CF + FD = 1, resulta
1
DF
3
 .
Por tanto el área buscada resulta
1 1 1 1
[ADF] AD DF 1
2 2 3 6
      
23. Sea ABC un triángulo isósceles con AB = BC = 10 y AC = 16. Sea BD la mediana trazada sobre el
lado AC y sea G el baricentro. Entonces el área del triángulo ADG es
A. 6 B. 8 C. 10 D. 12 E. 24
Por ser BD mediana, D es punto medio de AC, o sea
AD = DC = 8.
Ya que ABC es isósceles, BD además de mediana
también es altura, luego mADB = 90.
Aplicando el teorema de Pitágoras hallamos que
2 2
BD 10 8 6  
Como G es el baricentro, BG = 2GD y como
BD = BG + GD = 6, resulta GD = 2.
Por tanto el ADG resulta ser un triángulo rectángulo con catetos de longitudes 8 y 2, por tanto su área
es
1 1
[ADG] AD DG 8 2 8
2 2
      
C 2B
A D
F
1
E
A.
2
1
B.
3
1
C.
4
1
D.
6
1
E.
8
1
B
G
A 8 D 8 C
10 10
16
2
PROHIBIDA
LA
VENTA
GEOMETRÍA
15
24. Sea ABC un triángulo isósceles con AB = AC = 17 cm y P un punto cualquiera del lado BC, diferente
de los puntos extremos. Por P se trazan una paralela a AC que corta a AB en Q y una paralela a AB
que corta a AC en R. El perímetro del cuadrilátero AQPR es
Dado que QP AC y RP AB , AQPR es un paralelogramo y de ahí AQ = RP y AR = QP.
Del paralelismo de los segmentos señalados anteriormente también resulta que los QBP y RPC son
semejantes con el ABC y por tanto también son isósceles y de ahí QB = QP y RP = RC.
Por tanto el perímetro del cuadrilátero AQPR, resulta
P = AQ + QP + AR + RP = (AQ + QB) + (AR + RC) = AB + AC = 17 + 17 = 34
25. De acuerdo a la información que se proporciona en la figura, el segmento de mayor longitud es
Dado que la suma de los ángulos internos de un triángulo suman 180, en el ABD, resulta que el ángulo
ABD mide 180 – 70 – 60 = 50 y en el BDC, mBDC = 180 – 55 – 60 = 65
Una de las propiedades de los triángulos indica que el lado mayor se opone al ángulo mayor y viceversa.
Al comparar las medidas de los ángulos del ABD, resulta que el mayor mide 70 y su lado opuesto es
BD, luego BD es mayor que AB y AD. Pero al considerar el BDC, su ángulo mayor es 65 y el lado que
se le opone es BC y por tanto BC > BD. Luego el lado mayor de la figura resulta BC
A 70°
B
55°
60°
60°
C
D
A
B P C
Q
R A. 8.5 cm B. 17 cm C. 34 cm D. 51 cm E. 68 cm
A. AB B. BC C. CD D. DA E. BD
PROHIBIDA
LA
VENTA
GEOMETRÍA
16
26. En la figura ABCD es un cuadrado de lado 1, CMN es equilátero, El área de CMN es igual a
El área de un triángulo equilátero está dada por 23
x
4
, donde x es la longitud de su lado, luego debemos
encontrar primero cuanto mide cada lado del triángulo equilátero CMN.
Como ABCD es un cuadrado y CM = CN = x, se tiene que CDM  CBN y de ahí MD = NB y como
AD = AB, resulta AM = AN y por tanto el MAN es rectángulo isósceles, luego
x
MN x 2 AN AN
2
   
Como AB = 1, resulta
x 2 x
NB 1 AN 1
2 2

    
El CBN es un triángulo rectángulo luego al aplicar el teorema de Pitágoras resulta
2
2 2 2 22 x
CN CB NB 1 x
2
 
     
 
 
Al desarrollar, simplificar y resolver la ecuación resultante se obtiene x 6 2 
Por tanto el área buscada es  
23
[CMN] 6 2 0.4641
4
  
27. La siguiente figura muestra dos cuadrados de lado 1 cm., donde AEFG se ha obtenido de ABCD al
girar este cuadrado 45° sobre el vértice A. Entonces el área sombreada es
Al girar 45, la recta diagonal AC se convierte en la recta AB la cual equivale a la recta diagonal AF, por
tanto A, B, F son colineales. Además se tiene mBFH = 45 y por tanto FBH es un triángulo rectángulo
isósceles con FB = BH
Luego [AGHB] = [AGF] – [FBH]
Por ser AEFG un cuadrado de lado 1, su diagonal mide 2 y como AB = 1, BF = BH = 2 1
Como [AGF] tiene como área la mitad de la área del cuadrado, que tiene lado de longitud 1, resulta
   
21 1 1 1
[AGBH] 2 1 2 2 2 1 2 1
2 2 2 2
        
A D
B H C
GE
F
A N B
CD
M
A. 0.866 B. 0.7071 C. 0.75
D. 0.5 E. 0.4641
A. 2 – 1 B. 0.5 C. 0.451
D. 2 E. 0.375
PROHIBIDA
LA
VENTA
GEOMETRÍA
17
28. Los ángulos agudos de un triángulo rectángulo, que también es isósceles, miden
A. 30° B. 45° C. 35° D. 75° E. 60°
Por ser triángulo rectángulo isósceles tiene un ángulo de 90 y los otros dos ángulos congruentes, y dado
que la suma de los ángulos internos de un triángulo suman 180, cada uno de ellos mide 45
29. En la figura ABCD es un cuadrilátero con AD ||BC . La diagonal AC es perpendicular al lado CD .
mBAC = 30°, AC = 4 3 y AB = BC. Entonces el área de ABCD es igual a
Como AB = BC, el ABC es isósceles con mABC = 120 y
mBCA= 30 y su base AC = 4 3 .
Al trazar una perpendicular desde B a AC, sea E el pie de la
perpendicular.
Por ser isósceles, BE también es mediana es decir E es punto medio
de AC, luego se forman dos triángulos 30 – 60 con las hipotenusas
AB = BC y catetos mayor AE = EC =
AC
2 3
2

Luego como el cateto mayor en un triángulo 30 – 60, es 3 veces el cateto menor, en este caso se tiene
BE 2
Como AD ||BC el BAD es el suplemento del ABC, resulta mBAD = 180 – 120 = 60 y como
mBAC = 30, se tiene mCAD = 30 y de ahí también el ADC es un triángulo 30 – 60 con cateto
mayor AC = 4 3 . Como en todo triangulo 30 – 60, el cateto mayor es 3 el cateto menor, se tiene
CD = 4
Finalmente tenemos [ABCD] = [ABC] + [ACD] =
1 1 1 1
AC BE AC CD 4 3 2 4 3 4 12 3
2 2 2 2
           
30. Se tiene un trapecio ABCD donde BC es la base menor. BC = 10 cm y CD = 20 cm . Las medidas
de los ángulos A, B y C son 30°, 150° y 120° respectivamente, entonces el área del trapecio mide
A. 300 3 cm
2
. B. 400 cm
2
. C. 300 cm
2
. D. 200 cm
2
. E. 200 3 cm
2
.
Sean B’ y C’ las proyecciones de B y C sobre la base mayor y sean
AB’ = x, C’D = y.
Por ser BC paralela a AD, mD = 180 – mC = 180 – 120 = 60
El CC’D es un triángulo 30 – 60, luego h = CC’ = 10 3 y
y = C’D = 10
También el AB’B resulta ser un triángulo 30 – 60, con su cateto menor BB’ = h = 10 3 , luego
10 3 3 30 
AB’ = 10 3 3 30 
Tenemos entonces que la base mayor mide AD = x + 10 + y = 30 + 10 +10 = 50.
Luego el área del trapecio resulta [ABCD] =
 50 10 10 3
300 3
2
 

A D
B C
A. 6 B. 12 C. 12 3 D. 24 E. 30
30
30
30
60
60
60
E
A D
B C
B 10 C
20
A x B’ 10 C’ y D
h
30 60
PROHIBIDA
LA
VENTA
GEOMETRÍA
18
31. En la figura, mBAC =  , mBPC = mBQC = 90°. Entonces la medida de BHC es
Como mBPC = mBQC = 90°, también mAPH = mAQH = 90°. APHQ es un cuadrilátero convexo y
en todo cuadrilátero convexo la suma de sus ángulos internos es 360,
luego mBHC +  + 90 + 90 = 360 y de ahí mBHC = 180 – 
32. Si las medianas en un triángulo rectángulo, trazadas a partir de los vértices de los ángulos agudos
miden 5 cm y 20 cm, entonces la medida en cm de la hipotenusa del triángulo rectángulo es
A. 5 B. 6 C. 8 D. 9 E. 10
Sean M y N los puntos medios de BC y AB respectivamente.
Sean AM = 20 y CN = 5, BC = a, AB = c, luego
a c
BM y NB
2 2
  .
Sea la hipotenusa AC = b
Aplicando el teorema de Pitágoras en los ABM y BCN
2
2 2 2 2 a
AM AB BM c 20
4
     (1)
2
2 2 2 2 c
CN NB BC a 25
4
     (2)
Al sumar (1) y (2) resulta
2 2
2 2 25c 5a 4
45 a c 45 36 b b 36 6
4 4 5
          
B C
P H Q
A

A. 180 –  B.  C. 90 – 
D. 2 E. 3
N
A
B M C
PROHIBIDA
LA
VENTA
GEOMETRÍA
19
33. En la figura, los dos cuadrados tienen el mismo centro. La razón entre el lado del cuadrado menor y el
lado del cuadrado mayor es 2/5. Entonces la razón entre el área sombreada y el área del cuadrado mayor
es
Sean “b” la longitud del lado del cuadrado menor y “a” la longitud del lado del cuadrado mayor,
luego
b 2
a 5

Por la simetría de la figura se deduce que el área sombreada, es decir el trapecio ABFE, representa la
cuarta parte de la diferencia entre los dos cuadrados, luego  
1
[ABFE] [ABCD] [EFGH]
4
 
2 2
[ABCD] a y [EFGH] b  y de ahí  2 21
[ABFE] a b
4
 
La razón buscada será
 2 2
2 2 2
2 2 2
1
a b
[ABFE] 1 a b 1 b4 1
[ABCD] 4 4a a a
    
      
   
   
Como
b 2
a 5
 , resulta
[ABFE] 1 4 21
1
[ABCD] 4 25 100
 
   
 
34. En la figura, AB = AC = 4, BD = DC = 3 y mBAC = 60°, entonces la longitud del segmento AD es
Al unir B con C, obtenemos un triángulo equilátero, ya que AB = AC y mBAC = 60°.
Se tiene que ABD  ACE, ya que sus tres pares de lados son congruentes, de ahí resulta
mBAD = mCAD y por tanto AD es bisectriz del BAC
Al prolongar AD, sea E el punto donde corta a BC. Luego como el ABC es equilátero, AE además de
bisectriz es mediatriz y por tanto AE  BC y BE = EC = 2. Resulta entonces que el BED es rectángulo,
con hipotenusa BD = 3 y un cateto, BE = 2. Por el Teorema de Pitágoras, 2 2
DE 3 2 5  
Por otro lado AE es una altura en un triángulo equilátero de lado 4 y por tanto AE 2 3
Finalmente obtenemos que AD = AE – DE = 2 3 5
A
CB E
D
A a B
D C
H G
E F
b
A. 1/6 B. 21/100 C. 1/3
D. 2/5 E. 4/9
A. 2 3 – 5 B. 2 3 + 5 C. 1
D. 2 E. 3.5
PROHIBIDA
LA
VENTA
GEOMETRÍA
20
35. En la figura el cuadrilátero ACDE es un trapecio tal que ED = 15 cm., AC = 24 cm y la altura es 12 cm.
Sabiendo que B es el punto medio del lado AC, el área del cuadrilátero OBCD es
Como ED || A C , resulta que ABO  DEO, con razón de semejanza
AB 12 4
DE 15 5
 
Sean a y b las alturas de los triángulos ABO y DEO respectivamente, indicadas en la figura. Dado que los
elementos homólogos en triángulos semejantes están en la misma razón de semejanza, se tiene
a 4
b 5
 .
Como a + b = 12 (la altura del trapecio), al considerar la razón anterior resulta
16 20
a , b
3 3
  .
Al analizar la figura vemos que [OBCD] = [ACDE] – [ABE] – [DEO]
Tenemos que el área del trapecio ACDE resulta
24 15
[ACDE] 12 234
2
 
   
 
El ABE, tiene base 12 y altura 12, luego su área es
1
[ABE] 12 12 72
2
   
Para el DEO, resulta
1 20
[DEO] 15 50
2 3
     [OBCD] = 234 – 72 – 50 = 112
36. En la figura, ABCD es un cuadrado de lado 6 cm. y CE = DE = 5 cm., entonces la longitud de AE es
A 109 cm B. 15 cm C. 11 D. 30 E. 61
Sean F y G los puntos medios de CD y BA respectivamente.
Luego CF = FD = BG = GA = 3 y FG = 6
Como CE = DE, el CED es isósceles y por tanto E, F, G son colineales y
EF  CD y EG  AB.
El CFE es rectángulo en F, luego por el Teorema de Pitágoras,
2 2
EF 5 3 4   . EG = EF + FG = 4 + 6 = 10
El FGA también es rectángulo con EG = 10 y GA = 4, luego 2 2
EA 10 3 109  
E 15 D
O
A 12 B 12 C
12
b
a
A. 112 cm
2
B. 117 cm
2
C. 120 cm
2
D. 140 cm
2
E. 360 cm
2
E
C F D
B G A
PROHIBIDA
LA
VENTA
GEOMETRÍA
21
37. En la figura, a partir de la información dada, ¿cuál es el valor de x?
A. 76 B. 25 C. 13.2 D. 5.28 E. 5
Se tiene A  E , por dato, y ACB  ECD, por ser opuestos por
el vértice, luego ABC  EDC, por el teorema de semejanza AA.
Luego
CD BC x 66
x 5
CE AC 10 132
    
38. ABCD es un paralelogramo. P es un punto de la diagonal AC . Trazamos por P paralelas a los lados
del paralelogramo. Estas paralelas intersecan a los lados del paralelogramo en los puntos indicados en la
figura. Sabiendo que el área de ABCD es 40 cm
2
, entonces el área del cuadrilátero RQMN es igual a
Dado que RM AD BC y NQ AB DC , resulta que los cuadriláteros ANPR, PQBR, DMPN y MCQP son
paralelogramos y los segmentos NR, RQ, QM y MN son diagonales de esos paralelogramos. Es sabido
que una diagonal divide a un paralelogramo en dos triángulos congruentes, con áreas igual a la mitad del
área del paralelogramo. Por tanto [RQMN] =
1
2
[ABCD] = 20
Basta aplicar el teorema del cateto: 2
3x 6 x 12  
A
B C
3
6
x
En el triángulo rectángulo ABC ¿cuál es la longitud del
segmento BC?
A. 15 B. 12 C. 10 D. 9 E. 7.5
39
A R B
N P Q
D M C
A. 10 cm
2
B 20 cm
2
C. 30 cm
2
D. 40 cm
2
E. 50 cm
2
x
10
52.8
132
66
A
B
C
E
D
PROHIBIDA
LA
VENTA
GEOMETRÍA
22
40. Sea ABCD un cuadrado. Por el vértice A se traza un segmento que corta a la prolongación del lado
BC en E, al lado DC en F y a la diagonal BD en G. Si AG = 3 y GF = 1 ¿cuál es la longitud de FE?
Sea x la longitud de cada lado del cuadrado. Desde G tracemos una perpendicular a AD y sea H el pie de
esta perpendicular. Luego AGH  AFD. Como AG = 3 y GF = 1, resulta AF = 4
De la semejanza se tiene
AG AH 3 AH 3
AH x
AF AD 4 x 4
     y HD = AD – AH =
3 1
x x x
4 4
  
Como G esta sobre la diagonal, HG = HD =
1
x
4
De la misma semejanza se tiene
x
AG HG 3 14 DF x
AF DF 4 DF 3
     .
Luego FC = DC – DF =
1 2
x x x
3 3
 
Como ABCD es un cuadrado, AD BC y por tanto AD CE . De ahí resulta que ADF  ECF
De esta semejanza se tiene
2
x
FE FC FE 3 FE 8
1FA FD 4
x
3
    
A H D
G
F
B C E
A. 12 B. 10 C. 9 D. 8 E. 6
PROHIBIDA
LA
VENTA
GEOMETRÍA
23
3. EJERCICIOS SOBRE CIRCUNFERENCIAS Y POLÍGONOS
Tenemos que
mAC mCD
2

 
Dado que mAB mBC mCD mAD 360 y mBC mAD 80 140 220           , tenemos que
mAC mCD 140   , luego
mAC mCD 140
70
2 2
 
    
2. El triángulo ABC está inscrito en un semicírculo de diámetro AB. Si AC = 8
y BC = 6, el área de la región sombreada tiene un valor de
A. 15.27 B. 24 C. 36.37 D. 61.07 E. 48
El área sombreada es la diferencia entre el área del semicírculo y el área del triángulo.
El ABC es rectángulo en C, por estar inscrito en un semicírculo.
Luego por el Teorema de Pitágoras, 2 2
AB 8 6 10    r = 5
Por tanto el área del semicírculo es 2 21 1 25
r 5
2 2 2

   
El área del triángulo está dada por
1 1
AC BC 8 6 24
2 2
     
El área buscada es A =
25
24 15.27
2

 
3. El triángulo ABC está inscrito en un semicírculo de diámetro AB. Si AC = 8 y CD = 4.8, el área de la
región sombreada tiene un valor de
A. 15.27 B. 24 C. 36.37 D. 61.07 E. 48
Por el Teorema de Pitágoras, 2 2
AD 8 4.8 6.4  
Como el ABC es rectángulo en C, se tiene por el teorema del cateto
2 2 64
AC AB AD 8 AB 6.4 AB 10
6.4
        . Y de nuevo por el Teorema de Pitágoras resulta
BC = 6. Dado que estos valores coinciden con los datos del ejercicio anterior, el área resulta la misma.

En la figura de la derecha si la medida de los arcos AD y BC son 140º y
80° respectivamente, entonces el valor de  es
A. 40° B. 50° C. 60° D. 70° E. 80°
A

B
C
D
1.
A D B
C
A B
C
PROHIBIDA
LA
VENTA
GEOMETRÍA
24
Sea  la medida del ángulo central XOZ. La longitud de un arco está dada por
s = r, con el ángulo medido en radianes.
Tenemos s =  y r = 2, luego
s
r 2

   , es decir 90
Luego XOZ es un triángulo rectángulo isósceles con XZ como hipotenusa y por
tanto XZ = 2 2
5. En la figura el área del círculo mayor es 1 m
2
. El círculo menor es tangente internamente al círculo
mayor y también es tangente a los lados del ángulo inscrito que mide 60°. El vértice del ángulo inscrito y
los centros de los círculos están alineados. Entonces el área del círculo menor es
Desde el vértice del ángulo inscrito, trazamos un diámetro. Sean O y O’
los centros de los círculos, mayor y menor respectivamente. Sea B el
otro extremo del diámetro trazado, C el punto donde uno de los lados
(el arriba) del ángulo corta a la circunferencia. Y sea D el punto de
tangencia de este lado del ángulo con el círculo menor.
Sean R y r los radios de los círculos, mayor y menor respectivamente.
Tenemos que AO’ biseca al ángulo inscrito, luego
mO’AD = mBAC = 30.
Como AB es un diámetro del circulo mayor, mACB = 90 resultando que mABC = 60, y el  ABC es
30 – 60.
Dado que AB = 2R, se obtiene que BC = R, ya que BC es el cateto menor y AB la hipotenusa del ABC
Como AC es tangente al círculo menor en D, AD  DO’, es decir mADO’ = 90 y de ahí mAO’D = 60
Luego también el AO’D es un triángulo 30 – 60 y su cateto menor O’D = r, mide la mitad de su
hipotenusa, AO’.
Se tiene O’B = r, por ser radio del circulo menor y de ahí AO’ = AB – O’B = 2R – r.
Luego AO’ = 2 O’D  2R – r = 2 r 
2
r R
3
 .
Como el área del círculo mayor es 2
R 1  , el área del círculo menor es
2
2 22R 4 4
r R
3 9 9
 
      
 
X
Y
Z
La circunferencia de la figura tiene radio 2 y el arco XYZ tiene
longitud . ¿Cuánto mide la cuerda XZ?
A. 2 B. 2 C. 2 2 D.
2

E. 
4.
A.
2
1
B.
9
4
C.  D. 2 E. 
2
1
.
X Z
O

C
A O O’ B
D
PROHIBIDA
LA
VENTA
GEOMETRÍA
25
Como TP es tangente a la circunferencia en T, mPTC = 90. Luego aplicando el teorema de Pitágoras
resulta  
2 2 2
PC 2r r 5r r 5   
Como el área sombreada únicamente son los extremos y estos son semicírculos, al unirlos se forma un
circulo de diámetro 8, es decir de radio 4, luego 2 2
A r 4 16     
Dado que m AB = 50°, se tiene mBCA =
1
mAB
2
= 25°, por ser ángulo inscrito que subtiende dicho
arco; mABC = 90, por estar inscrito en una semicircunferencia (
___
AC es diámetro). Luego la medida del
ángulo buscado es: mBAC = 180 – mBCA – mABC = 180 – 25 – 90 = 65
C
Q
PT
En la figura C es el centro de la circunferencia de radio r y
__
TP
es un segmento tangente en T, de longitud 2r, entonces PC
mide
A. r 2 B. r 3 C. 3r D. r 5 E. 5r
6.
r
2r
10
8
Los extremos de la figura son semicírculos, ¿Cuál es el
área de la región sombreada?
A. 80 B. 8 C. 10 D. 16 E. 16 + 80
7.
O
A
B
C
En la figura
___
AC es un diámetro. Si m AB = 50°, entonces m  BAC = ?
A. 25° B. 50° C. 65° D. 90° E. 130°
8.
PROHIBIDA
LA
VENTA
GEOMETRÍA
26
El área de la región sombreada es la diferencia entre el área del cuadrado formado y las áreas de los
cuatro sectores circulares que se forman.
Dado que la distancia entre los centros de dos círculos tangentes exteriormente, es la suma de las
longitudes de los radios, resulta que el cuadrado formado tiene lado de longitud 20 y por tanto el área del
cuadrado es 400.
Cada sector formado tiene un ángulo central de 90, luego entre los cuatro forman un circulo de radio 10,
cuyas áreas suman entonces 2 2
r 10 100    
Por tanto el área buscada es: A = 400 – 100
Dado que BPA es un ángulo exterior, formado por dos secantes, su medida es la semidiferencia de los
las medidas de los arcos que intercepta.
Es decir
mCD mAB
m BPA
2

  . De esta expresión despejamos mAB , resultando mCD 2 m BPA mAB   
Introduciendo los datos se obtiene mCD 2 35 30 100      
Por otro lado se tiene que el DAC es un ángulo inscrito que subtiende el arco DC, luego
1 1
m DAC mCD 100 50
2 2
      
En la figura, los círculos son tangentes y tienen radio igual a 10.
Si se unen los centros de los círculos se forma un cuadrado.
¿Cuál es el área de la región sombreada?
A. (400 – 100) B. 400 – 100 C. 100 – 400
D. 400 – 100 E. 400 – 400
9.
D B
P
A
C
En la figura, la medida del arco AB es 30°, y la medida del
BPA es 35°. Las medidas del arco CD y el ángulo DAC
(en grados) son respectivamente
A. 100 y 25 B. 50 y 50 C. 100 y 50
D. 50 y 25 E. 25 y 50
10.
PROHIBIDA
LA
VENTA
GEOMETRÍA
27
11. La expresión (p + q) p = (r + s) r, se cumple en la situación representada por
Al recordar las relaciones métricas en una circunferencia, vemos que los productos de esta forma surgen
cuando se tienen dos secantes que se cortan (también aparecen cuando hay semejanzas de triángulos) o
una secante y una tangente que se cortan.
A partir de estas relaciones tenemos:
En la figura A, la relación es  2
r s s p  
En la figura B, la relación es r  (r + s) = p  (p + q), la cual es la misma expresión dada. La respuesta es
ésta.
Para estar más seguros vemos que resulta en las otras.
En la figura C, la relación es r  s = p  q y en la figura C,  2
r p p q   , que son diferentes a la dada.
Solo B satisface y por tanto es la respuesta.
12. En la figura se dan tres semicircunferencias mutuamente tangentes. DAyCD son diámetros de las
circunferencias menores. El punto B está en la semicircunferencia mayor. BD  CA . Si BD = 2, entonces
el área sombreada es igual a
El área de la región sombreada es la diferencia entre el área del semicírculo exterior menos las áreas de
los semicírculos interiores.
Sean 1 2r , r , R los radios del semicírculo menor, del semicírculo mediano y del semicírculo exterior
respectivamente. Luego CD = 12r , DA = 22r y CA = 2 R
Como CA = CD + DA, se tiene 2R = 1 22r 2r o sea R = 1 2r r (1)
Al unir B con A y con C, se forma un triángulo rectángulo, con CA como hipotenusa y BD como altura
relativa a la hipotenusa.
Por el teorema de la altura, 2 2
1 2 1 2BD CD DA 2 2r 2 r r r 1        (2)
Como el área de un semicírculo está dada por 21
r
2
 , el área buscada es  2 2 2
1 2
1
A R r r
2
   
Al considerar (1)  
2 2 2 2
1 2 1 2 1
1 1
A [ r r r r )] r
2 2
       2
1 2 22r r r  2
1r 2
2r  1
2
 2 1 2r r
Al considerar (2) 1 2A r r  
r
r
r
r
s s
s
p
p p p
sq
q
q
q
A B C D
C D A
B
A. 1 B.  C. 2 D.
4
3
E.
4
9
PROHIBIDA
LA
VENTA
GEOMETRÍA
28
13. Las medidas de los arcos AB y AC se indican en la figura. La medida del BAC es
El BAC es un ángulo inscrito en una circunferencia, por tanto su medida es la mitad de la medida del
arco que subtiende, en este caso el arco BC.
Tenemos que mBC 360 mAB mAC 360 110 130 120            , luego mBAC =
1
mBC 60
2
 
14. En la figura, BC une los centros de los círculos tangentes. AB  BC , BC = 8 y AC = 10, entonces
la longitud de la circunferencia pequeña es igual a
Sean R y r los radios de las circunferencias grande y pequeña respectivamente. Como las circunferencias
son tangentes exteriormente, R + r = BC = 8
Dado que el ABC es rectángulo en B, tenemos 2 2
R AB 10 8 6    y r = 2.
Luego la longitud de la circunferencia pequeña resulta C = 2  r = 4 
Todo hexágono regular puede dividirse en seis triángulos equiláteros congruentes. En la figura se indica
que x equivale al doble de la altura de cada triangulo: x = 2h
Como el lado de cada triangulo mide 6 3 , las alturas miden
3
h 6 3 9 x 2h 18
2
     
x
36
15.
A
B C
A
B
C
130°
110°
A. 55° B. 60° C. 65° D. 110° E. 130°
A.  B. 2 C. 3 D. 4 E. 5
La figura representa un hexágono regular, ¿cuál es el valor de x?
A. 3 3 B. 6 3 C. 6 D. 18 E. 9 3
PROHIBIDA
LA
VENTA
GEOMETRÍA
29
Si llamamos 1A al área del cuadrado mayor, 2A al área del cuadrado menor y 3A al área del círculo, el
área de la región sombreada resulta 1 2 3A A A A  
El lado del cuadrado mayor mide 0.4, luego su área es 1A = 0.16
Como B es punto medio AB = 0.2. Los triángulos que se forman en cada esquina del cuadrado mayor,
son rectángulos isósceles, y sus hipotenusas forman los lados del cuadrado menor, por tanto, el lado del
cuadrado menor resulta 0.2 2 y su área es 2A =  
2
0.2 2 0.08
Como el circulo está inscrito en el cuadrado menor, su diámetro es el lado de dicho cuadrado, y su radio
es la mitad o sea r = 0.1 2 , su área  
2
2
3A r 0.1 2 0.0628    
Luego A = 0.16 – 0.08 +0.0628 =0.1428
Dado que PB y PC son segmentos tangentes a la circunferencia de la izquierda, desde un mismo punto,
son congruentes, luego PC = PB = 19.
Como AC = AP + PC, AP = AC – PC = 31 – 19 = 12.
18. Seis triángulos equiláteros de 1 cm. de lado se unen para formar un hexágono como se muestra en
la figura. Se circunscribe un círculo alrededor del hexágono ¿cuál es el área de la región sombreada?
Tenemos que el área de la región sombreada es el área del circulo menos el área del hexágono.
El radio del circulo es la longitud del lado de los triángulos, es decir r = 1, luego su área es 2
r  
El área de cada triángulo equilátero es 23
x
4
, donde x es el lado del triángulo, y como el lado mide 1, se
reduce a
3
4
. Como hay seis triángulos, el área del hexágono es
3 3 3
6
4 2
 
Por tanto el área de la región sombreada es A = )
2
3
(  cm
2
A
B
P
D
C
Los segmentos AC y BD se cortan en P y son tangentes a
las circunferencias en los puntos A, C, B y D.
Si AC = 31, PB = 19 ¿Cuál es el valor de AP?
A. 6 B. 12 C. 15 D. 25 E. 50
17.
0.4
La figura representa un círculo inscrito en un cuadrado que a su
vez está inscrito en otro cuadrado. B es punto medio de AC
¿Cuál es el área de la región sombreada?
A. 0.025 B. 0.048 C. 0.1428 D. 0.153 E. 0.1582
A
B
C
A. )
2
3
(  cm
2
B. )
2
33
(  cm
2
C. )
2
3
( 2 cm
2
D.
3
3
cm
2
E. )3( 32  cm
2
16.
PROHIBIDA
LA
VENTA
GEOMETRÍA
30
19. Un triángulo ABC está inscrito en una circunferencia como se muestra en la figura.
Se tiene m A = 50º y m C = 60º. Se trazan tangentes por A, B y C de manera que se forma el triángulo
circunscrito A’B’C’. Entonces la medida del ángulo A’ es:
A. 40º B. 60º C. 80º D. 100º E. 120º
Como BA’ y CA’ son tangentes a la circunferencia, los A’BC y
A’CB son ángulos semiinscritos que subtienden el arco BC y el
ángulo A es un ángulo inscrito que subtiende el mismo arco. Por tanto
estos ángulos son congruentes, es decir
mA’BC = mA’CB = mA = 50
Luego al considerar el A’BC, se tiene
mA’ = 180 – mA’BC – mA’CB = 180 – 50 – 50 = 80
20. El triángulo ABC es equilátero y sus lados AC y BC son tangentes a la circunferencia con centro en
O y radio 3 . El área del cuadrilátero AOBC es
A. 3 B. 6 C 3 3 D. 6 E. 12
Se tiene OC  AB, ya que los triángulos OAB y ABC son isósceles.
También OAC  OBC, ya que sus tres lados son congruentes. Como
además el ABC es equilátero, mACO = 30, luego el OAC es un
triángulo 30 – 60 y de ahí resulta que OC = 2 3 y AC = 3.
Tenemos entonces [AOBC] = 2[OAC] =
1
2 3 3 3 3
2
   
21. Si un ángulo central de 30° en una circunferencia intercepta un arco de 6 m de longitud, entonces el
radio de la circunferencia mide
A. /36 B. /6 C.  D. 36/ E. 180
Se tiene s = r , donde s es la longitud del arco, r el radio de la circunferencia y  es el ángulo central
correspondiente, medido en radianes. Como  = 30 equivale a /6 radianes, tenemos
36
6 r r
6

   

C
C’ B
B’
A
A’
50
A
B
O C
3
PROHIBIDA
LA
VENTA
GEOMETRÍA
31
22. En la figura se tiene una circunferencia de radio 1 y un hexágono regular de lado 1. Si O es el centro
de la circunferencia, entonces el área de la región sombreada es
A. 0.5 B. 0.866 C. 1 D. 1.5 E. 2
En vista que el hexágono tiene lado 1 y la circunferencia tiene radio 1, el
centro del hexágono es un punto de la circunferencia. La región
sombreada puede descomponerse en dos triángulos que tienen la misma
base y la misma altura que los triángulos que forman el hexágono.
Luego el área buscada es A = 2 23 3
1 0.866
4 2
  
Sean 1r el radio del semicírculo mayor, 2r el radio del semicírculo mediano y 3r el radio del semicírculo
menor. Se tiene 1 2 2 1 1
AB 3
r , BC 2r 2AB r AB, 2r AB BC 3AB r AB
2 2
         
Luego las áreas de estos semicírculos son:
Semicírculo mayor:
2
2 2
1
1 1 3 9
r AB AB
2 2 2 8
 
     
 
Semicírculo mediano: 2 2
2
1 1
r AB
2 2
  
Semicírculo menor:
2
2 2
3
1 1 AB 1
r AB
2 2 2 8
 
     
 
El área no sombreada está dada por: área del semicírculo mayor más el área del semicírculo mediano
menos el área del semicírculo menor o sea
Área no sombreada = 2 2 2 29 1 1 3
AB AB AB AB
8 2 8 2
      
El área sombreada está dada por: área del semicírculo mayor menos el área del semicírculo mediano
más el área del semicírculo menor o sea
Área sombreada = 2 2 2 29 1 1 3
AB AB AB AB
8 2 8 4
      
La razón buscada resulta
2
2
3
AB
Area sombreada 14
3Area no sombreada 2
AB
2

 

A B C
Los arcos AB y BC son semicírculos cuyos centros están sobre un
diámetro del círculo que se muestra en la figura.
Si BC = 2 AB, entonces la razón entre el área de la región sombreada
y el área de la región no sombreada es:
A. 2 B.
2
3
C. 1 D.
3
2
E.
2
1
23
O’ O A
C
B PROHIBIDA
LA
VENTA
GEOMETRÍA
32
24. Una moneda circular de radio 1, está sobre una mesa. Si ponemos cuatro monedas más grandes de
igual tamaño alrededor de ella, ¿cuál es el radio de las monedas grandes que permite que cada una sea
tangente a las dos adyacentes y a la de radio 1?
A. 1 B. 1 + 2 C. 2 D. 2 + 2 E. 2.5
Sea R el radio de las monedas grandes. Como estas monedas son tangentes
a las monedas adyacentes y a la vez son tangentes a la moneda pequeña, al
unir los centros de las monedas grandes se forma un cuadrado de lado 2R.
Al trazar una diagonal, esta debe pasar por el centro de la moneda pequeña,
la cual tiene diámetro 2, luego la longitud de la diagonal resulta 2R + 2.
Por tanto, dado que en todo cuadrado de lado x, su diagonal mide 2 x , se
cumple en este caso que
Al racionalizar el denominador obtenemos R 2 1 
25. En la siguiente figura ABC y AEB son semicírculos, F es el punto medio del diámetro AC, B es punto
medio del arco AC y AF = 1¿Cuál es el área de la región sombreada?
El área de la región sombreada resulta de la diferencia entre el semicírculo AEB y el segmento circular
determinado por la cuerda AB en el semicírculo ABC.
Como F es el punto medio del diámetro AC, B es punto medio del arco AC, resulta BF  AC, luego el
ABF es un triángulo rectángulo isósceles de cateto 1 y por tanto AB = 2
AB es diámetro del semicírculo AEB, luego su radio es
2
2
y el área de este semicírculo resulta
2
1
1 2
A
2 2 4
  
    
 
 
El área del segmento circular, está dada por la diferencia entre el área del sector circular que lo contiene
y el área del triángulo determinado por la cuerda y los radios extremos.
En este caso el sector circular correspondiente tiene ángulo central de 90 y radio 1, por tanto su área es
la cuarta parte del área de un círculo de radio 1 o sea
1
4
 y el triángulo correspondiente tiene base 1 y
altura 1, luego su área es
1
2
. El área del segmento circular resulta 2
1 1
A
4 2
  
Finalmente el área buscada es 1 2
1 1
A A A
4 4 2 2
  
      
 
A. 1/2 B. 2 C. /4 D. 3/4 E. /4 – 1/2
A
B
F C
E
R R
R
2
R
R
R
    1
2R 2 2 2R 2R 2 2 2 R 2 2 2 R 2 R
2 1
         

PROHIBIDA
LA
VENTA
GEOMETRÍA
33
26. Si el radio de un círculo aumenta en  unidades, ¿cuánto aumenta su perímetro?
A.  B. 2 C. 3 D. 
2
E. 2
2
Sean L y L’ los perímetros del círculo original y el círculo con el radio aumentado, respectivamente.
Luego   2
L 2 r y L' 2 r 2 r 2         
El aumento es la diferencia  = L’ – L =  2 2
2 r 2 2 r 2      
27. Dos semicírculos de radio 3 están inscritos en un semicírculo de radio 6 como se muestra en la
figura. Un círculo de radio r es tangente a los tres semicírculos. ¿Cuánto vale r ?
Cuando se tienen círculos tangentes exteriormente, la distancia entre los centros es la suma de los
radios, y cuando son tangentes interiormente, la distancia entre los centros es la diferencia entre los
radios. Además en ambos casos los centros y el punto de tangencia están alineados.
Sean A, B, C y D los centros de los semicírculos y del círculo interior como se muestra en la figura.
Se tiene AB = AD = 3 + r, CA = 6 – r, BC = CD = 3. Como ABD es isósceles y C es punto medio de BD,
AC  BC, luego el ABC es rectángulo en C y por tanto sus lados cumplen con el teorema de Pitágoras.
Luego    
2 2 2
3 r 6 r 3 9     2
6r r  2
36 12r r   9 18r 36 r 2   
r
r r
A
3 3
B C D
A
B 3 C
3 + r 6 – r
D3 3
r
A. 1 B. 1.5 C. 2 D. 2.5 E. 3
r r + 
PROHIBIDA
LA
VENTA
GEOMETRÍA
34
Como el ABC es rectángulo en C, aplicamos el Teorema de Pitágoras para hallar BC:
2 2 2 2
BC AB AC 5 3 4    
Como BC es diámetro del círculo, se tiene r = 2 y su área resulta 2
A r 4   
30. El lado mayor del rectángulo de la figura mide 20. La curva trazada en su interior está formada por
cinco semicircunferencias ¿cuál es la longitud de la curva?
Se observa que la curva está formada por 5 semicircunferencias, cuyos diámetros suman 20, luego cada
diámetro mide 20  5 = 4 y los respectivos radios la mitad o sea 2 unidades.
Luego L =
1
5 2 r 5 r 5 2 10
2
        
Al considerar el círculo central y dos círculos externos
contiguos, vemos que encierran la sexta parte del área
buscada. Vemos también que esta fracción corresponde al
área de un triángulo equilátero de lado 2 menos tres
sectores circulares de radio 1 y de 60º cada uno, que
juntos forman un semicírculo de radio1.
Luego
A = 6 [    2 23 1
2 1
4 2
  ] = (6 3 – 3) u
2
En la figura los círculos adyacentes son tangentes y tienen
radio 1. ¿Cuánto vale el área de la región sombreada?
A. 6 3 – 3 B. 3 3 – 2 C. 2 – 1
D. 6 3 – 1 E. 6 – 3
28.
B O C
A En la figura, m  BCA = 90º, BA = 5 y AC = 3. ¿Cuál es el área del
círculo con centro en O?
A. 16 B. 8 C. 6 D. 5 E. 4
29.
A. 25  B. 20 C. 15 D. 10 E. 5
PROHIBIDA
LA
VENTA
GEOMETRÍA
35
Sea r el radio de la circunferencia buscado.
Sean A y C los centros de las circunferencias, pequeño y grande
respectivamente.
Desde A y C trazamos perpendiculares a los segmentos
perpendiculares iniciales, formando el cuadrado rotulado en la
figura como ABCD.
Sean E, F, G y H los puntos donde estas perpendiculares cortan a
los segmentos perpendiculares, como se indica en la figura.
Tenemos que AE = AG = DH = BF= 1, el radio de la circunferencia
pequeña.
Como CH = BG = CF = r, tenemos que
CD = CB = BA = DA = r – 1, luego por esto y la perpendicularidad
anterior ABCD es un cuadrado.
Como las circunferencias son tangentes exteriormente, la distancia entre sus centros es la suma de sus
radios, es decir AC = r + 1.
Luego el ABC es un triángulo isósceles, rectángulo en B, con AB = BC = r – 1 y AC = r + 1.
Luego AC = 2 AB, es decir r + 1 = 2 (r – 1).
Al despejar r, se obtiene r = 3 + 2 2
32. Tres círculos de radio 1, con sus centros colineales son tangentes como se muestra en la figura.
¿Cuál es el área de la región sombreada?
Rotulemos los puntos extremos de la región sombreada, como se muestra en la figura, vemos que se
forma un rectángulo. En los extremos de la región se tienen dos semicírculos, que juntos forman un
circulo. Luego la región sombreada es la diferencia entre las áreas del rectángulo y los dos círculos que
se forman.
Dado que el radio de los círculos es 1, AD = 2 y AB = 4.
Luego A = 2
A 2 4 2 1 8 2       
A. 8 – 2 B. 4 –  C. 12 – 3 D. 8 – 3 E. 4 + 
A 4 B
D C
2
La figura muestra dos segmentos perpendiculares tangentes a
ambas circunferencias, las cuales son tangentes entre sí. Si el radio
de la circunferencia pequeña mide 1, entonces el radio de la
circunferencia más grande mide
A. 3 + 2 2 B. 4 C. 6 D. 4 + 2 2 E. 8
31.
G A B
H D C
E F
PROHIBIDA
LA
VENTA
GEOMETRÍA
36
33. La figura muestra un hexágono regular inscrito en un círculo. Si el área del círculo es 1, ¿cuánto
mide el área del triángulo ABC?
Se observa que los triángulos ABC y ABO tienen la misma área, ya que tienen la misma base y la misma
altura. Por ser un hexágono regular el ABO es un triángulo equilátero de lado igual al radio del círculo.
Como el área del circulo es 1, se tiene 2 1
r 1 r   

Luego el área del triángulo es
2
3 1 3
[ABC]
4 4
 
  
 
34. ¿Qué polígono regular tiene la misma cantidad de diagonales que de lados?
A. Pentágono B. Hexágono C. Octógono D. Decágono E. Dodecágono
Como el número de diagonales en un polígono está dado por
 n n 3
D
2

 , donde es el número de lados
del polígono.
Luego
 
 2 2
n n 3
n n 3n 2n n 5n 0 n n 5 0 n 5
2

            . Se descarta n = 0, por
carecer de sentido.
Por tanto el polígono buscado es un pentágono.
35. Sean O el centro de una circunferencia de radio r y ED = r. Si mDEC = k  (m BOA), entonces
el valor de k es:
Trazamos el radio OD y vemos que el ODE es isósceles ya que OD = ED = r, luego DEC  DOC.
Como elDEC es un ángulo exterior con sus lados secantes a la circunferencia, su medida está dada por
mAB mCD
m DEC
2

  . (1)
Como los ángulos BOA y DOC, sus medidas están dadas por m BOA mAB y m DOC mDC m DEC     
Se tiene mDEC = k  (m BOA) = k mAB mDC  
Sustituyendo en (1):
mAB k mAB 1
k mAB 2k mAB mAB k mAB 3k 1 k
2 3
 
          
A O C E
B
D
A.
3
1
B.
2
1
C. 1 D. 2 E. 3
A B
C
O
 A.
6
1
B.
6

C.
4
3
D.
4
3
E.
12

PROHIBIDA
LA
VENTA
GEOMETRÍA
37
36. Si se aumenta el radio de un círculo en un 100%, ¿en qué porcentaje aumenta su área?
A. 50% B. 100% C. 200% D. 100% E. 300%
Si el radio original es r, el circulo con el radio aumentado, tiene radio 2r.
Se tiene  
22 2
1 2A r y A 2r 4 r     
El aumento está dado por 2 2 2
2 1A A A 4 r r 3 r        
Porcentaje de aumento:
2
1
3 rA
100%
A

 
2
r
100% 300% 
37. Se tienen tres círculos concéntricos de radios 1, 2 y 3 respectivamente. ¿Cuál es la razón entre el
área de la región cuadriculada y el área de la región oscura?
El circulo pequeño tiene área , ya que su radio es 1.
El círculo mediano tiene área 4, ya que su radio es 2
El círculo grande tiene área 9, ya que su radio es 3.
.
Área de la región oscura = área del circulo grande – área del circulo mediano = 9 – 4 = 5
Área de la región cuadriculada = área del circulo mediano – área del circulo pequeño = 4 –  = 3
área de la región cuadriculada 3 3
área de la región oscura 5 5

 

38. El segmento AB es diámetro de una circunferencia de radio 1 y lado del triángulo equilátero ABC. Si
la circunferencia corta a AC y BC en los puntos D y E respectivamente, entonces la longitud AE es:
A. 1 B. 3 C.
2
3
D.
3
5
E.
2
32 
Como mAEB = 90, AE es una altura del triángulo equilátero
ABC. Como el radio es 1, AB = 2, luego AE =
3
2 3
2
 
(En todo triángulo equilátero de lado x, la altura mide
3
x
2
 )
A.
3
2
B.
5
3
C.
9
4
D.
25
9
E. 2
C
A O B
D E
PROHIBIDA
LA
VENTA
GEOMETRÍA
38
39. En una circunferencia se tienen dos cuerdas paralelas de longitudes 10 y 14 que distan 6 entre sí.
Entonces la longitud de la cuerda paralela a ambas y que equidista de ellas mide:
A. 11 B. 12 C. 13 D. 184 E. 192
Sean CD = 10 y AB = 14, las cuerdas dadas. Como la distancia
entre ellas es 6, la cuerda paralela equidistante de ellas está a 3
unidades de cada una. Sea EF la cuerda buscada. Inicialmente no
sabemos la posición de las cuerdas con respecto a un diámetro
paralelo a ellas. Comencemos asumiendo que están al mismo
lado del diámetro paralelo, como se muestra en la figura.
Al trazar desde el centro una perpendicular a las cuerdas, esta
pasa por el punto medio de cada cuerda. Sean P, Q, R los puntos
medios de las cuerdas, como se muestra en la figura.
Se tiene PB = 7, RD = 5. Sea QF = y, la longitud de la cuerda
buscada es EF = 2y
Supongamos que la cuerda AB está a x unidades del centro. Se forman tres triángulos rectángulos, todos
ellos con hipotenusa igual al radio de la circunferencia.
Al aplicar el teorema de Pitágoras en cada uno ellos se forma el siguiente sistema de ecuaciones
2 2 2 2
2 2 2
2 2
r x 12x 36 25 r x 12x 61
r x 6x 9 y
r x 49
       
   
 
Restando la tercera ecuación de la primera
2 2
2 2
r x 12x 61
r x 49
  
 
 12x = – 12  x = – 1
El valor negativo de x, nos indica que las cuerdas están en lados opuestos del diámetro paralelo a las
cuerdas.
Sustituyendo el valor de x en la tercera ecuación, obtenemos 2
r 50
Sustituyendo el valor de x y 2
r en la segunda ecuación obtenemos
50 = 1 – 6 + 9 + 2
y  y 46 y EF = 2y = 2 46 184
R 5 D Q y F P 7 B
O
O
O
x
x +3 r r
x + 6 r
C R 5 D
E Q y F
A P 7 B
x
O
3
3
PROHIBIDA
LA
VENTA
GEOMETRÍA
39
40. Un triángulo equilátero y un hexágono regular están inscritos en el mismo círculo. Si se divide el área
del hexágono entre el área del triángulo se obtiene:
A. 1.5 B. 2 C. 2.5 D. 3 E. 3
Al observar el grafico fácilmente se deduce que el área del hexágono es el doble del área del triángulo.
Esto puede verificarse considerando que el lado de un triángulo equilátero inscrito en un círculo de radio
r, está dada por 3 r y por tanto su área es  
2
2
1
3 3 3
A 3 r r
4 4
 
También se tiene que el lado de un hexágono inscrito es igual al radio de la circunferencia, luego su área
es 2
2
6 3
A r
4
  , luego
2
2
1 2
6 3
rA 4 2
A 3 3
r
4

 

41. El área del círculo circunscrito a un hexágono regular es 2 cm
2
. Entonces el área del hexágono, en
cm
2
es
A. 6 B. 3 3 C. 4 2 D. 2 6 E.
2
63
Tenemos que el radio del círculo es 2 2
r 2 r 2    
Como el hexágono está inscrito su lado mide r, luego su área es
6 3
A 2 3 3
4
  
42. En una circunferencia se trazan tres cuerdas con las siguientes longitudes: C1: 3.05, C2: 3.50 y
C3: 0.305. ¿Cuál de las siguientes es una lista de las cuerdas en el orden en que se incrementa la
distancia desde el centro de la circunferencia a la cuerda?
A. C1C2C3 B. C3C1C2 C. C2C3C1 D. C2C1C3 E. C3C2C1
La cuerda de mayor longitud está más cerca del centro y la de menor longitud está más alejada.
Luego como 2C 3.5 es la que está más cerca y le sigue 1C 3.05 y la más alejada es 3C 0.305
Por tanto el orden en que se incrementa la distancia es 2 1 3C C C
PROHIBIDA
LA
VENTA
GEOMETRÍA
40
43. En la figura, ABDE es un cuadrado de lado 1.
Los arcos EB y DC tienen centro en A.
Entonces el área sombreada mide:
A.
2
1
B.
42
1 
 C.
42
1 
 D.
8


2
1
E.
4

1
La región EDB es la diferencia entre el área del cuadrado y el área del
cuadrante ABE, es decir [EDB] = 2 2
1 1 1
4 4
 
   
El área de la región DBC es la diferencia entre el sector circular ACD y el
triángulo rectángulo ABD, cuya área es
1
2
.
El sector circular tiene radio AD, el cual es la diagonal del cuadrado y por
tanto AD = 2 y su ángulo central es 45, es decir la octava parte de un circulo de
radio 2 .
Luego [BCD] =  
2 1 1
2
8 2 4 2
 
  
Finalmente se tiene [EBCD] = [EDB] + [BCD] =
1 1
1
4 4 2 2
    
      
   
Si tomamos tres vértices como se indica en la figura, se forma un triángulo rectángulo con catetos AB = 8
y AC = 24, la hipotenusa es el diámetro del círculo.
Por el teorema de Pitágoras 2 2
BC 8 24 8 10   , luego r = 4 10 y el área del círculo es 2
r 160  
El polígono está formado por 5 cuadrados de lado 8 y su área es 2
5 8 320 
Luego el área sombreada es 160 – 320 = 160 ( – 2)
El polígono de la figura tiene todos sus lados congruentes
de longitud 8, sus lados consecutivos son perpendiculares
y ocho vértices están sobre la circunferencia. Entonces el
área de la región sombreada mide
A. 144 – 320 B. 160 ( – 2) C. 144 – 320 D. 320(2 – 1) E. 80( – 2)
44.
A B
C
A B C
E D
A B C
D
PROHIBIDA
LA
VENTA
GEOMETRÍA
41
Al tomar el centro del semicírculo grande (A), el de un semicírculo mediano (B) y de un círculo pequeño
(C), se forma un triángulo, en el cual AB = 2, BC = 2 + r y AC = 4 – r
Al considerar el punto de tangencia entre los círculos pequeños, por la simetría de la figura CT es
paralela al diámetro y mCTA = 90, formándose el triángulo rectángulo ATC con AT igual a la altura del
ABC, CD: AT = CD y CT = r. De ahí resulta que ACT  CAD (teorema hipotenusa cateto), luego
AD = CT = r. Se tiene AB = 2 y DB = 2 – r
Por el Teorema de Pitágoras  
2 2
AT 4 r r 16 8r CD     
En el CDB se tiene    
2 2
2 r 16 8r 2 r 4      2
4r r  16 8r 4   2
4r r  16r 16 r 1   
46. El ABC es un triángulo rectángulo con el ángulo recto en C. Se traza la altura relativa a la
hipotenusa y se inscriben dos circunferencias en los triángulos que se forman. Si los radios de estas
circunferencias son 2 y 4 respectivamente, calcule el radio de la circunferencia inscrita en el ABC.
Como el ABC es rectángulo en C y CD es la altura relativa a
la hipotenusa, se tiene que ABC  ACD  CBD.
Como en los triángulos semejantes, todos los elementos
homólogos están en la misma razón de semejanza en la que
están los triángulos, la razón de semejanza entre los
triángulos CBD y ACD es la misma razón entre los radios de
sus circunferencias inscritas, que en este ejercicio es 4 a 2,
A D B
C
A. 52 B. 3 C. 25 D. 6 E. 8
45. Dos semicírculos de radio 2 están inscritos en un semicírculo de
radio 4 como se muestra en la figura. Dos círculos de radio r, son
tangentes a dos semicírculos y entre ellos. ¿Cuánto vale r ?
r
A.
2
1
B. 1 C.
2
3
D. 2 E.
2
5
C
A B
T
A D B
T r C
A
4 – r
4 – r 2 + r
C C
D B
2 + r
2 – r
A D B
C
PROHIBIDA
LA
VENTA
GEOMETRÍA
42
es decir 2 a 1. De manera que todos los elementos del CBD son el doble de los elementos
correspondientes del ACD.
Dado que las áreas están en proporción según el cuadrado de la razón de semejanza, se tiene
[CBD] = 4 [ACD] y como [ABC] = [ACD] + [CBD] = 5 [ACD], la razón de semejanza entre los triángulos
ABC y ACD es 5 y por tanto el radio de su circunferencia inscrita es 5 veces el radio de la
circunferencia inscrita o sea 2 5
Al unir los centros de los círculos se forma un triángulo equilátero de lado 2. Observamos que h es igual a
dos veces el radio de los círculos más la altura del triángulo equilátero, la cual como el lado mide 2, es
igual a 3 , luego h = 2 + 3 .
48. Tres semicírculos iguales, de radio R, tienen sus centros en los puntos colineales A, B y C tales que
cada uno de estos puntos se encuentra sobre uno de los semicírculos. Se traza un cuarto círculo con
centro D (el círculo sombreado) tal que es tangente a los tres semicírculos, tal como se muestra en la
figura. Si r es el radio del círculo pequeño, la razón R a r es:
A. 4:1 B. 15:4 C. 11:3 D. 10:3 E. 3:1
Al considerar el ABD, se tiene un triángulo rectángulo con AB = R, BD = R – r y AD = R + r. Por el
teorema de Pitágoras,
   
2 22 2 2 2 2
AD AB BD R r R R r R        2
2Rr r  2
R 2 2
R 2Rr r  
2 R
4Rr R 4
r
   
El área sombreada es la diferencia entre el área del triángulo y el
segmento circular que forma por la intersección entre el triángulo y el
círculo. Este segmento circular tiene un ángulo central de 60, ya que el
ángulo inscrito mide 30, y un radio de 6, ya que el OAB es equilátero.
A B C
D
30º 6
8
8
49. ¿Cuál es el área de la región sombreada,
redondeada al entero más cercano?
A. 12 B. 16 C. 19 D. 22 E. 24
h
47. Si cada círculo tiene radio 1, y son tangentes entre si
y a las líneas paralelas como se muestra en la figura
¿Cuánto mide h?
A. 3 B. 3 2 C. 4 D. 2 + 3 E. 3 3
r
r
30º 6
8
8
60º
A
B
O
PROHIBIDA
LA
VENTA
GEOMETRÍA
43
El área del segmento circular es el área del sector circular menos el área del triángulo equilátero:
2 21 3
6 6 6 9 3
6 4
      
Para el área del triángulo usamos la fórmula de Herón:    A s s a s b s c     , donde s es el
semiperímetro. Tenemos
6 8 8
s 11
2
 
  , luego    A 11 11 6 11 8 11 8 3 55     
Luego el área buscada es 3 55 – ( 6 9 3  ) = 18.9885  19
50. Los semicírculos de la figura tienen su centro sobre el segmento AB. Si el segmento CD es paralelo
al segmento AB y CD = 24, ¿Cuál es el área de la región sombreada?
Sean R y r los radios del semicírculo grande y del semicírculo interior respectivamente.
El área buscada es la diferencia entre las áreas de los semicírculos o sea  2 2
R r
2


Sea O el centro del semicírculo y E el punto medio de la cuerda CD. Luego OE  CD, OE = r y ED = 12.
El OED es rectángulo en E y OD = R. Por el teorema de Pitágoras 2 2 2
R r 12 144  
Luego A =  2 2
R r 144 72
2 2
 
    
Soluciones
1 D 11 B 21 D 31 A 41 B
2 A 12 B 22 B 32 A 42 D
3 A 13 B 23 E 33 C 43 A
4 C 14 D 24 B 34 A 44 B
5 B 15 D 25 A 35 A 45 D
6 D 16 C 26 E 36 E 46 A
7 D 17 B 27 C 37 B 47 D
8 C 18 B 28 A 38 B 48 A
9 B 19 C 29 E 39 D 49 C
10 C 20 C 30 D 40 B 50 A
C D
A O B
A. 72 B. 108 C. 144 D. 288 E. 576
E
r
PROHIBIDA
LA
VENTA
GEOMETRÍA
44
4. CUERPOS SOLIDOS
Como las bases son triángulos equiláteros de perímetro 30 cm, sus lados miden 10 cm y por tanto tienen
una área de 2
b
3
A 10 25 3
4
   . Su área lateral es LA P h 30 10 300    
El área total está dada por T b LA 2 A A 2 25 3 300 50 3 300       
2. Tres vértices de un cubo, de los cuales no hay dos que estén en la misma arista, se unen para formar
un triángulo. Si la arista del cubo tiene longitud 1, ¿Cuál es el área del triángulo formado?
A.
2
6
B.
2
3
C.
2
2
D.
4
6
E.
4
3
En la figura se muestra un triángulo que satisface el enunciado. Vemos que sus
lados son diagonales de las caras y como las aristas de los cubos tienen longitud 1,
estas diagonales miden 2 . Como el área de un triángulo equilátero de lado x está dada
por
23
x
4
, en este caso tenemos  
23 3
A 2
4 2
 
Al bosquejar los planos indicados vemos que
comparten los puntos B y F, y dado que la intersección
de dos planos diferentes es una única recta, la
intersección es la recta BF

En el prisma recto de la figura, las bases son triángulos equiláteros, con
perímetros de 30 cm.. Si la altura del prisma es 10 cm. ¿cuál es el área
total de la superficie del prisma?
A. 100 B.
250
3
C. 100 3 D. 300 E. 50 3 + 300
1.
A
B
C
D
E
F
G
La figura representa un cubo. La intersección del plano ABG y el
plano BCE es la recta
A. AG C. CE D. CFB. .BF E BE
   
3.
A C
B
A
B
C
D
E
F
G
A C
D
E
F G
Plano ABG Plano BCE
B
PROHIBIDA
LA
VENTA
GEOMETRÍA
45
El volumen de la parte sobrante es la diferencia entre el volumen del cubo y el volumen del cilindro, luego
2
3 3 45
V 5 5 125 125 35.34 89.67 90
2 4
  
          
 
5. La altura de un prisma rectangular es un tercio de su longitud y el ancho es la mitad de su longitud. Si
la diagonal del prisma mide 30 cm., su volumen es
A. 900 cm
3
B. 1688.25 cm
3
C. 2833.8 cm
3
D. 4583.5 cm
3
E. 9000 cm
3
Sean z la altura, y la longitud y x el ancho del prisma. Se tiene
y
z
3
 ,
y
x
2

La diagonal está dada por 2 2 2 2 2 2
d x y z 30 x y z 900        .
Al expresar en términos de “y” esta ecuación, se obtiene
2 2
2 2 2y y 1 1 49 180
y 900 1 y 900 y 900 y
4 9 4 9 36 7
 
           
 
.
El volumen está dado por V = x y z =
33
3y y y 1 180
y 2833.8 cm
2 3 6 6 7
 
     
 
6. Al introducir un trozo de metal en un tanque rectangular con agua, de dimensiones 50 cm. x 37 cm., el
nivel del agua subió 1 cm. ¿cuál es el volumen del trozo de metal?
A. 13 cm
3
B. 87 cm
3
C. 88 cm
3
D. 1850 cm
3
E. 9250 cm
3
El volumen del trozo de metal es equivalente al volumen que incrementó el tanque,
lo cual equivale al volumen de un paralelepípedo de dimensiones 1 x 50 x 37, es
decir 1850 cc
7. ¿Cuál es el número máximo de diagonales que pueden trazarse sobre las caras de un cubo de manera
que no hayan dos diagonales que tengan un punto en común?
A. 2 B. 3 C. 4 D. 5 E. 6
Trazamos inicialmente sobre una de las caras una diagonal, digamos AD, ninguna
otra puede involucrar estos puntos para satisfacer la condición. Con los puntos
restantes trazamos otra diagonal, digamos BE. Nos quedan cuatro vértices en este
caso los vértices C, F, G y H, con los cuales solo podemos trazar dos diagonales
más. En total cuatro diagonales. Cualquier otra variante conduce a la misma
cantidad.
De un cubo de 5” de arista se forma un cilindro circular recto de 3” de
diámetro, entonces el volumen de la parte sobrante del cubo, en pulgadas
cúbicas, es aproximadamente
A. 8 B. 10 C. 80 D. 90 E. 100
4.
z
x
y
50
37
A
B
C
D
E
F
G
H
PROHIBIDA
LA
VENTA
GEOMETRÍA
46
8. En la figura se muestra un paralelepípedo rectangular. Si a = 2b y b =
2
c
, ¿Cuál es el volumen en
términos de c?
A.
2
c2
B. 2c2 C. 3
c D.
2
c3
E.
4
c3
Como a = 2b y b =
2
c
, entonces a = c. Por ser un paralelepípedo rectangular,
V = a b c = c
2
c
 c =
3
c
2
9. El área de la base de una pirámide es 45 y el área de una sección transversal es 20. Si la altura de la
pirámide es 6 ¿a qué distancia de la sección transversal está el vértice?
A. 1.5 B. 2.25 C. 4 D. 4.75 E. 5
Sea A’ = 20, el área de la sección transversal y A = 45, el área de la base
de la pirámide.
Sea h la distancia desde la sección transversal al vértice. Se tiene
2
A' h 20 4 h 2
h 4
A 6 45 9 6 3
 
       
 
10. El área de la base de una pirámide es 45 y el área de una sección transversal es 20. Si la altura de la
pirámide es 6 ¿cuál es la razón entre los volúmenes de la pirámide mayor y la menor?
A. 3/2 B. 2 C. 9/4 D. 3 E. 27/8
Los datos forman parte del ejercicio anterior, de manera que ya sabemos que la distancia desde la sección
transversal al vértice es h = 4. Luego Si V es el volumen de la pirámide mayor y V’ el de la pirámide menor,
se tiene
3 3
V 6 3 27
V' 4 2 8
   
     
   
11. La base de una pirámide es un triángulo equilátero cuyo perímetro es 12. Si la altura es 10, el volumen
de la pirámide es
A. 40 B.
3
40
C.
3
340
D. 40 3 E. 120
Como la base de la pirámide es un triángulo equilátero cuyo perímetro es 12, su lado mide 4, luego
2
b
3
A 4 4 3
4
   y como h = 10, el volumen de la pirámide es b
1 1 40 3
V A h 4 3 10
3 3 3
     
a
b
c
6
h
PROHIBIDA
LA
VENTA
GEOMETRÍA
47
12. En un tronco de pirámide, la altura mide 10 m y las bases son cuadradas de 5 m y 9 m de lado
respectivamente. Hallar la diferencia (en m
3
) entre su volumen y la de un prisma recto de igual altura y de
base igual a la sección del tronco paralela a las bases y equidistante de ellas.
A. 4 B. 7 C. 40 D.
3
40
E. 70
Como cada lateral del tronco de pirámide, la sección que
equidista de las bases tiene arista igual a la base media del
trapecio, en este caso es
9 5
B 7
2

  .
El volumen del tronco de pirámide resulta
 B b B b
h 10 1510
V [A A A A ] 81 25 81 25
3 3 3
        
El volumen del prisma formado es V’ = 2
BA h 7 10 490   
La diferencia resulta V – V’ =
1510 40
490
3 3
 
13. En una pirámide cuadrada, en la que el lado de la base mide 8 cm y la altura mide 20 cm, se traza una
sección paralela a la base a 14 cm de ésta. Entonces el área de dicha sección es
A. 2.14 cm
2
B. 5.76 cm
2
C. 16.32 cm
2
D. 31.36 cm
2
E. 44.08 cm
2
Sea x la longitud de la arista de la sección, se tiene entonces
x 6 48
x 2.4
8 20 20
   
Como es un cuadrado, su área es  
22
A x 2.4 5.76  
14. Los diámetros de dos cilindros circulares rectos concéntricos son 12 y 6 pulgadas respectivamente y la
generatriz común es de 20 pulgadas, entonces el volumen del espacio que queda entre ambos cilindros es
A. 270 pulg
3
B. 270 pulg
3
C. 540 pulg
3
D. 540 pulg
3
E. 2160 pulg
3
El volumen buscado es la diferencia entre los volúmenes de los cilindros.
Como los diámetros son 12 y 6, los radios son 6 y 3, luego,
   2 2 2 2
V R r h 6 3 20 540         
10
5
5
5
9
7
7
10
20
6
14
8
PROHIBIDA
LA
VENTA
GEOMETRÍA
48
15. El volumen de una cisterna cilíndrica es 1200 m
3
y su altura es igual al diámetro, por lo tanto su área
total es
A. 190.98 m
2
B. 576.25 m
2
C. 600 m
2
D. 625.13 m
2
E. 712 m
2
Como el diámetro es igual a la altura se tiene
h
r
2
 .
Luego
2 3
2 3h h 4800
V r h h 1200 h 11.5176
2 4
  
           
 
El área total está dada por T B LA 2 A A  
El área de la base es
2 2
2
B
h h
A r
2 4
  
     
 
El area lateral es 2
L
h
A 2 rh 2 h h
2
      
Luego
2
2 2
T B L
3h
A 2 A A 2 h h
4 2

       
Al sustituir el valor de h obtenemos 2
T
3
A h 625.13
2

 
16. Un cono de revolución tiene 13 cm. de generatriz y el radio de la base es de 5 cm. Se corta por un plano
paralelo a la base que corta a la generatriz en un punto distante 5.2 cm. del vértice. Entonces el volumen del
tronco de cono formado es
A. 351.52 cm
3
B. 294.05 cm
3
C. 202.8 cm
3
D. 135.2 cm
3
E. 67.6 cm
3
Sea r el radio de la base menor del cono truncado. Sea y la altura del cono menor y H la altura del cono
truncado. Al considerar la altura del cono, los radios de las bases y la generatriz obtenemos los triángulos
rectángulos que se muestran en la figura.
La altura del cono original es 2 2 2 2
h g r 13 5 12    
Como los triángulos formados son semejantes se tiene
y r 5.2 24
y , r 2
12 5 13 5
    
Como H + h = 12, se tiene
24 36
H 12
5 5
  
El volumen buscado es    2 2 36
V H R r Rr 25 4 10 294.05
3 3 5
 
        
5.2
13 13
5.2
5
12
y
H
r
PROHIBIDA
LA
VENTA
GEOMETRÍA
49
17. Dado un cono circular recto con radio 3 m y generatriz 5 m, entonces su área lateral es
A. 2 B. 12 C. 15 D. 16 E. 30
El área lateral está dada por LA rg 3 5 15      
18. El área lateral de un tronco de cono que se forma cuando se corta un cono recto de 6 cm. de radio y 8
cm. de altura, por medio de un plano paralelo a la base del cono y que lo corta a una altura de 4.5 cm. es
A. 304.84 m
2
B. 216 m
2
C. 152.42 m
2
D. 84.82 m
2
E. 28.27 m
2
Se tiene  LA g R r   , donde g es la generatriz del tronco de
cono.
La generatriz del cono está dada por
2 2 2 2
g' h r 8 6 10    
Como los triángulos que se forman con la altura, la generatriz y
los radios son semejantes, se tiene
r 3.5
r 2.625
6 8
  
Por el teorema de Thales,
g 10
g 5.625
4.5 8
  
Luego    LA g R r 5.625 6 2.625 152.42       
19. Dos esferas de metal de radios 2a y 3a se funden juntos para hacer una esfera mayor. El radio de la
nueva esfera es
A. 2.5a B. 5a C. 6.5a D. 3
35 a E.

a5
El volumen de la nueva esfera es la suma de los volúmenes de las esferas dadas.
La esfera de radio 2 a tiene un volumen  
33 3
1
4 4 32
V r 2a a
3 3 3
     
La esfera de radio 3 a tiene un volumen  
33 3
2
4 4 108
V r 3a a
3 3 3
     
El volumen de la nueva esfera es
33 3 3 3
1 2
32 108 140 4
V V V a a a r r a 35
3 3 3 3
           
r
6
4.5
10
x
6
8
3.5
g
r
PROHIBIDA
LA
VENTA
GEOMETRÍA
50
20. Un cono tiene una altura igual al doble de su radio. Una esfera tiene un radio igual al radio de la base
del cono. La razón entre el volumen del cono y el volumen de la esfera es
A. 1/2 B. 1 C. 3/2 D. 2 E. 4
Volumen del cono 2 2 3
C
1 1 2
V r h r 2r r
3 3 3
      
Volumen de la esfera 3
E
4
V r
3
 
Luego
3
C
3E
2
rV 13
4V 2
r
3

 

21. Un cono tiene una altura igual al triple de su radio. Una esfera tiene un radio igual al radio de la base del
cono. La razón entre el volumen del cono y el volumen de la esfera es
A. 1/2 B. 1 C. 3/2 D. 3/4 E. 3
Volumen del cono 2 2 3
C
1 1
V r h r 3r r
3 3
      
Volumen de la esfera 3
E
4
V r
3
 
Luego
3
C
3E
V r 3
4V 4
r
3

 

22. La altura de un cono es 5 cm. Un plano a 2 cm. del vértice es paralelo a la base del cono. Si el volumen
del cono más pequeño es 24 cm
3
, el volumen del cono más grande es
A. 750 cm
3
B. 375 cm
3
C. 240 cm
3
D. 120 cm
3
E. 48 cm
3
Se tiene que los volúmenes de cono semejantes son proporcionales al cubo
de su razón de semejanza
3
1
2
V h
V H
 
  
 
Luego el volumen del cono más grande es
3
5
V 24 375
2
 
   
 
2 r
r
r
3 r
r
r
2
5
PROHIBIDA
LA
VENTA
GEOMETRÍA
51
23. Un cubo está inscrito en una esfera. Si el área de la superficie total del cubo es 2
m

40
, entonces el
área de la superficie de la esfera es
A. 10 m
2
B. 15 m
2
C. 20 m
2
D. 30 m
2
E. 40 m
2
Tenemos que la diagonal del cubo es el diámetro de la esfera.
Si x es la longitud de la arista del cubo, su área total es 2 240 20
6x x
3
  
 
En un cubo la diagonal es d 3 x , luego el radio de la esfera es
3
r x
2

El área de la superficie de la esfera es 2
S 4 r  , al sustituir los valores encontrados
resulta 2 23 20
S 4 r 4 x 3 20
4 3
      

24. La base de una pirámide hexagonal tiene un área de 26 m
2
. Si el volumen de dicha pirámide es 78
m
3
, entonces su altura mide
A. 3 m B. 4 m C. 6 m D. 9 m E. 12 m
Tenemos que el volumen de una pirámide esta dado por
B
B
3V1
V A h h
3 A
    
Al sustituir los datos se obtiene
B
3V 3 78
h 9
A 26

  
Al considerar el triángulo formado por el diámetro AB y el vértice, tenemos que mCAB = mCBA = 30
Luego la altura del cono es
r
h
3
 . El volumen es
3
2 2 r1 1 r
V r h r
3 3 3 3 3

     
Se tiene entonces
3
3r 1000 3
r 1000 r 10
93 3
 
     y por tanto el diámetro es d = 20.
A
C
B
25.
h
r
Si el cono de la figura tiene un volumen de 31000 3
cm
9

,
C es el vértice, AB un diámetro y mACB = 120°,
entonces el diámetro de la base, en centímetros, es
A. 5 B. 10 C. 15 D. 20 E. 30
PROHIBIDA
LA
VENTA
GEOMETRÍA
52
26. El área de la superficie total de un cubo es 12 m
2
. Entonces la longitud de su diagonal es
A. 2 B. 3 C. 2 D. 5 E. 6
El área de la superficie total de un cubo de lado x, está dada por 2
TA 6x y su diagonal por d 3 x
Luego 2
6x 12 x 2   y d 3 x 3 2 6   
27. Si la generatriz de un cono mide 25 m y el diámetro de su base es 8 m, su volumen mide
A. 200 m
3
B. 400 m
3
C. 413.48 m
3
D. 418.88 m
3
E. 1587.4 m
3
El volumen está dado por 21
V r h
3
  y la altura es 2 2
h g r 
Tenemos que g = 25 y d = 8 o sea r = 4, luego 2 2
h 25 4 609   , por tanto
21
V 4 609 413.48
3
  
28. En una esfera de radio 2, se tiene inscrito un cilindro de manera que el diámetro del cilindro es igual
al radio de la esfera. Entonces el área lateral del cilindro es
A. 4 B. 8 C. 2 3 D. 4 3  E. 8 3
Sean A, B y C los puntos marcados en la figura. Tenemos que AB es diámetro
del cilindro y BC diámetro de la esfera, luego AB = 2 y BC = 4.
La altura del cilindro es AC, el cual al aplicar el Teorema de Pitágoras resulta
2 2 2 2
AC BC AB 4 2 12 2 3     
El área lateral de un cilindro está dada por LA 2 rh  . Como el diámetro del
cilindro mide 2, su radio mide 1.
Luego LA 2 1 2 3 4 3    
29. En una esfera de radio 2, se tiene inscrito un cilindro de manera que el diámetro del cilindro es igual
al radio de la esfera. Entonces el volumen del cilindro es
A. 4 B. 8 C. 2 3  D. 4 3 E. 8 3
Este ejercicio contiene los mismos datos del anterior, luego el cilindro tiene r = 1 y h = 2 3
Luego su volumen 2 2
V r h 1 2 3 2 3      
A B
C
PROHIBIDA
LA
VENTA
GEOMETRÍA
53
30. Si un cilindro circular tiene la altura igual que su diámetro y su área total mide 150 m
2
, entonces su
volumen, en m
3
, es
A. 25 B. 250 C. 250 D. 25 E. 150
Tenemos que 2 2 2
TA 2 r 2 rh 2 r 2 r 2r 6 r 150 r 5               y h = 10.
Luego 2 2
V r h 5 10 250      
31. Se desea construir un tubo cilíndrico de 50 cm. de largo, un diámetro externo de 12 cm. y un
diámetro interno de 10 cm. ¿Qué cantidad de material se necesita?
A. 550 cm
3
B. 500 cc
3
C. 375 cm
3
D. 750 cm
3
E. 1275 cm
3
.
La cantidad de material es el volumen del tubo y este es la diferencia del
volumen del cilindro exterior menos el volumen del cilindro interior,
que representa el agujero.
Tenemos: 1 2h 50, r 6, r 5  
Luego V = (6)
2
(50) – (5)
2
(50) = 550  cm
3
32. Se desea construir un cono circular recto de 4’’ de altura y radio en la base de 3’’. Para formar la
superficie lateral se corta un sector circular como el que se muestra en la figura. Determine el radio R y la
medida del ángulo central  del sector circular necesario.
Se tiene que la generatriz del cono mide 2 2 2 2
g h r 4 3 5     y que ésta es el radio del sector
circular con la que se construye el cono, es decir R = 5.
El área lateral del cono es el área del sector circular necesario para su construcción.
Tenemos LA rg 3 5 15       . Como el área de un sector circular está dada por
2
R
360
 

Se tiene
2 2
R 5 1080
15 216
360 360 5
    
       
 

R A. R = 7’’;  = 200º B. R = 6’’;  = 200º
C. R = 6’’;  = 215º D. R = 5’’;  = 216º
E. R = 5’’;  = 240º
4 g
3
r
2r
PROHIBIDA
LA
VENTA
GEOMETRÍA
54
33. La figura muestra dos esferas tangentes que descansan sobre una mesa plana. Si los radios de las
esferas son 8 cm. y 16 cm. respectivamente, entonces la distancia en cm. entre los puntos de contacto de
las esferas con la mesa es:
Al considerar los centros de las esferas y los puntos de tangencia con la mesa, se forma un trapecio, como
el que se muestra en la figura, en el cual OO’ = 24 ya que las esferas son tangentes. OA = 16 y O’B = 8.
Si trazamos una paralela a la mes desde O’, sea D el punto donde corta al
radio OA. Tenemos que AB = PO’, PA = O’B = 8, luego OP = OA – PA = 8
Como OA  AB, también se tiene OP  PO’. Luego por el teorema de
Pitágoras se tiene 2 2
PO' AB 24 8 16 2   
Tenemos una pirámide de base triangular. La base es un triángulo rectángulo (
__ __
BD DC ) de catetos BD = 12
y CD = 16, luego su área es b
1
A 12 16 96
2
   
Como
__ __
AB BC y
__ __
AB BD , se deduce que
__
AB es perpendicular al plano de la base y por tanto es altura
de la pirámide, luego b
1 1
V A h 96 9 288
3 3
      
x A. 8 2 B. 12 C. 16 2 D. 24 E. 24 2
A
B
C
D
34. En la figura que se muestra,
____
BCAB  ,
____
BDAB  y
____
DCBD  . Si AB = 9, BD = 12 y CD = 16, entonces el
volumen del sólido formado es:
A. 144 B. 288 C. 432 D. 576 E. 864
O
O’P
A B
PROHIBIDA
LA
VENTA
GEOMETRÍA
55
35. Un paralelepípedo rectangular se recorta como se muestra en
la figura. El volumen del sólido obtenido es:
A. 72 B. 48 C. 24 D. 16 E. 12
Basta observar que la parte recortada (la parte superior punteada) es idéntica a la parte que queda del
sólido, luego el volumen buscado es la mitad del volumen del paralelepípedo original, es decir
1
V 6 4 4 48
2
    
1 E 8 D 15 D 22 B 29 C
2 B 9 C 16 B 23 C 30 B
3 B 10 E 17 C 24 D 31 A
4 D 11 C 18 C 25 D 32 D
5 C 12 D 19 D 26 E 33 C
6 D 13 B 20 A 27 C 34 B
7 C 14 D 21 D 28 D 35 B
4
2
4
2
6
4
PROHIBIDA
LA
VENTA

Más contenido relacionado

La actualidad más candente

3.vectores en el plano
3.vectores en el plano3.vectores en el plano
3.vectores en el planosemoroca
 
áNgulos cortados por dos paralelas y una secante
áNgulos cortados por dos paralelas y una secanteáNgulos cortados por dos paralelas y una secante
áNgulos cortados por dos paralelas y una secanteJuan Jose Tello
 
Teoría y problemas de Geometría ADUNI ccesa007
Teoría y problemas de Geometría ADUNI  ccesa007Teoría y problemas de Geometría ADUNI  ccesa007
Teoría y problemas de Geometría ADUNI ccesa007Demetrio Ccesa Rayme
 
Teorema del seno y el coseno
Teorema  del seno y el cosenoTeorema  del seno y el coseno
Teorema del seno y el cosenofrinconr
 
Areas sombreadas
Areas sombreadasAreas sombreadas
Areas sombreadasasteteli
 
Problemas resueltos
Problemas resueltosProblemas resueltos
Problemas resueltosdoriscelia
 
Problemas Leyes de Newton Nivel 0B
Problemas Leyes de Newton Nivel 0BProblemas Leyes de Newton Nivel 0B
Problemas Leyes de Newton Nivel 0BESPOL
 
Problemas resueltos de geometria analitica plana
Problemas resueltos de geometria analitica planaProblemas resueltos de geometria analitica plana
Problemas resueltos de geometria analitica planaCarlos Chaparro
 
Trigonometria
TrigonometriaTrigonometria
Trigonometriasitayanis
 
Libro de geometria de preparatoria preuniversitaria
Libro de geometria de preparatoria preuniversitariaLibro de geometria de preparatoria preuniversitaria
Libro de geometria de preparatoria preuniversitariaRuben Espiritu Gonzales
 
Rectas paralelas-cortadas-por-una-secante ejercicios
Rectas paralelas-cortadas-por-una-secante ejerciciosRectas paralelas-cortadas-por-una-secante ejercicios
Rectas paralelas-cortadas-por-una-secante ejerciciospiros200320
 
Ejercicios Cinemática
Ejercicios CinemáticaEjercicios Cinemática
Ejercicios CinemáticaKike Prieto
 
Aplicaciones de la congruencia de triángulos
Aplicaciones  de la congruencia de triángulosAplicaciones  de la congruencia de triángulos
Aplicaciones de la congruencia de triángulosMarlube3
 
ANALISIS DE FUERZAS EN 2D Y 3D
ANALISIS DE FUERZAS EN 2D Y 3DANALISIS DE FUERZAS EN 2D Y 3D
ANALISIS DE FUERZAS EN 2D Y 3DIrlanda Gt
 
09 ejercicios razones y proporciones
09 ejercicios razones y proporciones09 ejercicios razones y proporciones
09 ejercicios razones y proporcionesMarcelo Calderón
 
Solucionario guía 1 unidad i algebra lineal
Solucionario guía 1 unidad  i  algebra linealSolucionario guía 1 unidad  i  algebra lineal
Solucionario guía 1 unidad i algebra linealRafael Beas Rivera
 

La actualidad más candente (20)

3.vectores en el plano
3.vectores en el plano3.vectores en el plano
3.vectores en el plano
 
áNgulos cortados por dos paralelas y una secante
áNgulos cortados por dos paralelas y una secanteáNgulos cortados por dos paralelas y una secante
áNgulos cortados por dos paralelas y una secante
 
Teoría y problemas de Geometría ADUNI ccesa007
Teoría y problemas de Geometría ADUNI  ccesa007Teoría y problemas de Geometría ADUNI  ccesa007
Teoría y problemas de Geometría ADUNI ccesa007
 
Teorema del seno y el coseno
Teorema  del seno y el cosenoTeorema  del seno y el coseno
Teorema del seno y el coseno
 
Areas sombreadas
Areas sombreadasAreas sombreadas
Areas sombreadas
 
Problemas resueltos
Problemas resueltosProblemas resueltos
Problemas resueltos
 
Geometria 5°
Geometria 5°   Geometria 5°
Geometria 5°
 
Problemas Leyes de Newton Nivel 0B
Problemas Leyes de Newton Nivel 0BProblemas Leyes de Newton Nivel 0B
Problemas Leyes de Newton Nivel 0B
 
Geometria
GeometriaGeometria
Geometria
 
Problemas resueltos de geometria analitica plana
Problemas resueltos de geometria analitica planaProblemas resueltos de geometria analitica plana
Problemas resueltos de geometria analitica plana
 
Cuadrilatero
CuadrilateroCuadrilatero
Cuadrilatero
 
Trigonometria
TrigonometriaTrigonometria
Trigonometria
 
Libro de geometria de preparatoria preuniversitaria
Libro de geometria de preparatoria preuniversitariaLibro de geometria de preparatoria preuniversitaria
Libro de geometria de preparatoria preuniversitaria
 
Optimizacion
OptimizacionOptimizacion
Optimizacion
 
Rectas paralelas-cortadas-por-una-secante ejercicios
Rectas paralelas-cortadas-por-una-secante ejerciciosRectas paralelas-cortadas-por-una-secante ejercicios
Rectas paralelas-cortadas-por-una-secante ejercicios
 
Ejercicios Cinemática
Ejercicios CinemáticaEjercicios Cinemática
Ejercicios Cinemática
 
Aplicaciones de la congruencia de triángulos
Aplicaciones  de la congruencia de triángulosAplicaciones  de la congruencia de triángulos
Aplicaciones de la congruencia de triángulos
 
ANALISIS DE FUERZAS EN 2D Y 3D
ANALISIS DE FUERZAS EN 2D Y 3DANALISIS DE FUERZAS EN 2D Y 3D
ANALISIS DE FUERZAS EN 2D Y 3D
 
09 ejercicios razones y proporciones
09 ejercicios razones y proporciones09 ejercicios razones y proporciones
09 ejercicios razones y proporciones
 
Solucionario guía 1 unidad i algebra lineal
Solucionario guía 1 unidad  i  algebra linealSolucionario guía 1 unidad  i  algebra lineal
Solucionario guía 1 unidad i algebra lineal
 

Similar a Ejercicios sobre conceptos_basicos_geometria euclidiana

Similar a Ejercicios sobre conceptos_basicos_geometria euclidiana (20)

Geometria%20(bolet%c3%a dn%20 n%c2%ba%2001%20-%20ab2%20sm%202015)
Geometria%20(bolet%c3%a dn%20 n%c2%ba%2001%20-%20ab2%20sm%202015)Geometria%20(bolet%c3%a dn%20 n%c2%ba%2001%20-%20ab2%20sm%202015)
Geometria%20(bolet%c3%a dn%20 n%c2%ba%2001%20-%20ab2%20sm%202015)
 
Sb1 2016 GEOMETRIA_01
Sb1 2016 GEOMETRIA_01Sb1 2016 GEOMETRIA_01
Sb1 2016 GEOMETRIA_01
 
Segmentos y angulos
Segmentos y angulosSegmentos y angulos
Segmentos y angulos
 
Segmentos y angulos
Segmentos y angulosSegmentos y angulos
Segmentos y angulos
 
Uni 2005-c
Uni 2005-cUni 2005-c
Uni 2005-c
 
Geom1 2014 g_01
Geom1 2014 g_01Geom1 2014 g_01
Geom1 2014 g_01
 
Balotario de geometria junio 2013 seleccion
Balotario de geometria junio  2013 seleccionBalotario de geometria junio  2013 seleccion
Balotario de geometria junio 2013 seleccion
 
394261866-Practica-01-Geometria-1.pdf
394261866-Practica-01-Geometria-1.pdf394261866-Practica-01-Geometria-1.pdf
394261866-Practica-01-Geometria-1.pdf
 
Ma11 c3a1ngulos-y-tric3a1ngulos
Ma11 c3a1ngulos-y-tric3a1ngulosMa11 c3a1ngulos-y-tric3a1ngulos
Ma11 c3a1ngulos-y-tric3a1ngulos
 
7° elementos secundarios de triangulo
7° elementos secundarios de triangulo7° elementos secundarios de triangulo
7° elementos secundarios de triangulo
 
Geometria 5° 2 b
Geometria 5° 2 bGeometria 5° 2 b
Geometria 5° 2 b
 
Líneas notables del triángulo 4º
Líneas notables del triángulo   4ºLíneas notables del triángulo   4º
Líneas notables del triángulo 4º
 
Balotario de geometria final 2013 ok
Balotario de geometria final 2013 okBalotario de geometria final 2013 ok
Balotario de geometria final 2013 ok
 
César vallejo 2015
César  vallejo   2015César  vallejo   2015
César vallejo 2015
 
Geometria 4° 2 b
Geometria 4° 2 bGeometria 4° 2 b
Geometria 4° 2 b
 
23 ángulos y triangulos
23 ángulos y triangulos23 ángulos y triangulos
23 ángulos y triangulos
 
24 ejercitación ángulos y triangulos
24 ejercitación ángulos y triangulos24 ejercitación ángulos y triangulos
24 ejercitación ángulos y triangulos
 
Solguia1
Solguia1Solguia1
Solguia1
 
2º semana cs
2º semana cs2º semana cs
2º semana cs
 
1 tema de trigonometria 5 to
1 tema de trigonometria 5 to1 tema de trigonometria 5 to
1 tema de trigonometria 5 to
 

Último

PRIMER SEMESTRE 2024 ASAMBLEA DEPARTAMENTAL.pptx
PRIMER SEMESTRE 2024 ASAMBLEA DEPARTAMENTAL.pptxPRIMER SEMESTRE 2024 ASAMBLEA DEPARTAMENTAL.pptx
PRIMER SEMESTRE 2024 ASAMBLEA DEPARTAMENTAL.pptxinformacionasapespu
 
Informatica Generalidades - Conceptos Básicos
Informatica Generalidades - Conceptos BásicosInformatica Generalidades - Conceptos Básicos
Informatica Generalidades - Conceptos BásicosCesarFernandez937857
 
Planificacion Anual 2do Grado Educacion Primaria 2024 Ccesa007.pdf
Planificacion Anual 2do Grado Educacion Primaria   2024   Ccesa007.pdfPlanificacion Anual 2do Grado Educacion Primaria   2024   Ccesa007.pdf
Planificacion Anual 2do Grado Educacion Primaria 2024 Ccesa007.pdfDemetrio Ccesa Rayme
 
el CTE 6 DOCENTES 2 2023-2024abcdefghijoklmnñopqrstuvwxyz
el CTE 6 DOCENTES 2 2023-2024abcdefghijoklmnñopqrstuvwxyzel CTE 6 DOCENTES 2 2023-2024abcdefghijoklmnñopqrstuvwxyz
el CTE 6 DOCENTES 2 2023-2024abcdefghijoklmnñopqrstuvwxyzprofefilete
 
Manual - ABAS II completo 263 hojas .pdf
Manual - ABAS II completo 263 hojas .pdfManual - ABAS II completo 263 hojas .pdf
Manual - ABAS II completo 263 hojas .pdfMaryRotonda1
 
Clasificaciones, modalidades y tendencias de investigación educativa.
Clasificaciones, modalidades y tendencias de investigación educativa.Clasificaciones, modalidades y tendencias de investigación educativa.
Clasificaciones, modalidades y tendencias de investigación educativa.José Luis Palma
 
NARRACIONES SOBRE LA VIDA DEL GENERAL ELOY ALFARO
NARRACIONES SOBRE LA VIDA DEL GENERAL ELOY ALFARONARRACIONES SOBRE LA VIDA DEL GENERAL ELOY ALFARO
NARRACIONES SOBRE LA VIDA DEL GENERAL ELOY ALFAROJosé Luis Palma
 
DE LAS OLIMPIADAS GRIEGAS A LAS DEL MUNDO MODERNO.ppt
DE LAS OLIMPIADAS GRIEGAS A LAS DEL MUNDO MODERNO.pptDE LAS OLIMPIADAS GRIEGAS A LAS DEL MUNDO MODERNO.ppt
DE LAS OLIMPIADAS GRIEGAS A LAS DEL MUNDO MODERNO.pptELENA GALLARDO PAÚLS
 
La triple Naturaleza del Hombre estudio.
La triple Naturaleza del Hombre estudio.La triple Naturaleza del Hombre estudio.
La triple Naturaleza del Hombre estudio.amayarogel
 
programa dia de las madres 10 de mayo para evento
programa dia de las madres 10 de mayo  para eventoprograma dia de las madres 10 de mayo  para evento
programa dia de las madres 10 de mayo para eventoDiegoMtsS
 
6° SEM30 WORD PLANEACIÓN PROYECTOS DARUKEL 23-24.docx
6° SEM30 WORD PLANEACIÓN PROYECTOS DARUKEL 23-24.docx6° SEM30 WORD PLANEACIÓN PROYECTOS DARUKEL 23-24.docx
6° SEM30 WORD PLANEACIÓN PROYECTOS DARUKEL 23-24.docxCeciliaGuerreroGonza1
 
TEMA 13 ESPAÑA EN DEMOCRACIA:DISTINTOS GOBIERNOS
TEMA 13 ESPAÑA EN DEMOCRACIA:DISTINTOS GOBIERNOSTEMA 13 ESPAÑA EN DEMOCRACIA:DISTINTOS GOBIERNOS
TEMA 13 ESPAÑA EN DEMOCRACIA:DISTINTOS GOBIERNOSjlorentemartos
 
Historia y técnica del collage en el arte
Historia y técnica del collage en el arteHistoria y técnica del collage en el arte
Historia y técnica del collage en el arteRaquel Martín Contreras
 
LINEAMIENTOS INICIO DEL AÑO LECTIVO 2024-2025.pptx
LINEAMIENTOS INICIO DEL AÑO LECTIVO 2024-2025.pptxLINEAMIENTOS INICIO DEL AÑO LECTIVO 2024-2025.pptx
LINEAMIENTOS INICIO DEL AÑO LECTIVO 2024-2025.pptxdanalikcruz2000
 
texto argumentativo, ejemplos y ejercicios prácticos
texto argumentativo, ejemplos y ejercicios prácticostexto argumentativo, ejemplos y ejercicios prácticos
texto argumentativo, ejemplos y ejercicios prácticosisabeltrejoros
 
RETO MES DE ABRIL .............................docx
RETO MES DE ABRIL .............................docxRETO MES DE ABRIL .............................docx
RETO MES DE ABRIL .............................docxAna Fernandez
 
Heinsohn Privacidad y Ciberseguridad para el sector educativo
Heinsohn Privacidad y Ciberseguridad para el sector educativoHeinsohn Privacidad y Ciberseguridad para el sector educativo
Heinsohn Privacidad y Ciberseguridad para el sector educativoFundación YOD YOD
 
cortes de luz abril 2024 en la provincia de tungurahua
cortes de luz abril 2024 en la provincia de tungurahuacortes de luz abril 2024 en la provincia de tungurahua
cortes de luz abril 2024 en la provincia de tungurahuaDANNYISAACCARVAJALGA
 
codigos HTML para blogs y paginas web Karina
codigos HTML para blogs y paginas web Karinacodigos HTML para blogs y paginas web Karina
codigos HTML para blogs y paginas web Karinavergarakarina022
 

Último (20)

Defendamos la verdad. La defensa es importante.
Defendamos la verdad. La defensa es importante.Defendamos la verdad. La defensa es importante.
Defendamos la verdad. La defensa es importante.
 
PRIMER SEMESTRE 2024 ASAMBLEA DEPARTAMENTAL.pptx
PRIMER SEMESTRE 2024 ASAMBLEA DEPARTAMENTAL.pptxPRIMER SEMESTRE 2024 ASAMBLEA DEPARTAMENTAL.pptx
PRIMER SEMESTRE 2024 ASAMBLEA DEPARTAMENTAL.pptx
 
Informatica Generalidades - Conceptos Básicos
Informatica Generalidades - Conceptos BásicosInformatica Generalidades - Conceptos Básicos
Informatica Generalidades - Conceptos Básicos
 
Planificacion Anual 2do Grado Educacion Primaria 2024 Ccesa007.pdf
Planificacion Anual 2do Grado Educacion Primaria   2024   Ccesa007.pdfPlanificacion Anual 2do Grado Educacion Primaria   2024   Ccesa007.pdf
Planificacion Anual 2do Grado Educacion Primaria 2024 Ccesa007.pdf
 
el CTE 6 DOCENTES 2 2023-2024abcdefghijoklmnñopqrstuvwxyz
el CTE 6 DOCENTES 2 2023-2024abcdefghijoklmnñopqrstuvwxyzel CTE 6 DOCENTES 2 2023-2024abcdefghijoklmnñopqrstuvwxyz
el CTE 6 DOCENTES 2 2023-2024abcdefghijoklmnñopqrstuvwxyz
 
Manual - ABAS II completo 263 hojas .pdf
Manual - ABAS II completo 263 hojas .pdfManual - ABAS II completo 263 hojas .pdf
Manual - ABAS II completo 263 hojas .pdf
 
Clasificaciones, modalidades y tendencias de investigación educativa.
Clasificaciones, modalidades y tendencias de investigación educativa.Clasificaciones, modalidades y tendencias de investigación educativa.
Clasificaciones, modalidades y tendencias de investigación educativa.
 
NARRACIONES SOBRE LA VIDA DEL GENERAL ELOY ALFARO
NARRACIONES SOBRE LA VIDA DEL GENERAL ELOY ALFARONARRACIONES SOBRE LA VIDA DEL GENERAL ELOY ALFARO
NARRACIONES SOBRE LA VIDA DEL GENERAL ELOY ALFARO
 
DE LAS OLIMPIADAS GRIEGAS A LAS DEL MUNDO MODERNO.ppt
DE LAS OLIMPIADAS GRIEGAS A LAS DEL MUNDO MODERNO.pptDE LAS OLIMPIADAS GRIEGAS A LAS DEL MUNDO MODERNO.ppt
DE LAS OLIMPIADAS GRIEGAS A LAS DEL MUNDO MODERNO.ppt
 
La triple Naturaleza del Hombre estudio.
La triple Naturaleza del Hombre estudio.La triple Naturaleza del Hombre estudio.
La triple Naturaleza del Hombre estudio.
 
programa dia de las madres 10 de mayo para evento
programa dia de las madres 10 de mayo  para eventoprograma dia de las madres 10 de mayo  para evento
programa dia de las madres 10 de mayo para evento
 
6° SEM30 WORD PLANEACIÓN PROYECTOS DARUKEL 23-24.docx
6° SEM30 WORD PLANEACIÓN PROYECTOS DARUKEL 23-24.docx6° SEM30 WORD PLANEACIÓN PROYECTOS DARUKEL 23-24.docx
6° SEM30 WORD PLANEACIÓN PROYECTOS DARUKEL 23-24.docx
 
TEMA 13 ESPAÑA EN DEMOCRACIA:DISTINTOS GOBIERNOS
TEMA 13 ESPAÑA EN DEMOCRACIA:DISTINTOS GOBIERNOSTEMA 13 ESPAÑA EN DEMOCRACIA:DISTINTOS GOBIERNOS
TEMA 13 ESPAÑA EN DEMOCRACIA:DISTINTOS GOBIERNOS
 
Historia y técnica del collage en el arte
Historia y técnica del collage en el arteHistoria y técnica del collage en el arte
Historia y técnica del collage en el arte
 
LINEAMIENTOS INICIO DEL AÑO LECTIVO 2024-2025.pptx
LINEAMIENTOS INICIO DEL AÑO LECTIVO 2024-2025.pptxLINEAMIENTOS INICIO DEL AÑO LECTIVO 2024-2025.pptx
LINEAMIENTOS INICIO DEL AÑO LECTIVO 2024-2025.pptx
 
texto argumentativo, ejemplos y ejercicios prácticos
texto argumentativo, ejemplos y ejercicios prácticostexto argumentativo, ejemplos y ejercicios prácticos
texto argumentativo, ejemplos y ejercicios prácticos
 
RETO MES DE ABRIL .............................docx
RETO MES DE ABRIL .............................docxRETO MES DE ABRIL .............................docx
RETO MES DE ABRIL .............................docx
 
Heinsohn Privacidad y Ciberseguridad para el sector educativo
Heinsohn Privacidad y Ciberseguridad para el sector educativoHeinsohn Privacidad y Ciberseguridad para el sector educativo
Heinsohn Privacidad y Ciberseguridad para el sector educativo
 
cortes de luz abril 2024 en la provincia de tungurahua
cortes de luz abril 2024 en la provincia de tungurahuacortes de luz abril 2024 en la provincia de tungurahua
cortes de luz abril 2024 en la provincia de tungurahua
 
codigos HTML para blogs y paginas web Karina
codigos HTML para blogs y paginas web Karinacodigos HTML para blogs y paginas web Karina
codigos HTML para blogs y paginas web Karina
 

Ejercicios sobre conceptos_basicos_geometria euclidiana

  • 1. 1 1. EJERCICIOS SOBRE CONCEPTOS BÁSICOS 1. En la figura, el ángulo COB mide 120º y el ángulo COD mide la mitad del ángulo BOA. Entonces, la medida del  BOA es: Sea mBOA = x, luego mCOD = x/2. Se tiene x 3x 120 x 180 60 x 40 2 2            2. Si dos planos diferentes se intersecan, su intersección es A. Un punto. B. Dos puntos C. Una única recta D. Dos rectas diferentes E. Falta información Esto es uno de los axiomas de la Geometría Euclidiana. 3. En la figura, 3241 mmmm   , ¿cuál de las siguientes expresiones es siempre verdadera? Con la información dada las parejas de rectas perpendiculares están “libres”, luego pueden ser giradas y seguirían satisfaciendo los datos dados. Por tanto no puede afirmarse ni A, ni B, ni C, ni D. 4. R, S y T son tres puntos colineales como se muestran en la figura. Si ST = 4x + 4 y RS es la mitad de ST, entonces la longitud de RT es A. 3x – 4 B. 3x – 6 C. 3x + 2 D. 6x – 12 E. 6x + 6 Dado que los puntos son colineales, se tiene RT = RS + ST = 1 3 3 ST ST ST 2 2 2    (4x +4) = 6x + 6 C B D O A A. 20º B. 30º C. 40º D. 60º E. 80º 120 x/2 x 1m  2m  3m  4m  R S T    A. 21 mm  || B. 31 mm   C. 43 mm  || D. 42 mm   E. NDLA 1m  2m  3m  4m  PROHIBIDA LA VENTA
  • 2. GEOMETRÍA 2 Sean A, B y C los puntos indicados en la figura, y sean mBAC = , mACB = . Se tiene  = 50, por ser opuesto por el vértice con el ángulo que mide 50 y  = 180 – 130 = 50. El ángulo que mide y es un ángulo exterior con respecto al ABC, luego su medida equivale a la suma de los ángulos internos no adyacentes, es decir y =  +  = 50 + 50 = 100 Dado que las rectas son paralelas, x = mFCE, por ser ángulos correspondientes. A su vez este ángulo por ser externo al ECD, es la suma de las medidas de los ángulos CED y EDC. Se tiene mCED = 90 y mEDC = 180 – 140 = 40, luego x = 90 + 40 = 130 Las marcas en el ángulo A, indican que AD es bisectriz de dicho ángulo, luego x = 40 y mA = 80. Al considerar el ABC, se tiene z = 180 – 80 – 70 = 30 Dado que __ __ EB ||DC , se tiene x = 180 – 130 = 50 por ser ángulos internos al mismo lado, entre paralelas y como __ __ AD AC, el ADC es triangulo rectángulo y por tanto “y” es el complemento de “x”, luego y = 90 – 50 = 40 x y 130º A B C D E En la figura, ________ DC||EB,ACAD , entonces el valor de y es: A. 30º B. 40º C. 45º D. 50º E. 60º 8. 50º 120º 130º xº yº A partir de la información indicada en la figura, el valor de y es: A. 170º B. 130º C. 120º D. 100º E. 50º 5. A B C   90º 140º xº En la figura, si ____ CD||AB , el valor de x es: A. 50º 70º C. 130º D. 140º E. 230º A B F C D 6. E xº A partir de la información brindada en la figura, el valor de z resulta: A. 30º B. 40º C. 70º D. 80º E. 110º x z 40 70º 7. A B D C PROHIBIDA LA VENTA
  • 3. GEOMETRÍA 3 Se tiene mABC = 180 – 140 = 40, x = 115 – 40 = 75 ya que el ACD es externo al ABC Se tiene mEDB = 180 – 150 = 30, ABC  DBE por ser opuestos por el vértice y por el teorema de semejanza AA, ABC  DBE, luego mBAC = x = mEDB = 30. 11. A – B – C – D; E y F son puntos medios de AB y CD respectivamente; Si AC = 10 y BD = 12, entonces EF = ? A. 5 B. 6 C. 9 D. 11 E. 22 Sean AE = EB = x, CF = FD = y (E y F son puntos medios de AB y CD respectivamente) Se tiene BC = AC – AB = 10 – 2x (1) y también BC = BD – CD = 12 – 2y (2) Igualando (1) y (2): 10 – 2x = 12 – 2y Al simplificar se obtiene y – x = 1 (3). Por otro lado se tiene EF = EB + BC + CF = x + (10 – 2x) + y = 10 – x + y = 10 + (y – x) Al introducir (3) resulta EF + 10 + 1 = 11 x 140º 115º 9. En la figura, el valor de x es A. 25º B. 40º C. 45º D. 65 E. 75º A B C D xº 150º 10. En la figura, el valor de x es A. 30º B. 40º C. 45º D. 50º E. 60º A B C D E A E B C F D x x yy 10 12 PROHIBIDA LA VENTA
  • 4. GEOMETRÍA 4 En el ABC, tenemos que mA = 180 – mABC – mACB = 180 – (mABC – mACB) (1) Se tiene que mABC = 180 –  y mACB = 180 – , luego mABC + mACB = 360 – ( + ) Como  +  = 255, resulta mABC + mACB = 360 – 255 = 105 Sustituyendo en (1) obtenemos mA = 180 – 105 = 75 13. ¿Para qué valor de x, los segmentos AB y CD son paralelos? Como el ángulo a la izquierda de C es congruente con el ángulo a la derecha, también mide 25. Luego mACD = 180 – 2  25 = 130. Como el APC, es recto en P, mPAC = 90 – 25 = 65 Para que AB y CD sean paralelos, el ángulo CAB debe ser el suplemento del ángulo ACD ya que serían ángulos internos al mismo lado entre paralelas o sea mCAB = 180 – 130 = 50. Se tiene entonces x + 50 + 65 = 180  x = 180 – 50 – 65 = 65 14. Si ____ CD||AB , ¿Cuál es el valor de x? A. 170º B. 150º C. 120º D. 100º E. 80º Trazamos EF, paralela a las rectas AB y CD, luego mAEF = 180º – 120º = 60º y mFEC = 180º – xº . Además se tiene mAEF + mFEC = mAEC = 90 , luego 60 + (180º – xº) = 90. Al despejar x, resulta x = 150   A B C En la figura º + º = 255º, entonces ¿m A = ? A. 75º B. 105º C. 127.5º D. 30º E. 45º 12. 25º xº A. 25 B. 50 C. 65 D. 75 E. 130 A B C D 25º 130º P 65 120º xº A B C D E F PROHIBIDA LA VENTA
  • 5. GEOMETRÍA 5 15. Si la medida de un ángulo es tres veces la medida de su suplemento, ¿cuál es la medida de dicho ángulo? A. 30º B. 60º C. 90º D. 120º E. 135º Sean  la medida del ángulo buscado y  la medida de su suplemento, luego  +  = 180   = 180 – . El ejercicio indica que  = 3, luego  = 3 (180 – ) = 540 – 3   4 = 540   = 135. 16. Dos veces la medida de un ángulo es 30° menos que cinco veces la medida de su complemento, ¿cuál es la medida de dicho ángulo? A. 30º B. 60º C. 90º D. 120º E. 135º Sean  la medida del ángulo buscado y  la medida de su complemento, luego  +  = 90   = 90 – . Al interpretar la información del ejercicio se tiene 2 = 5 – 30  2 = 5 (90 – ) – 30 = 450 – 5  – 30  7 = 420   = 60 Sean A, B, C y D los puntos indicados en la figura. Al trazar por B una paralela a  1m y  2m , se forman ángulos alternos – internos entre paralelas, y por tanto congruentes con los ángulos indicados inicialmente, luego x + 60 = 110  x = 110 – 60 = 50. OTRA FORMA: Al prolongar CB, sea D el punto donde corta a la recta  1m Se tiene mADB = x, por ser alterno – interno con el ángulo que se forma en C. El angulo ABC es externo al ABD, luego 60 + x = 110  x = 50  1m y  2m son paralelas.18. En la figura las rectas Entonces el valor de x es A. 170 B. 50 C. 85 D. 25 E. 20 Como las rectas son paralelas se tiene (3x + 10) + (x – 6) = 84  4x + 4 = 84  4x = 80  x = 20 60º 110º xº  2m  1m17. En la figura las rectas  1m y  2m son paralelas. Entonces el valor de x es A. 170 B. 50 C. 85 D. 25 E. 30 A B C xº 60º 60º 110º xº  2m  1m A D B C xº 84° (x – 6)° (3x + 10)°  1m  2m PROHIBIDA LA VENTA
  • 6. GEOMETRÍA 6 Sean  y  las medidas de los ángulos indicados en la figura. Se tiene  +  = 90. Como SQR = 2 y  1  2, se tiene 2mSQR = 180 – . (1) Similarmente se obtiene que 2mQSR = 180 – . (2) Al sumar (1) y (2) resulta 2mSQR + 2mQSR = (180 – ) + (180 – ) = 360 – ( + ) Como  +  = 90, 2(mSQR + mQSR) = 360 – 90 = 270  mSQR + mQSR) = 135 Luego mR = 180 – (mSQR + mQSR) = 180 – 135 = 45 . 20. En una recta se toman los puntos A, B y C, de manera que B es punto medio de __ AC . Se toma otro punto O, tal que B – O – C. Encuentre el valor numérico de OB OCAO  . A. 2 B. 1 C. 2 1 D. 2 3 E. Falta información. Se tiene AB = BC = x, por ser B punto medio de AC. Sea OB = y, luego OC = x – y, AO = x + y. Al sustituir estos valores en la expresión dada se tiene: AO – OC = (x – y) – (x + y) = 2y, OB = y, luego AO OC OB   2y 2 y  Nota: en ejercicios de este tipo no se admite asignar valores arbitrarios, ya que se estaría resolviendo para valores específicos. El planteamiento es general. Cuando se afirma que B – O – C, está indicando que O es un punto cualquiera que se encuentra entre B y C, y el valor numérico encontrado es valido para cualquier punto O que esté entre B y C. P Q S R 1 2 3 4 Si m  P = 90º,  1  2,  3   4, entonces m  R es A. 30º B. 45º C. 60º D. 90º E. Falta información. 19.   A B O C PROHIBIDA LA VENTA
  • 7. GEOMETRÍA 7 2. EJERCICIOS SOBRE TRIÁNGULOS Y CUADRILÁTEROS 1. Un poste cercano a un árbol mide 2 m y su sombra en un momento dado mide 1.8 m, entonces si la sombra del árbol en ese momento mide 11 m, la altura del árbol es A. 11 m B. 11.22 m C. 12. D. 12.22 E. 13 m Dado que los rayos del sol prácticamente caen paralelos y que el poste y el tronco del árbol son perpendiculares al piso, el árbol y su sombra y la línea que une sus extremos forman un triángulo semejante al formado por el poste su sombra y la línea que une sus extremos, tenemos h 2 11 2 h 12.22 11 1.8 1.8      2. Una varilla clavada en el piso y cercana a un árbol mide 3 m y su sombra mide 1.5 m, entonces si el árbol mide 36 m, su sombra mide A. 36 m B. 30 m C. 18 m D. 15 m E. 9 m El problema es similar al anterior, en este caso se tiene x 1.5 36 1.5 x 18 36 3 3      3. El perímetro de un triángulo rectángulo isósceles con hipotenusa igual a 10 redondeado a dos decimales es A. 7.07 B. 14.14 C. 24.14 D. 24.99 E. 50 En un triángulo rectángulo isósceles, la hipotenusa mide 2 x , siendo x la longitud de sus catetos, luego 10 2 x 10 x 5 2 2     Su perímetro será P 10 2 5 2 10 10 2 24.14      2 1.8 11 h 3 1.5 x 36 PROHIBIDA LA VENTA
  • 8. GEOMETRÍA 8 Por el teorema de la altura se tiene 2 4 x 8 64 x 16     y la hipotenusa mide 4 + x = 20 Por el teorema de los catetos se tiene 2 y 4 20 80 y 80 4 5 8.94       5. Un método para encontrar la altura de un edificio es colocar un espejo en el suelo y después situarse de manera que la parte más alta del edificio pueda verse en el espejo ¿qué altura tiene un edificio si una persona cuyos ojos están a 1.5 m del piso observa la parte superior del edificio cuando el espejo está a 120 m del edificio y la persona está a 6 m del espejo? A. 20 m B. 30 m C. 31.5 m D. 120 m E. 126 m Dado que las leyes de la óptica indican que en un espejo plano, el ángulo de incidencia es igual al ángulo de reflexión, se forman dos triángulos rectángulos semejantes, luego h 1.5 h 30 120 6    metros. 6. La altura respecto a la hipotenusa de un triángulo rectángulo mide 10 m y los segmentos que determina sobre la hipotenusa son entre sí como 7 es a 14. Entonces la longitud del cateto menor es A. 4 m B. 7.07 m C. 12.25 m D. 14 m E. 15.5 m Sean m y n los segmentos determinados por la altura sobre la hipotenusa, con m < n, luego m 7 n 2m n 14    . Por el teorema de la altura 2 2 m n 10 m 2m 100 m 50 m 5 2 y n 10 2         La hipotenusa mide c = m + n = 15 2 Por el teorema de los catetos 2 a = m(m + n) = 5 2 15 2 150  a 150 5 6 12.25   8 y x4 4. En el triángulo rectángulo de la figura, los valores de x e y, respectivamente son A. 11 y 13 B. 15 y 16 C. 9 y 8 D. 16 y 8.94 E. 12 y 8.94 h 6 120 1.5 10 a m n PROHIBIDA LA VENTA
  • 9. GEOMETRÍA 9 7. El perímetro de un rectángulo es 85 m y su diagonal mide 32.5 m. Por lo tanto los lados del rectángulo miden: A. 15 m y 27.5 m B. 20 m y 22.5 m C. 7.5 m y 25 m D. 30m y 12.5 m E. 40m y 2.5 m Sean a y b los lados del rectángulo. Se tiene P = 2 (a + b) = 85  a + b = 42.5 (1) 2 2 2 a b 32.5 1056.25   (2) Despejando b de (1), e introduciendo en (2) 2 a + 2 [42.5 a] = 1056.25  2 2a 85a 750 0    a = 12.5  a = 30 Al sustituir en (1) se obtiene b = 30  b = 12.5. 8. El perímetro de un triángulo mide 50 y sus lados son proporcionales a 4, 6 y 8. Entonces su lado mayor mide A. 50/3 B. 25/9 C. 100/9 D. 25 E. 200/9 Sean a, b y c las longitudes de los lados, con a < b < c, luego P = a + b + c =50 y a b c 4 6 8   Por las propiedades de las proporciones a b c a b c 50 8 50 200 c 4 6 8 4 6 8 18 18 9             9. En un triángulo rectángulo, un lado mide 2 106 , otro 5 15 . Si el lado desconocido es el menor, ¿cuánto mide? A. 7 B. 8 C. 9 D. 10 E. 11 Como 2 106 5 15 , la hipotenusa de este triángulo es 2 106 , luego el cateto menor es 2 2 a [2 106 ] [5 15 ]   424 375 49 7   10. El área del triángulo de la figura, redondeada al entero más cercano, mide: A. 21 B. 22 C. 27 D. 31 E. 54 Aplicamos la fórmula de Herón:      A s s a s b s c       , donde s es el semiperímetro. Se tiene 6 7 9 s 11 2     , luego      A 11 11 6 11 7 11 9 11 5 4 2 440 20.97             a b 32.5 6 7 9 PROHIBIDA LA VENTA
  • 10. GEOMETRÍA 10 Dado que es un triángulo rectángulo su área es la mitad del producto de sus catetos. El cateto desconocido mide 2 2 b 10 6 100 36 64 8      Por tanto 1 A 2  (6) (8) = 24 12. Si un rectángulo de 3 m de ancho y 10 m de largo tiene la misma área que un triángulo rectángulo isósceles, entonces la longitud de cada cateto del triángulo es A. 7.5 B. 2 15 C. 15 D. 15 3 E. 10 El área de un triángulo rectángulo isósceles está dada por 21 A x 2  , donde x es la longitud de sus catetos, luego tenemos que el área del rectángulo es 30, por tanto 21 x 30 x 60 2 15 2     13. El área de un trapecio isósceles de bases 22 m y 10 m y cuyos lados congruentes miden 10 es A. 2220 m 2 B. 160 m 2 C. 128 m 2 D. 80 m 2 E. 64 m 2 Por ser un trapecio isósceles, al proyectar la base menor sobre la base mayor, la base mayor queda dividida en tres segmentos de 6, 10 y 6 metros. Aplicando el teorema de Pitágoras, se tiene 2 2 h 10 6 64 8    Aplicando la fórmula para el área de un trapecio:  B b h A 2    resulta   2 22 10 8 A 128 m 2     6 10 ¿Cuál es el área del triángulo de la figura? A. 20 B. 24 C. 30 D. 48 E. 60 11. b 3 10 x x 10 22 10 10 6 10 6 h PROHIBIDA LA VENTA
  • 11. GEOMETRÍA 11 14. La siguiente figura consta de siete cuadrados congruentes. El área total de esta figura es 63 cm 2 . Entonces el perímetro de la figura es: Observamos que el perímetro está formado por 16 veces el lado de cada cuadrado. Como hay siete cuadrados congruentes, cada uno tiene un área de 2 63 x 9 x 3 7     . Por tanto el perímetro de la figura es P = 16  3 = 48 cm. La figura indica que B, D, F y H son puntos medios de los lados del cuadrado ACEG, luego su área es el doble del área del cuadrado BDFH, es decir [ACEG] = 2  162 = 324 luego AC 324 18  16. Se tiene un trapecio ABCD donde __ BC es la base menor. BC = 10 cm. y CD = 20 cm. Las medidas de los ángulos A, B y C son 30°, 150° y 120° respectivamente, entonces AD = ? A. 60 cm. B. 50 cm. C. 40 cm. D. 30 cm. E. 20 cm. Sean B’ y C’ las proyecciones de B y C sobre la base mayor y sean AB’ = x, C’D = y. Por ser BC paralela a AD, mD = 180 – mC = 180 – 120 = 60 El CC’D es un triángulo 30 – 60, luego h = CC’ = 10 3 y y = C’D = 10 También el AB’B resulta ser un triángulo 30 – 60, con su cateto menor BB’ = h = 10 3 , luego AB’ = 10 3 3 30  Tenemos entonces que la base mayor mide AD = x + 10 + y = 30 + 10 +10 = 50. A B C G F E H D Si ACEG es un cuadrado y el área del cuadrilátero BDFH mide 162 ¿cuánto mide AC? (las marcas iguales representan partes congruentes) A. 9 B. 12.72 C. 18 D. 25.44 E. 81 15. A. 16 cm B. 21 cm C. 24 cm D. 48 cm E. 84 cm B 10 C 20 A x B’ 10 C’ y D h 30 60 PROHIBIDA LA VENTA
  • 12. GEOMETRÍA 12 17. Si las medianas en un triángulo rectángulo, trazadas a partir de los vértices de los ángulos agudos miden 5 cm y 40 cm, entonces la medida de la hipotenusa del triángulo rectángulo es A. 2 405  cm B. 2 13 cm C. 45 cm D. 11.32 cm E. 5.66 cm Sean M y N los puntos medios de BC y AB respectivamente. Sean AM = 5 y CN = 40 , BC = a, AB = c, luego a c BM y NB 2 2   . Sea la hipotenusa AC = b Aplicando el teorema de Pitágoras en los ABM y BCN 2 2 2 2 2 a AM AB BM c 25 4      (1) 2 2 2 2 2 c CN NB BC a 40 4      (2) Al sumar (1) y (2) resulta 2 2 2 2 25c 5a 4 65 a c 65 52 b b 52 2 13 4 4 5            18. En la figura, los cuadrados ABCD y EFGH son congruentes. AB = 10 cm y G es el centro del cuadrado ABCD. Entonces el área total cubierta por el polígono AHEFBCDA es Dado que los cuadrados son congruentes sus áreas son iguales y como el lado AB mide 10, cada uno tiene un área de 100 cm 2 , pero ellos comparten el ABG de manera que para el área total del polígono a la suma de las áreas de los cuadrados debemos restarle el área de este triángulo para que sea considerada solo una vez. Dado que G es el centro del cuadrado ABCD, el área del triángulo es la cuarta parte del área del cuadrado o sea 25 cm 2 . Luego el área buscada es A = [ABCD] + [EFGH] – [ABG] = 100 + 100 – 25 = 175 cm 2 D C G A B H F E A. 100 cm 2 B. 120 cm 2 C. 150 cm 2 D. 175 cm 2 E. 200 cm 2 N A B M C PROHIBIDA LA VENTA
  • 13. GEOMETRÍA 13 19. ABCD es un cuadrado, el  ABE es isósceles, CF = FB. Entonces, la medida del ángulo EFB es igual a Como el ABE es isósceles, AE = BE y por ser ABCD un cuadrado, E es el punto medio de DC y por tanto EC = CF, ya que por ser CF = FB, F es punto medio de BC. Luego el ECF resulta ser triangulo rectángulo isósceles y como consecuencia mCFE = 45. El ángulo buscado es el suplemento del CFE, luego mEFB = 180 – mCFE = 180 – 45 = 135 Se tiene que CF = AB = 18, ya que ABCF es un paralelogramo. CE = CF – FE = 18 – 12 = 6. Por otro lado BD y AF son paralelas, luego FAE  CDE, ya que son alternos internos entre paralelas y FEA  CED, ya que son opuestos por el vértice. Como consecuencia se tiene FEA  CED. Sea AE = x, luego ED = AD – AE = 30 – x. Por la semejanza anterior, CE ED 6 30 x 6x 360 12x 18x 360 x 20 FE EA 12 x            21. En un trapecio isósceles, la diferencia de las bases es de 10 m. La altura mide 12 m. y el perímetro 76 m. Entonces su área es: A. 86 m 2 B. 176 m 2 C. 226 m 2 D. 288 m 2 E. 300 m 2 Como B – b = 10, al proyectar la base menor sobre la base mayor se forman tres segmentos de longitudes 5, b y 5 como se muestra en la figura. Luego como la altura es 12, en los extremos del trapecio se forman triángulos rectángulos de catetos 5 y 12. Aplicando el teorema de Pitágoras hallamos que la hipotenusa mide 13 lo cual corresponde a la longitud de los lados no paralelos del trapecio. Considerando que el perímetro mide 76 m. se tiene: 2b + 2 (13) + 2 (5) = 76  b = 20 Al considerar que B – b = 10, resulta B = 30 Aplicando la fórmula para el área de un trapecio, el área buscada resulta     2 B b h 20 30 12 A 300 m 2 2        D E C F A B A. 150° B. 135° C. 90° D. 60° E. 45° B C D A F E En la figura,  ABCF es un paralelogramo. B, C y D son colineales. Si AB = 18, AD = 30 y FE = 12. ¿Cuánto mide AE? A. 10 B. 12 C. 15 D. 20 E. 25 20. 13 B b 13 5 b 5 12 PROHIBIDA LA VENTA
  • 14. GEOMETRÍA 14 22. En la figura ABCD es un cuadrado de lado 1 cm. y CE = 2 cm., entonces el área del triángulo ADF en cm 2 es igual a Dado que ABCD es un cuadrado AD y CE son paralelas, resultando que ADF  ECF por el teorema de semejanza AA, ya que DAF  CEF por ser alternos internos entre paralelas y DFA  CFE por ser opuestos por el vértice. De la semejanza resulta que AD DF 1 DF CF 2DF CE CF 2 CF      (1) Como CD = CF + FD = 1, resulta 1 DF 3  . Por tanto el área buscada resulta 1 1 1 1 [ADF] AD DF 1 2 2 3 6        23. Sea ABC un triángulo isósceles con AB = BC = 10 y AC = 16. Sea BD la mediana trazada sobre el lado AC y sea G el baricentro. Entonces el área del triángulo ADG es A. 6 B. 8 C. 10 D. 12 E. 24 Por ser BD mediana, D es punto medio de AC, o sea AD = DC = 8. Ya que ABC es isósceles, BD además de mediana también es altura, luego mADB = 90. Aplicando el teorema de Pitágoras hallamos que 2 2 BD 10 8 6   Como G es el baricentro, BG = 2GD y como BD = BG + GD = 6, resulta GD = 2. Por tanto el ADG resulta ser un triángulo rectángulo con catetos de longitudes 8 y 2, por tanto su área es 1 1 [ADG] AD DG 8 2 8 2 2        C 2B A D F 1 E A. 2 1 B. 3 1 C. 4 1 D. 6 1 E. 8 1 B G A 8 D 8 C 10 10 16 2 PROHIBIDA LA VENTA
  • 15. GEOMETRÍA 15 24. Sea ABC un triángulo isósceles con AB = AC = 17 cm y P un punto cualquiera del lado BC, diferente de los puntos extremos. Por P se trazan una paralela a AC que corta a AB en Q y una paralela a AB que corta a AC en R. El perímetro del cuadrilátero AQPR es Dado que QP AC y RP AB , AQPR es un paralelogramo y de ahí AQ = RP y AR = QP. Del paralelismo de los segmentos señalados anteriormente también resulta que los QBP y RPC son semejantes con el ABC y por tanto también son isósceles y de ahí QB = QP y RP = RC. Por tanto el perímetro del cuadrilátero AQPR, resulta P = AQ + QP + AR + RP = (AQ + QB) + (AR + RC) = AB + AC = 17 + 17 = 34 25. De acuerdo a la información que se proporciona en la figura, el segmento de mayor longitud es Dado que la suma de los ángulos internos de un triángulo suman 180, en el ABD, resulta que el ángulo ABD mide 180 – 70 – 60 = 50 y en el BDC, mBDC = 180 – 55 – 60 = 65 Una de las propiedades de los triángulos indica que el lado mayor se opone al ángulo mayor y viceversa. Al comparar las medidas de los ángulos del ABD, resulta que el mayor mide 70 y su lado opuesto es BD, luego BD es mayor que AB y AD. Pero al considerar el BDC, su ángulo mayor es 65 y el lado que se le opone es BC y por tanto BC > BD. Luego el lado mayor de la figura resulta BC A 70° B 55° 60° 60° C D A B P C Q R A. 8.5 cm B. 17 cm C. 34 cm D. 51 cm E. 68 cm A. AB B. BC C. CD D. DA E. BD PROHIBIDA LA VENTA
  • 16. GEOMETRÍA 16 26. En la figura ABCD es un cuadrado de lado 1, CMN es equilátero, El área de CMN es igual a El área de un triángulo equilátero está dada por 23 x 4 , donde x es la longitud de su lado, luego debemos encontrar primero cuanto mide cada lado del triángulo equilátero CMN. Como ABCD es un cuadrado y CM = CN = x, se tiene que CDM  CBN y de ahí MD = NB y como AD = AB, resulta AM = AN y por tanto el MAN es rectángulo isósceles, luego x MN x 2 AN AN 2     Como AB = 1, resulta x 2 x NB 1 AN 1 2 2       El CBN es un triángulo rectángulo luego al aplicar el teorema de Pitágoras resulta 2 2 2 2 22 x CN CB NB 1 x 2             Al desarrollar, simplificar y resolver la ecuación resultante se obtiene x 6 2  Por tanto el área buscada es   23 [CMN] 6 2 0.4641 4    27. La siguiente figura muestra dos cuadrados de lado 1 cm., donde AEFG se ha obtenido de ABCD al girar este cuadrado 45° sobre el vértice A. Entonces el área sombreada es Al girar 45, la recta diagonal AC se convierte en la recta AB la cual equivale a la recta diagonal AF, por tanto A, B, F son colineales. Además se tiene mBFH = 45 y por tanto FBH es un triángulo rectángulo isósceles con FB = BH Luego [AGHB] = [AGF] – [FBH] Por ser AEFG un cuadrado de lado 1, su diagonal mide 2 y como AB = 1, BF = BH = 2 1 Como [AGF] tiene como área la mitad de la área del cuadrado, que tiene lado de longitud 1, resulta     21 1 1 1 [AGBH] 2 1 2 2 2 1 2 1 2 2 2 2          A D B H C GE F A N B CD M A. 0.866 B. 0.7071 C. 0.75 D. 0.5 E. 0.4641 A. 2 – 1 B. 0.5 C. 0.451 D. 2 E. 0.375 PROHIBIDA LA VENTA
  • 17. GEOMETRÍA 17 28. Los ángulos agudos de un triángulo rectángulo, que también es isósceles, miden A. 30° B. 45° C. 35° D. 75° E. 60° Por ser triángulo rectángulo isósceles tiene un ángulo de 90 y los otros dos ángulos congruentes, y dado que la suma de los ángulos internos de un triángulo suman 180, cada uno de ellos mide 45 29. En la figura ABCD es un cuadrilátero con AD ||BC . La diagonal AC es perpendicular al lado CD . mBAC = 30°, AC = 4 3 y AB = BC. Entonces el área de ABCD es igual a Como AB = BC, el ABC es isósceles con mABC = 120 y mBCA= 30 y su base AC = 4 3 . Al trazar una perpendicular desde B a AC, sea E el pie de la perpendicular. Por ser isósceles, BE también es mediana es decir E es punto medio de AC, luego se forman dos triángulos 30 – 60 con las hipotenusas AB = BC y catetos mayor AE = EC = AC 2 3 2  Luego como el cateto mayor en un triángulo 30 – 60, es 3 veces el cateto menor, en este caso se tiene BE 2 Como AD ||BC el BAD es el suplemento del ABC, resulta mBAD = 180 – 120 = 60 y como mBAC = 30, se tiene mCAD = 30 y de ahí también el ADC es un triángulo 30 – 60 con cateto mayor AC = 4 3 . Como en todo triangulo 30 – 60, el cateto mayor es 3 el cateto menor, se tiene CD = 4 Finalmente tenemos [ABCD] = [ABC] + [ACD] = 1 1 1 1 AC BE AC CD 4 3 2 4 3 4 12 3 2 2 2 2             30. Se tiene un trapecio ABCD donde BC es la base menor. BC = 10 cm y CD = 20 cm . Las medidas de los ángulos A, B y C son 30°, 150° y 120° respectivamente, entonces el área del trapecio mide A. 300 3 cm 2 . B. 400 cm 2 . C. 300 cm 2 . D. 200 cm 2 . E. 200 3 cm 2 . Sean B’ y C’ las proyecciones de B y C sobre la base mayor y sean AB’ = x, C’D = y. Por ser BC paralela a AD, mD = 180 – mC = 180 – 120 = 60 El CC’D es un triángulo 30 – 60, luego h = CC’ = 10 3 y y = C’D = 10 También el AB’B resulta ser un triángulo 30 – 60, con su cateto menor BB’ = h = 10 3 , luego 10 3 3 30  AB’ = 10 3 3 30  Tenemos entonces que la base mayor mide AD = x + 10 + y = 30 + 10 +10 = 50. Luego el área del trapecio resulta [ABCD] =  50 10 10 3 300 3 2    A D B C A. 6 B. 12 C. 12 3 D. 24 E. 30 30 30 30 60 60 60 E A D B C B 10 C 20 A x B’ 10 C’ y D h 30 60 PROHIBIDA LA VENTA
  • 18. GEOMETRÍA 18 31. En la figura, mBAC =  , mBPC = mBQC = 90°. Entonces la medida de BHC es Como mBPC = mBQC = 90°, también mAPH = mAQH = 90°. APHQ es un cuadrilátero convexo y en todo cuadrilátero convexo la suma de sus ángulos internos es 360, luego mBHC +  + 90 + 90 = 360 y de ahí mBHC = 180 –  32. Si las medianas en un triángulo rectángulo, trazadas a partir de los vértices de los ángulos agudos miden 5 cm y 20 cm, entonces la medida en cm de la hipotenusa del triángulo rectángulo es A. 5 B. 6 C. 8 D. 9 E. 10 Sean M y N los puntos medios de BC y AB respectivamente. Sean AM = 20 y CN = 5, BC = a, AB = c, luego a c BM y NB 2 2   . Sea la hipotenusa AC = b Aplicando el teorema de Pitágoras en los ABM y BCN 2 2 2 2 2 a AM AB BM c 20 4      (1) 2 2 2 2 2 c CN NB BC a 25 4      (2) Al sumar (1) y (2) resulta 2 2 2 2 25c 5a 4 45 a c 45 36 b b 36 6 4 4 5            B C P H Q A  A. 180 –  B.  C. 90 –  D. 2 E. 3 N A B M C PROHIBIDA LA VENTA
  • 19. GEOMETRÍA 19 33. En la figura, los dos cuadrados tienen el mismo centro. La razón entre el lado del cuadrado menor y el lado del cuadrado mayor es 2/5. Entonces la razón entre el área sombreada y el área del cuadrado mayor es Sean “b” la longitud del lado del cuadrado menor y “a” la longitud del lado del cuadrado mayor, luego b 2 a 5  Por la simetría de la figura se deduce que el área sombreada, es decir el trapecio ABFE, representa la cuarta parte de la diferencia entre los dos cuadrados, luego   1 [ABFE] [ABCD] [EFGH] 4   2 2 [ABCD] a y [EFGH] b  y de ahí  2 21 [ABFE] a b 4   La razón buscada será  2 2 2 2 2 2 2 2 1 a b [ABFE] 1 a b 1 b4 1 [ABCD] 4 4a a a                     Como b 2 a 5  , resulta [ABFE] 1 4 21 1 [ABCD] 4 25 100         34. En la figura, AB = AC = 4, BD = DC = 3 y mBAC = 60°, entonces la longitud del segmento AD es Al unir B con C, obtenemos un triángulo equilátero, ya que AB = AC y mBAC = 60°. Se tiene que ABD  ACE, ya que sus tres pares de lados son congruentes, de ahí resulta mBAD = mCAD y por tanto AD es bisectriz del BAC Al prolongar AD, sea E el punto donde corta a BC. Luego como el ABC es equilátero, AE además de bisectriz es mediatriz y por tanto AE  BC y BE = EC = 2. Resulta entonces que el BED es rectángulo, con hipotenusa BD = 3 y un cateto, BE = 2. Por el Teorema de Pitágoras, 2 2 DE 3 2 5   Por otro lado AE es una altura en un triángulo equilátero de lado 4 y por tanto AE 2 3 Finalmente obtenemos que AD = AE – DE = 2 3 5 A CB E D A a B D C H G E F b A. 1/6 B. 21/100 C. 1/3 D. 2/5 E. 4/9 A. 2 3 – 5 B. 2 3 + 5 C. 1 D. 2 E. 3.5 PROHIBIDA LA VENTA
  • 20. GEOMETRÍA 20 35. En la figura el cuadrilátero ACDE es un trapecio tal que ED = 15 cm., AC = 24 cm y la altura es 12 cm. Sabiendo que B es el punto medio del lado AC, el área del cuadrilátero OBCD es Como ED || A C , resulta que ABO  DEO, con razón de semejanza AB 12 4 DE 15 5   Sean a y b las alturas de los triángulos ABO y DEO respectivamente, indicadas en la figura. Dado que los elementos homólogos en triángulos semejantes están en la misma razón de semejanza, se tiene a 4 b 5  . Como a + b = 12 (la altura del trapecio), al considerar la razón anterior resulta 16 20 a , b 3 3   . Al analizar la figura vemos que [OBCD] = [ACDE] – [ABE] – [DEO] Tenemos que el área del trapecio ACDE resulta 24 15 [ACDE] 12 234 2         El ABE, tiene base 12 y altura 12, luego su área es 1 [ABE] 12 12 72 2     Para el DEO, resulta 1 20 [DEO] 15 50 2 3      [OBCD] = 234 – 72 – 50 = 112 36. En la figura, ABCD es un cuadrado de lado 6 cm. y CE = DE = 5 cm., entonces la longitud de AE es A 109 cm B. 15 cm C. 11 D. 30 E. 61 Sean F y G los puntos medios de CD y BA respectivamente. Luego CF = FD = BG = GA = 3 y FG = 6 Como CE = DE, el CED es isósceles y por tanto E, F, G son colineales y EF  CD y EG  AB. El CFE es rectángulo en F, luego por el Teorema de Pitágoras, 2 2 EF 5 3 4   . EG = EF + FG = 4 + 6 = 10 El FGA también es rectángulo con EG = 10 y GA = 4, luego 2 2 EA 10 3 109   E 15 D O A 12 B 12 C 12 b a A. 112 cm 2 B. 117 cm 2 C. 120 cm 2 D. 140 cm 2 E. 360 cm 2 E C F D B G A PROHIBIDA LA VENTA
  • 21. GEOMETRÍA 21 37. En la figura, a partir de la información dada, ¿cuál es el valor de x? A. 76 B. 25 C. 13.2 D. 5.28 E. 5 Se tiene A  E , por dato, y ACB  ECD, por ser opuestos por el vértice, luego ABC  EDC, por el teorema de semejanza AA. Luego CD BC x 66 x 5 CE AC 10 132      38. ABCD es un paralelogramo. P es un punto de la diagonal AC . Trazamos por P paralelas a los lados del paralelogramo. Estas paralelas intersecan a los lados del paralelogramo en los puntos indicados en la figura. Sabiendo que el área de ABCD es 40 cm 2 , entonces el área del cuadrilátero RQMN es igual a Dado que RM AD BC y NQ AB DC , resulta que los cuadriláteros ANPR, PQBR, DMPN y MCQP son paralelogramos y los segmentos NR, RQ, QM y MN son diagonales de esos paralelogramos. Es sabido que una diagonal divide a un paralelogramo en dos triángulos congruentes, con áreas igual a la mitad del área del paralelogramo. Por tanto [RQMN] = 1 2 [ABCD] = 20 Basta aplicar el teorema del cateto: 2 3x 6 x 12   A B C 3 6 x En el triángulo rectángulo ABC ¿cuál es la longitud del segmento BC? A. 15 B. 12 C. 10 D. 9 E. 7.5 39 A R B N P Q D M C A. 10 cm 2 B 20 cm 2 C. 30 cm 2 D. 40 cm 2 E. 50 cm 2 x 10 52.8 132 66 A B C E D PROHIBIDA LA VENTA
  • 22. GEOMETRÍA 22 40. Sea ABCD un cuadrado. Por el vértice A se traza un segmento que corta a la prolongación del lado BC en E, al lado DC en F y a la diagonal BD en G. Si AG = 3 y GF = 1 ¿cuál es la longitud de FE? Sea x la longitud de cada lado del cuadrado. Desde G tracemos una perpendicular a AD y sea H el pie de esta perpendicular. Luego AGH  AFD. Como AG = 3 y GF = 1, resulta AF = 4 De la semejanza se tiene AG AH 3 AH 3 AH x AF AD 4 x 4      y HD = AD – AH = 3 1 x x x 4 4    Como G esta sobre la diagonal, HG = HD = 1 x 4 De la misma semejanza se tiene x AG HG 3 14 DF x AF DF 4 DF 3      . Luego FC = DC – DF = 1 2 x x x 3 3   Como ABCD es un cuadrado, AD BC y por tanto AD CE . De ahí resulta que ADF  ECF De esta semejanza se tiene 2 x FE FC FE 3 FE 8 1FA FD 4 x 3      A H D G F B C E A. 12 B. 10 C. 9 D. 8 E. 6 PROHIBIDA LA VENTA
  • 23. GEOMETRÍA 23 3. EJERCICIOS SOBRE CIRCUNFERENCIAS Y POLÍGONOS Tenemos que mAC mCD 2    Dado que mAB mBC mCD mAD 360 y mBC mAD 80 140 220           , tenemos que mAC mCD 140   , luego mAC mCD 140 70 2 2        2. El triángulo ABC está inscrito en un semicírculo de diámetro AB. Si AC = 8 y BC = 6, el área de la región sombreada tiene un valor de A. 15.27 B. 24 C. 36.37 D. 61.07 E. 48 El área sombreada es la diferencia entre el área del semicírculo y el área del triángulo. El ABC es rectángulo en C, por estar inscrito en un semicírculo. Luego por el Teorema de Pitágoras, 2 2 AB 8 6 10    r = 5 Por tanto el área del semicírculo es 2 21 1 25 r 5 2 2 2      El área del triángulo está dada por 1 1 AC BC 8 6 24 2 2       El área buscada es A = 25 24 15.27 2    3. El triángulo ABC está inscrito en un semicírculo de diámetro AB. Si AC = 8 y CD = 4.8, el área de la región sombreada tiene un valor de A. 15.27 B. 24 C. 36.37 D. 61.07 E. 48 Por el Teorema de Pitágoras, 2 2 AD 8 4.8 6.4   Como el ABC es rectángulo en C, se tiene por el teorema del cateto 2 2 64 AC AB AD 8 AB 6.4 AB 10 6.4         . Y de nuevo por el Teorema de Pitágoras resulta BC = 6. Dado que estos valores coinciden con los datos del ejercicio anterior, el área resulta la misma.  En la figura de la derecha si la medida de los arcos AD y BC son 140º y 80° respectivamente, entonces el valor de  es A. 40° B. 50° C. 60° D. 70° E. 80° A  B C D 1. A D B C A B C PROHIBIDA LA VENTA
  • 24. GEOMETRÍA 24 Sea  la medida del ángulo central XOZ. La longitud de un arco está dada por s = r, con el ángulo medido en radianes. Tenemos s =  y r = 2, luego s r 2     , es decir 90 Luego XOZ es un triángulo rectángulo isósceles con XZ como hipotenusa y por tanto XZ = 2 2 5. En la figura el área del círculo mayor es 1 m 2 . El círculo menor es tangente internamente al círculo mayor y también es tangente a los lados del ángulo inscrito que mide 60°. El vértice del ángulo inscrito y los centros de los círculos están alineados. Entonces el área del círculo menor es Desde el vértice del ángulo inscrito, trazamos un diámetro. Sean O y O’ los centros de los círculos, mayor y menor respectivamente. Sea B el otro extremo del diámetro trazado, C el punto donde uno de los lados (el arriba) del ángulo corta a la circunferencia. Y sea D el punto de tangencia de este lado del ángulo con el círculo menor. Sean R y r los radios de los círculos, mayor y menor respectivamente. Tenemos que AO’ biseca al ángulo inscrito, luego mO’AD = mBAC = 30. Como AB es un diámetro del circulo mayor, mACB = 90 resultando que mABC = 60, y el  ABC es 30 – 60. Dado que AB = 2R, se obtiene que BC = R, ya que BC es el cateto menor y AB la hipotenusa del ABC Como AC es tangente al círculo menor en D, AD  DO’, es decir mADO’ = 90 y de ahí mAO’D = 60 Luego también el AO’D es un triángulo 30 – 60 y su cateto menor O’D = r, mide la mitad de su hipotenusa, AO’. Se tiene O’B = r, por ser radio del circulo menor y de ahí AO’ = AB – O’B = 2R – r. Luego AO’ = 2 O’D  2R – r = 2 r  2 r R 3  . Como el área del círculo mayor es 2 R 1  , el área del círculo menor es 2 2 22R 4 4 r R 3 9 9            X Y Z La circunferencia de la figura tiene radio 2 y el arco XYZ tiene longitud . ¿Cuánto mide la cuerda XZ? A. 2 B. 2 C. 2 2 D. 2  E.  4. A. 2 1 B. 9 4 C.  D. 2 E.  2 1 . X Z O  C A O O’ B D PROHIBIDA LA VENTA
  • 25. GEOMETRÍA 25 Como TP es tangente a la circunferencia en T, mPTC = 90. Luego aplicando el teorema de Pitágoras resulta   2 2 2 PC 2r r 5r r 5    Como el área sombreada únicamente son los extremos y estos son semicírculos, al unirlos se forma un circulo de diámetro 8, es decir de radio 4, luego 2 2 A r 4 16      Dado que m AB = 50°, se tiene mBCA = 1 mAB 2 = 25°, por ser ángulo inscrito que subtiende dicho arco; mABC = 90, por estar inscrito en una semicircunferencia ( ___ AC es diámetro). Luego la medida del ángulo buscado es: mBAC = 180 – mBCA – mABC = 180 – 25 – 90 = 65 C Q PT En la figura C es el centro de la circunferencia de radio r y __ TP es un segmento tangente en T, de longitud 2r, entonces PC mide A. r 2 B. r 3 C. 3r D. r 5 E. 5r 6. r 2r 10 8 Los extremos de la figura son semicírculos, ¿Cuál es el área de la región sombreada? A. 80 B. 8 C. 10 D. 16 E. 16 + 80 7. O A B C En la figura ___ AC es un diámetro. Si m AB = 50°, entonces m  BAC = ? A. 25° B. 50° C. 65° D. 90° E. 130° 8. PROHIBIDA LA VENTA
  • 26. GEOMETRÍA 26 El área de la región sombreada es la diferencia entre el área del cuadrado formado y las áreas de los cuatro sectores circulares que se forman. Dado que la distancia entre los centros de dos círculos tangentes exteriormente, es la suma de las longitudes de los radios, resulta que el cuadrado formado tiene lado de longitud 20 y por tanto el área del cuadrado es 400. Cada sector formado tiene un ángulo central de 90, luego entre los cuatro forman un circulo de radio 10, cuyas áreas suman entonces 2 2 r 10 100     Por tanto el área buscada es: A = 400 – 100 Dado que BPA es un ángulo exterior, formado por dos secantes, su medida es la semidiferencia de los las medidas de los arcos que intercepta. Es decir mCD mAB m BPA 2    . De esta expresión despejamos mAB , resultando mCD 2 m BPA mAB    Introduciendo los datos se obtiene mCD 2 35 30 100       Por otro lado se tiene que el DAC es un ángulo inscrito que subtiende el arco DC, luego 1 1 m DAC mCD 100 50 2 2        En la figura, los círculos son tangentes y tienen radio igual a 10. Si se unen los centros de los círculos se forma un cuadrado. ¿Cuál es el área de la región sombreada? A. (400 – 100) B. 400 – 100 C. 100 – 400 D. 400 – 100 E. 400 – 400 9. D B P A C En la figura, la medida del arco AB es 30°, y la medida del BPA es 35°. Las medidas del arco CD y el ángulo DAC (en grados) son respectivamente A. 100 y 25 B. 50 y 50 C. 100 y 50 D. 50 y 25 E. 25 y 50 10. PROHIBIDA LA VENTA
  • 27. GEOMETRÍA 27 11. La expresión (p + q) p = (r + s) r, se cumple en la situación representada por Al recordar las relaciones métricas en una circunferencia, vemos que los productos de esta forma surgen cuando se tienen dos secantes que se cortan (también aparecen cuando hay semejanzas de triángulos) o una secante y una tangente que se cortan. A partir de estas relaciones tenemos: En la figura A, la relación es  2 r s s p   En la figura B, la relación es r  (r + s) = p  (p + q), la cual es la misma expresión dada. La respuesta es ésta. Para estar más seguros vemos que resulta en las otras. En la figura C, la relación es r  s = p  q y en la figura C,  2 r p p q   , que son diferentes a la dada. Solo B satisface y por tanto es la respuesta. 12. En la figura se dan tres semicircunferencias mutuamente tangentes. DAyCD son diámetros de las circunferencias menores. El punto B está en la semicircunferencia mayor. BD  CA . Si BD = 2, entonces el área sombreada es igual a El área de la región sombreada es la diferencia entre el área del semicírculo exterior menos las áreas de los semicírculos interiores. Sean 1 2r , r , R los radios del semicírculo menor, del semicírculo mediano y del semicírculo exterior respectivamente. Luego CD = 12r , DA = 22r y CA = 2 R Como CA = CD + DA, se tiene 2R = 1 22r 2r o sea R = 1 2r r (1) Al unir B con A y con C, se forma un triángulo rectángulo, con CA como hipotenusa y BD como altura relativa a la hipotenusa. Por el teorema de la altura, 2 2 1 2 1 2BD CD DA 2 2r 2 r r r 1        (2) Como el área de un semicírculo está dada por 21 r 2  , el área buscada es  2 2 2 1 2 1 A R r r 2     Al considerar (1)   2 2 2 2 1 2 1 2 1 1 1 A [ r r r r )] r 2 2        2 1 2 22r r r  2 1r 2 2r  1 2  2 1 2r r Al considerar (2) 1 2A r r   r r r r s s s p p p p sq q q q A B C D C D A B A. 1 B.  C. 2 D. 4 3 E. 4 9 PROHIBIDA LA VENTA
  • 28. GEOMETRÍA 28 13. Las medidas de los arcos AB y AC se indican en la figura. La medida del BAC es El BAC es un ángulo inscrito en una circunferencia, por tanto su medida es la mitad de la medida del arco que subtiende, en este caso el arco BC. Tenemos que mBC 360 mAB mAC 360 110 130 120            , luego mBAC = 1 mBC 60 2   14. En la figura, BC une los centros de los círculos tangentes. AB  BC , BC = 8 y AC = 10, entonces la longitud de la circunferencia pequeña es igual a Sean R y r los radios de las circunferencias grande y pequeña respectivamente. Como las circunferencias son tangentes exteriormente, R + r = BC = 8 Dado que el ABC es rectángulo en B, tenemos 2 2 R AB 10 8 6    y r = 2. Luego la longitud de la circunferencia pequeña resulta C = 2  r = 4  Todo hexágono regular puede dividirse en seis triángulos equiláteros congruentes. En la figura se indica que x equivale al doble de la altura de cada triangulo: x = 2h Como el lado de cada triangulo mide 6 3 , las alturas miden 3 h 6 3 9 x 2h 18 2       x 36 15. A B C A B C 130° 110° A. 55° B. 60° C. 65° D. 110° E. 130° A.  B. 2 C. 3 D. 4 E. 5 La figura representa un hexágono regular, ¿cuál es el valor de x? A. 3 3 B. 6 3 C. 6 D. 18 E. 9 3 PROHIBIDA LA VENTA
  • 29. GEOMETRÍA 29 Si llamamos 1A al área del cuadrado mayor, 2A al área del cuadrado menor y 3A al área del círculo, el área de la región sombreada resulta 1 2 3A A A A   El lado del cuadrado mayor mide 0.4, luego su área es 1A = 0.16 Como B es punto medio AB = 0.2. Los triángulos que se forman en cada esquina del cuadrado mayor, son rectángulos isósceles, y sus hipotenusas forman los lados del cuadrado menor, por tanto, el lado del cuadrado menor resulta 0.2 2 y su área es 2A =   2 0.2 2 0.08 Como el circulo está inscrito en el cuadrado menor, su diámetro es el lado de dicho cuadrado, y su radio es la mitad o sea r = 0.1 2 , su área   2 2 3A r 0.1 2 0.0628     Luego A = 0.16 – 0.08 +0.0628 =0.1428 Dado que PB y PC son segmentos tangentes a la circunferencia de la izquierda, desde un mismo punto, son congruentes, luego PC = PB = 19. Como AC = AP + PC, AP = AC – PC = 31 – 19 = 12. 18. Seis triángulos equiláteros de 1 cm. de lado se unen para formar un hexágono como se muestra en la figura. Se circunscribe un círculo alrededor del hexágono ¿cuál es el área de la región sombreada? Tenemos que el área de la región sombreada es el área del circulo menos el área del hexágono. El radio del circulo es la longitud del lado de los triángulos, es decir r = 1, luego su área es 2 r   El área de cada triángulo equilátero es 23 x 4 , donde x es el lado del triángulo, y como el lado mide 1, se reduce a 3 4 . Como hay seis triángulos, el área del hexágono es 3 3 3 6 4 2   Por tanto el área de la región sombreada es A = ) 2 3 (  cm 2 A B P D C Los segmentos AC y BD se cortan en P y son tangentes a las circunferencias en los puntos A, C, B y D. Si AC = 31, PB = 19 ¿Cuál es el valor de AP? A. 6 B. 12 C. 15 D. 25 E. 50 17. 0.4 La figura representa un círculo inscrito en un cuadrado que a su vez está inscrito en otro cuadrado. B es punto medio de AC ¿Cuál es el área de la región sombreada? A. 0.025 B. 0.048 C. 0.1428 D. 0.153 E. 0.1582 A B C A. ) 2 3 (  cm 2 B. ) 2 33 (  cm 2 C. ) 2 3 ( 2 cm 2 D. 3 3 cm 2 E. )3( 32  cm 2 16. PROHIBIDA LA VENTA
  • 30. GEOMETRÍA 30 19. Un triángulo ABC está inscrito en una circunferencia como se muestra en la figura. Se tiene m A = 50º y m C = 60º. Se trazan tangentes por A, B y C de manera que se forma el triángulo circunscrito A’B’C’. Entonces la medida del ángulo A’ es: A. 40º B. 60º C. 80º D. 100º E. 120º Como BA’ y CA’ son tangentes a la circunferencia, los A’BC y A’CB son ángulos semiinscritos que subtienden el arco BC y el ángulo A es un ángulo inscrito que subtiende el mismo arco. Por tanto estos ángulos son congruentes, es decir mA’BC = mA’CB = mA = 50 Luego al considerar el A’BC, se tiene mA’ = 180 – mA’BC – mA’CB = 180 – 50 – 50 = 80 20. El triángulo ABC es equilátero y sus lados AC y BC son tangentes a la circunferencia con centro en O y radio 3 . El área del cuadrilátero AOBC es A. 3 B. 6 C 3 3 D. 6 E. 12 Se tiene OC  AB, ya que los triángulos OAB y ABC son isósceles. También OAC  OBC, ya que sus tres lados son congruentes. Como además el ABC es equilátero, mACO = 30, luego el OAC es un triángulo 30 – 60 y de ahí resulta que OC = 2 3 y AC = 3. Tenemos entonces [AOBC] = 2[OAC] = 1 2 3 3 3 3 2     21. Si un ángulo central de 30° en una circunferencia intercepta un arco de 6 m de longitud, entonces el radio de la circunferencia mide A. /36 B. /6 C.  D. 36/ E. 180 Se tiene s = r , donde s es la longitud del arco, r el radio de la circunferencia y  es el ángulo central correspondiente, medido en radianes. Como  = 30 equivale a /6 radianes, tenemos 36 6 r r 6       C C’ B B’ A A’ 50 A B O C 3 PROHIBIDA LA VENTA
  • 31. GEOMETRÍA 31 22. En la figura se tiene una circunferencia de radio 1 y un hexágono regular de lado 1. Si O es el centro de la circunferencia, entonces el área de la región sombreada es A. 0.5 B. 0.866 C. 1 D. 1.5 E. 2 En vista que el hexágono tiene lado 1 y la circunferencia tiene radio 1, el centro del hexágono es un punto de la circunferencia. La región sombreada puede descomponerse en dos triángulos que tienen la misma base y la misma altura que los triángulos que forman el hexágono. Luego el área buscada es A = 2 23 3 1 0.866 4 2    Sean 1r el radio del semicírculo mayor, 2r el radio del semicírculo mediano y 3r el radio del semicírculo menor. Se tiene 1 2 2 1 1 AB 3 r , BC 2r 2AB r AB, 2r AB BC 3AB r AB 2 2           Luego las áreas de estos semicírculos son: Semicírculo mayor: 2 2 2 1 1 1 3 9 r AB AB 2 2 2 8           Semicírculo mediano: 2 2 2 1 1 r AB 2 2    Semicírculo menor: 2 2 2 3 1 1 AB 1 r AB 2 2 2 8           El área no sombreada está dada por: área del semicírculo mayor más el área del semicírculo mediano menos el área del semicírculo menor o sea Área no sombreada = 2 2 2 29 1 1 3 AB AB AB AB 8 2 8 2        El área sombreada está dada por: área del semicírculo mayor menos el área del semicírculo mediano más el área del semicírculo menor o sea Área sombreada = 2 2 2 29 1 1 3 AB AB AB AB 8 2 8 4        La razón buscada resulta 2 2 3 AB Area sombreada 14 3Area no sombreada 2 AB 2     A B C Los arcos AB y BC son semicírculos cuyos centros están sobre un diámetro del círculo que se muestra en la figura. Si BC = 2 AB, entonces la razón entre el área de la región sombreada y el área de la región no sombreada es: A. 2 B. 2 3 C. 1 D. 3 2 E. 2 1 23 O’ O A C B PROHIBIDA LA VENTA
  • 32. GEOMETRÍA 32 24. Una moneda circular de radio 1, está sobre una mesa. Si ponemos cuatro monedas más grandes de igual tamaño alrededor de ella, ¿cuál es el radio de las monedas grandes que permite que cada una sea tangente a las dos adyacentes y a la de radio 1? A. 1 B. 1 + 2 C. 2 D. 2 + 2 E. 2.5 Sea R el radio de las monedas grandes. Como estas monedas son tangentes a las monedas adyacentes y a la vez son tangentes a la moneda pequeña, al unir los centros de las monedas grandes se forma un cuadrado de lado 2R. Al trazar una diagonal, esta debe pasar por el centro de la moneda pequeña, la cual tiene diámetro 2, luego la longitud de la diagonal resulta 2R + 2. Por tanto, dado que en todo cuadrado de lado x, su diagonal mide 2 x , se cumple en este caso que Al racionalizar el denominador obtenemos R 2 1  25. En la siguiente figura ABC y AEB son semicírculos, F es el punto medio del diámetro AC, B es punto medio del arco AC y AF = 1¿Cuál es el área de la región sombreada? El área de la región sombreada resulta de la diferencia entre el semicírculo AEB y el segmento circular determinado por la cuerda AB en el semicírculo ABC. Como F es el punto medio del diámetro AC, B es punto medio del arco AC, resulta BF  AC, luego el ABF es un triángulo rectángulo isósceles de cateto 1 y por tanto AB = 2 AB es diámetro del semicírculo AEB, luego su radio es 2 2 y el área de este semicírculo resulta 2 1 1 2 A 2 2 4             El área del segmento circular, está dada por la diferencia entre el área del sector circular que lo contiene y el área del triángulo determinado por la cuerda y los radios extremos. En este caso el sector circular correspondiente tiene ángulo central de 90 y radio 1, por tanto su área es la cuarta parte del área de un círculo de radio 1 o sea 1 4  y el triángulo correspondiente tiene base 1 y altura 1, luego su área es 1 2 . El área del segmento circular resulta 2 1 1 A 4 2    Finalmente el área buscada es 1 2 1 1 A A A 4 4 2 2             A. 1/2 B. 2 C. /4 D. 3/4 E. /4 – 1/2 A B F C E R R R 2 R R R     1 2R 2 2 2R 2R 2 2 2 R 2 2 2 R 2 R 2 1            PROHIBIDA LA VENTA
  • 33. GEOMETRÍA 33 26. Si el radio de un círculo aumenta en  unidades, ¿cuánto aumenta su perímetro? A.  B. 2 C. 3 D.  2 E. 2 2 Sean L y L’ los perímetros del círculo original y el círculo con el radio aumentado, respectivamente. Luego   2 L 2 r y L' 2 r 2 r 2          El aumento es la diferencia  = L’ – L =  2 2 2 r 2 2 r 2       27. Dos semicírculos de radio 3 están inscritos en un semicírculo de radio 6 como se muestra en la figura. Un círculo de radio r es tangente a los tres semicírculos. ¿Cuánto vale r ? Cuando se tienen círculos tangentes exteriormente, la distancia entre los centros es la suma de los radios, y cuando son tangentes interiormente, la distancia entre los centros es la diferencia entre los radios. Además en ambos casos los centros y el punto de tangencia están alineados. Sean A, B, C y D los centros de los semicírculos y del círculo interior como se muestra en la figura. Se tiene AB = AD = 3 + r, CA = 6 – r, BC = CD = 3. Como ABD es isósceles y C es punto medio de BD, AC  BC, luego el ABC es rectángulo en C y por tanto sus lados cumplen con el teorema de Pitágoras. Luego     2 2 2 3 r 6 r 3 9     2 6r r  2 36 12r r   9 18r 36 r 2    r r r A 3 3 B C D A B 3 C 3 + r 6 – r D3 3 r A. 1 B. 1.5 C. 2 D. 2.5 E. 3 r r +  PROHIBIDA LA VENTA
  • 34. GEOMETRÍA 34 Como el ABC es rectángulo en C, aplicamos el Teorema de Pitágoras para hallar BC: 2 2 2 2 BC AB AC 5 3 4     Como BC es diámetro del círculo, se tiene r = 2 y su área resulta 2 A r 4    30. El lado mayor del rectángulo de la figura mide 20. La curva trazada en su interior está formada por cinco semicircunferencias ¿cuál es la longitud de la curva? Se observa que la curva está formada por 5 semicircunferencias, cuyos diámetros suman 20, luego cada diámetro mide 20  5 = 4 y los respectivos radios la mitad o sea 2 unidades. Luego L = 1 5 2 r 5 r 5 2 10 2          Al considerar el círculo central y dos círculos externos contiguos, vemos que encierran la sexta parte del área buscada. Vemos también que esta fracción corresponde al área de un triángulo equilátero de lado 2 menos tres sectores circulares de radio 1 y de 60º cada uno, que juntos forman un semicírculo de radio1. Luego A = 6 [    2 23 1 2 1 4 2   ] = (6 3 – 3) u 2 En la figura los círculos adyacentes son tangentes y tienen radio 1. ¿Cuánto vale el área de la región sombreada? A. 6 3 – 3 B. 3 3 – 2 C. 2 – 1 D. 6 3 – 1 E. 6 – 3 28. B O C A En la figura, m  BCA = 90º, BA = 5 y AC = 3. ¿Cuál es el área del círculo con centro en O? A. 16 B. 8 C. 6 D. 5 E. 4 29. A. 25  B. 20 C. 15 D. 10 E. 5 PROHIBIDA LA VENTA
  • 35. GEOMETRÍA 35 Sea r el radio de la circunferencia buscado. Sean A y C los centros de las circunferencias, pequeño y grande respectivamente. Desde A y C trazamos perpendiculares a los segmentos perpendiculares iniciales, formando el cuadrado rotulado en la figura como ABCD. Sean E, F, G y H los puntos donde estas perpendiculares cortan a los segmentos perpendiculares, como se indica en la figura. Tenemos que AE = AG = DH = BF= 1, el radio de la circunferencia pequeña. Como CH = BG = CF = r, tenemos que CD = CB = BA = DA = r – 1, luego por esto y la perpendicularidad anterior ABCD es un cuadrado. Como las circunferencias son tangentes exteriormente, la distancia entre sus centros es la suma de sus radios, es decir AC = r + 1. Luego el ABC es un triángulo isósceles, rectángulo en B, con AB = BC = r – 1 y AC = r + 1. Luego AC = 2 AB, es decir r + 1 = 2 (r – 1). Al despejar r, se obtiene r = 3 + 2 2 32. Tres círculos de radio 1, con sus centros colineales son tangentes como se muestra en la figura. ¿Cuál es el área de la región sombreada? Rotulemos los puntos extremos de la región sombreada, como se muestra en la figura, vemos que se forma un rectángulo. En los extremos de la región se tienen dos semicírculos, que juntos forman un circulo. Luego la región sombreada es la diferencia entre las áreas del rectángulo y los dos círculos que se forman. Dado que el radio de los círculos es 1, AD = 2 y AB = 4. Luego A = 2 A 2 4 2 1 8 2        A. 8 – 2 B. 4 –  C. 12 – 3 D. 8 – 3 E. 4 +  A 4 B D C 2 La figura muestra dos segmentos perpendiculares tangentes a ambas circunferencias, las cuales son tangentes entre sí. Si el radio de la circunferencia pequeña mide 1, entonces el radio de la circunferencia más grande mide A. 3 + 2 2 B. 4 C. 6 D. 4 + 2 2 E. 8 31. G A B H D C E F PROHIBIDA LA VENTA
  • 36. GEOMETRÍA 36 33. La figura muestra un hexágono regular inscrito en un círculo. Si el área del círculo es 1, ¿cuánto mide el área del triángulo ABC? Se observa que los triángulos ABC y ABO tienen la misma área, ya que tienen la misma base y la misma altura. Por ser un hexágono regular el ABO es un triángulo equilátero de lado igual al radio del círculo. Como el área del circulo es 1, se tiene 2 1 r 1 r     Luego el área del triángulo es 2 3 1 3 [ABC] 4 4        34. ¿Qué polígono regular tiene la misma cantidad de diagonales que de lados? A. Pentágono B. Hexágono C. Octógono D. Decágono E. Dodecágono Como el número de diagonales en un polígono está dado por  n n 3 D 2   , donde es el número de lados del polígono. Luego    2 2 n n 3 n n 3n 2n n 5n 0 n n 5 0 n 5 2              . Se descarta n = 0, por carecer de sentido. Por tanto el polígono buscado es un pentágono. 35. Sean O el centro de una circunferencia de radio r y ED = r. Si mDEC = k  (m BOA), entonces el valor de k es: Trazamos el radio OD y vemos que el ODE es isósceles ya que OD = ED = r, luego DEC  DOC. Como elDEC es un ángulo exterior con sus lados secantes a la circunferencia, su medida está dada por mAB mCD m DEC 2    . (1) Como los ángulos BOA y DOC, sus medidas están dadas por m BOA mAB y m DOC mDC m DEC      Se tiene mDEC = k  (m BOA) = k mAB mDC   Sustituyendo en (1): mAB k mAB 1 k mAB 2k mAB mAB k mAB 3k 1 k 2 3              A O C E B D A. 3 1 B. 2 1 C. 1 D. 2 E. 3 A B C O  A. 6 1 B. 6  C. 4 3 D. 4 3 E. 12  PROHIBIDA LA VENTA
  • 37. GEOMETRÍA 37 36. Si se aumenta el radio de un círculo en un 100%, ¿en qué porcentaje aumenta su área? A. 50% B. 100% C. 200% D. 100% E. 300% Si el radio original es r, el circulo con el radio aumentado, tiene radio 2r. Se tiene   22 2 1 2A r y A 2r 4 r      El aumento está dado por 2 2 2 2 1A A A 4 r r 3 r         Porcentaje de aumento: 2 1 3 rA 100% A    2 r 100% 300%  37. Se tienen tres círculos concéntricos de radios 1, 2 y 3 respectivamente. ¿Cuál es la razón entre el área de la región cuadriculada y el área de la región oscura? El circulo pequeño tiene área , ya que su radio es 1. El círculo mediano tiene área 4, ya que su radio es 2 El círculo grande tiene área 9, ya que su radio es 3. . Área de la región oscura = área del circulo grande – área del circulo mediano = 9 – 4 = 5 Área de la región cuadriculada = área del circulo mediano – área del circulo pequeño = 4 –  = 3 área de la región cuadriculada 3 3 área de la región oscura 5 5     38. El segmento AB es diámetro de una circunferencia de radio 1 y lado del triángulo equilátero ABC. Si la circunferencia corta a AC y BC en los puntos D y E respectivamente, entonces la longitud AE es: A. 1 B. 3 C. 2 3 D. 3 5 E. 2 32  Como mAEB = 90, AE es una altura del triángulo equilátero ABC. Como el radio es 1, AB = 2, luego AE = 3 2 3 2   (En todo triángulo equilátero de lado x, la altura mide 3 x 2  ) A. 3 2 B. 5 3 C. 9 4 D. 25 9 E. 2 C A O B D E PROHIBIDA LA VENTA
  • 38. GEOMETRÍA 38 39. En una circunferencia se tienen dos cuerdas paralelas de longitudes 10 y 14 que distan 6 entre sí. Entonces la longitud de la cuerda paralela a ambas y que equidista de ellas mide: A. 11 B. 12 C. 13 D. 184 E. 192 Sean CD = 10 y AB = 14, las cuerdas dadas. Como la distancia entre ellas es 6, la cuerda paralela equidistante de ellas está a 3 unidades de cada una. Sea EF la cuerda buscada. Inicialmente no sabemos la posición de las cuerdas con respecto a un diámetro paralelo a ellas. Comencemos asumiendo que están al mismo lado del diámetro paralelo, como se muestra en la figura. Al trazar desde el centro una perpendicular a las cuerdas, esta pasa por el punto medio de cada cuerda. Sean P, Q, R los puntos medios de las cuerdas, como se muestra en la figura. Se tiene PB = 7, RD = 5. Sea QF = y, la longitud de la cuerda buscada es EF = 2y Supongamos que la cuerda AB está a x unidades del centro. Se forman tres triángulos rectángulos, todos ellos con hipotenusa igual al radio de la circunferencia. Al aplicar el teorema de Pitágoras en cada uno ellos se forma el siguiente sistema de ecuaciones 2 2 2 2 2 2 2 2 2 r x 12x 36 25 r x 12x 61 r x 6x 9 y r x 49               Restando la tercera ecuación de la primera 2 2 2 2 r x 12x 61 r x 49       12x = – 12  x = – 1 El valor negativo de x, nos indica que las cuerdas están en lados opuestos del diámetro paralelo a las cuerdas. Sustituyendo el valor de x en la tercera ecuación, obtenemos 2 r 50 Sustituyendo el valor de x y 2 r en la segunda ecuación obtenemos 50 = 1 – 6 + 9 + 2 y  y 46 y EF = 2y = 2 46 184 R 5 D Q y F P 7 B O O O x x +3 r r x + 6 r C R 5 D E Q y F A P 7 B x O 3 3 PROHIBIDA LA VENTA
  • 39. GEOMETRÍA 39 40. Un triángulo equilátero y un hexágono regular están inscritos en el mismo círculo. Si se divide el área del hexágono entre el área del triángulo se obtiene: A. 1.5 B. 2 C. 2.5 D. 3 E. 3 Al observar el grafico fácilmente se deduce que el área del hexágono es el doble del área del triángulo. Esto puede verificarse considerando que el lado de un triángulo equilátero inscrito en un círculo de radio r, está dada por 3 r y por tanto su área es   2 2 1 3 3 3 A 3 r r 4 4   También se tiene que el lado de un hexágono inscrito es igual al radio de la circunferencia, luego su área es 2 2 6 3 A r 4   , luego 2 2 1 2 6 3 rA 4 2 A 3 3 r 4     41. El área del círculo circunscrito a un hexágono regular es 2 cm 2 . Entonces el área del hexágono, en cm 2 es A. 6 B. 3 3 C. 4 2 D. 2 6 E. 2 63 Tenemos que el radio del círculo es 2 2 r 2 r 2     Como el hexágono está inscrito su lado mide r, luego su área es 6 3 A 2 3 3 4    42. En una circunferencia se trazan tres cuerdas con las siguientes longitudes: C1: 3.05, C2: 3.50 y C3: 0.305. ¿Cuál de las siguientes es una lista de las cuerdas en el orden en que se incrementa la distancia desde el centro de la circunferencia a la cuerda? A. C1C2C3 B. C3C1C2 C. C2C3C1 D. C2C1C3 E. C3C2C1 La cuerda de mayor longitud está más cerca del centro y la de menor longitud está más alejada. Luego como 2C 3.5 es la que está más cerca y le sigue 1C 3.05 y la más alejada es 3C 0.305 Por tanto el orden en que se incrementa la distancia es 2 1 3C C C PROHIBIDA LA VENTA
  • 40. GEOMETRÍA 40 43. En la figura, ABDE es un cuadrado de lado 1. Los arcos EB y DC tienen centro en A. Entonces el área sombreada mide: A. 2 1 B. 42 1   C. 42 1   D. 8   2 1 E. 4  1 La región EDB es la diferencia entre el área del cuadrado y el área del cuadrante ABE, es decir [EDB] = 2 2 1 1 1 4 4       El área de la región DBC es la diferencia entre el sector circular ACD y el triángulo rectángulo ABD, cuya área es 1 2 . El sector circular tiene radio AD, el cual es la diagonal del cuadrado y por tanto AD = 2 y su ángulo central es 45, es decir la octava parte de un circulo de radio 2 . Luego [BCD] =   2 1 1 2 8 2 4 2      Finalmente se tiene [EBCD] = [EDB] + [BCD] = 1 1 1 4 4 2 2                 Si tomamos tres vértices como se indica en la figura, se forma un triángulo rectángulo con catetos AB = 8 y AC = 24, la hipotenusa es el diámetro del círculo. Por el teorema de Pitágoras 2 2 BC 8 24 8 10   , luego r = 4 10 y el área del círculo es 2 r 160   El polígono está formado por 5 cuadrados de lado 8 y su área es 2 5 8 320  Luego el área sombreada es 160 – 320 = 160 ( – 2) El polígono de la figura tiene todos sus lados congruentes de longitud 8, sus lados consecutivos son perpendiculares y ocho vértices están sobre la circunferencia. Entonces el área de la región sombreada mide A. 144 – 320 B. 160 ( – 2) C. 144 – 320 D. 320(2 – 1) E. 80( – 2) 44. A B C A B C E D A B C D PROHIBIDA LA VENTA
  • 41. GEOMETRÍA 41 Al tomar el centro del semicírculo grande (A), el de un semicírculo mediano (B) y de un círculo pequeño (C), se forma un triángulo, en el cual AB = 2, BC = 2 + r y AC = 4 – r Al considerar el punto de tangencia entre los círculos pequeños, por la simetría de la figura CT es paralela al diámetro y mCTA = 90, formándose el triángulo rectángulo ATC con AT igual a la altura del ABC, CD: AT = CD y CT = r. De ahí resulta que ACT  CAD (teorema hipotenusa cateto), luego AD = CT = r. Se tiene AB = 2 y DB = 2 – r Por el Teorema de Pitágoras   2 2 AT 4 r r 16 8r CD      En el CDB se tiene     2 2 2 r 16 8r 2 r 4      2 4r r  16 8r 4   2 4r r  16r 16 r 1    46. El ABC es un triángulo rectángulo con el ángulo recto en C. Se traza la altura relativa a la hipotenusa y se inscriben dos circunferencias en los triángulos que se forman. Si los radios de estas circunferencias son 2 y 4 respectivamente, calcule el radio de la circunferencia inscrita en el ABC. Como el ABC es rectángulo en C y CD es la altura relativa a la hipotenusa, se tiene que ABC  ACD  CBD. Como en los triángulos semejantes, todos los elementos homólogos están en la misma razón de semejanza en la que están los triángulos, la razón de semejanza entre los triángulos CBD y ACD es la misma razón entre los radios de sus circunferencias inscritas, que en este ejercicio es 4 a 2, A D B C A. 52 B. 3 C. 25 D. 6 E. 8 45. Dos semicírculos de radio 2 están inscritos en un semicírculo de radio 4 como se muestra en la figura. Dos círculos de radio r, son tangentes a dos semicírculos y entre ellos. ¿Cuánto vale r ? r A. 2 1 B. 1 C. 2 3 D. 2 E. 2 5 C A B T A D B T r C A 4 – r 4 – r 2 + r C C D B 2 + r 2 – r A D B C PROHIBIDA LA VENTA
  • 42. GEOMETRÍA 42 es decir 2 a 1. De manera que todos los elementos del CBD son el doble de los elementos correspondientes del ACD. Dado que las áreas están en proporción según el cuadrado de la razón de semejanza, se tiene [CBD] = 4 [ACD] y como [ABC] = [ACD] + [CBD] = 5 [ACD], la razón de semejanza entre los triángulos ABC y ACD es 5 y por tanto el radio de su circunferencia inscrita es 5 veces el radio de la circunferencia inscrita o sea 2 5 Al unir los centros de los círculos se forma un triángulo equilátero de lado 2. Observamos que h es igual a dos veces el radio de los círculos más la altura del triángulo equilátero, la cual como el lado mide 2, es igual a 3 , luego h = 2 + 3 . 48. Tres semicírculos iguales, de radio R, tienen sus centros en los puntos colineales A, B y C tales que cada uno de estos puntos se encuentra sobre uno de los semicírculos. Se traza un cuarto círculo con centro D (el círculo sombreado) tal que es tangente a los tres semicírculos, tal como se muestra en la figura. Si r es el radio del círculo pequeño, la razón R a r es: A. 4:1 B. 15:4 C. 11:3 D. 10:3 E. 3:1 Al considerar el ABD, se tiene un triángulo rectángulo con AB = R, BD = R – r y AD = R + r. Por el teorema de Pitágoras,     2 22 2 2 2 2 AD AB BD R r R R r R        2 2Rr r  2 R 2 2 R 2Rr r   2 R 4Rr R 4 r     El área sombreada es la diferencia entre el área del triángulo y el segmento circular que forma por la intersección entre el triángulo y el círculo. Este segmento circular tiene un ángulo central de 60, ya que el ángulo inscrito mide 30, y un radio de 6, ya que el OAB es equilátero. A B C D 30º 6 8 8 49. ¿Cuál es el área de la región sombreada, redondeada al entero más cercano? A. 12 B. 16 C. 19 D. 22 E. 24 h 47. Si cada círculo tiene radio 1, y son tangentes entre si y a las líneas paralelas como se muestra en la figura ¿Cuánto mide h? A. 3 B. 3 2 C. 4 D. 2 + 3 E. 3 3 r r 30º 6 8 8 60º A B O PROHIBIDA LA VENTA
  • 43. GEOMETRÍA 43 El área del segmento circular es el área del sector circular menos el área del triángulo equilátero: 2 21 3 6 6 6 9 3 6 4        Para el área del triángulo usamos la fórmula de Herón:    A s s a s b s c     , donde s es el semiperímetro. Tenemos 6 8 8 s 11 2     , luego    A 11 11 6 11 8 11 8 3 55      Luego el área buscada es 3 55 – ( 6 9 3  ) = 18.9885  19 50. Los semicírculos de la figura tienen su centro sobre el segmento AB. Si el segmento CD es paralelo al segmento AB y CD = 24, ¿Cuál es el área de la región sombreada? Sean R y r los radios del semicírculo grande y del semicírculo interior respectivamente. El área buscada es la diferencia entre las áreas de los semicírculos o sea  2 2 R r 2   Sea O el centro del semicírculo y E el punto medio de la cuerda CD. Luego OE  CD, OE = r y ED = 12. El OED es rectángulo en E y OD = R. Por el teorema de Pitágoras 2 2 2 R r 12 144   Luego A =  2 2 R r 144 72 2 2        Soluciones 1 D 11 B 21 D 31 A 41 B 2 A 12 B 22 B 32 A 42 D 3 A 13 B 23 E 33 C 43 A 4 C 14 D 24 B 34 A 44 B 5 B 15 D 25 A 35 A 45 D 6 D 16 C 26 E 36 E 46 A 7 D 17 B 27 C 37 B 47 D 8 C 18 B 28 A 38 B 48 A 9 B 19 C 29 E 39 D 49 C 10 C 20 C 30 D 40 B 50 A C D A O B A. 72 B. 108 C. 144 D. 288 E. 576 E r PROHIBIDA LA VENTA
  • 44. GEOMETRÍA 44 4. CUERPOS SOLIDOS Como las bases son triángulos equiláteros de perímetro 30 cm, sus lados miden 10 cm y por tanto tienen una área de 2 b 3 A 10 25 3 4    . Su área lateral es LA P h 30 10 300     El área total está dada por T b LA 2 A A 2 25 3 300 50 3 300        2. Tres vértices de un cubo, de los cuales no hay dos que estén en la misma arista, se unen para formar un triángulo. Si la arista del cubo tiene longitud 1, ¿Cuál es el área del triángulo formado? A. 2 6 B. 2 3 C. 2 2 D. 4 6 E. 4 3 En la figura se muestra un triángulo que satisface el enunciado. Vemos que sus lados son diagonales de las caras y como las aristas de los cubos tienen longitud 1, estas diagonales miden 2 . Como el área de un triángulo equilátero de lado x está dada por 23 x 4 , en este caso tenemos   23 3 A 2 4 2   Al bosquejar los planos indicados vemos que comparten los puntos B y F, y dado que la intersección de dos planos diferentes es una única recta, la intersección es la recta BF  En el prisma recto de la figura, las bases son triángulos equiláteros, con perímetros de 30 cm.. Si la altura del prisma es 10 cm. ¿cuál es el área total de la superficie del prisma? A. 100 B. 250 3 C. 100 3 D. 300 E. 50 3 + 300 1. A B C D E F G La figura representa un cubo. La intersección del plano ABG y el plano BCE es la recta A. AG C. CE D. CFB. .BF E BE     3. A C B A B C D E F G A C D E F G Plano ABG Plano BCE B PROHIBIDA LA VENTA
  • 45. GEOMETRÍA 45 El volumen de la parte sobrante es la diferencia entre el volumen del cubo y el volumen del cilindro, luego 2 3 3 45 V 5 5 125 125 35.34 89.67 90 2 4                 5. La altura de un prisma rectangular es un tercio de su longitud y el ancho es la mitad de su longitud. Si la diagonal del prisma mide 30 cm., su volumen es A. 900 cm 3 B. 1688.25 cm 3 C. 2833.8 cm 3 D. 4583.5 cm 3 E. 9000 cm 3 Sean z la altura, y la longitud y x el ancho del prisma. Se tiene y z 3  , y x 2  La diagonal está dada por 2 2 2 2 2 2 d x y z 30 x y z 900        . Al expresar en términos de “y” esta ecuación, se obtiene 2 2 2 2 2y y 1 1 49 180 y 900 1 y 900 y 900 y 4 9 4 9 36 7                 . El volumen está dado por V = x y z = 33 3y y y 1 180 y 2833.8 cm 2 3 6 6 7           6. Al introducir un trozo de metal en un tanque rectangular con agua, de dimensiones 50 cm. x 37 cm., el nivel del agua subió 1 cm. ¿cuál es el volumen del trozo de metal? A. 13 cm 3 B. 87 cm 3 C. 88 cm 3 D. 1850 cm 3 E. 9250 cm 3 El volumen del trozo de metal es equivalente al volumen que incrementó el tanque, lo cual equivale al volumen de un paralelepípedo de dimensiones 1 x 50 x 37, es decir 1850 cc 7. ¿Cuál es el número máximo de diagonales que pueden trazarse sobre las caras de un cubo de manera que no hayan dos diagonales que tengan un punto en común? A. 2 B. 3 C. 4 D. 5 E. 6 Trazamos inicialmente sobre una de las caras una diagonal, digamos AD, ninguna otra puede involucrar estos puntos para satisfacer la condición. Con los puntos restantes trazamos otra diagonal, digamos BE. Nos quedan cuatro vértices en este caso los vértices C, F, G y H, con los cuales solo podemos trazar dos diagonales más. En total cuatro diagonales. Cualquier otra variante conduce a la misma cantidad. De un cubo de 5” de arista se forma un cilindro circular recto de 3” de diámetro, entonces el volumen de la parte sobrante del cubo, en pulgadas cúbicas, es aproximadamente A. 8 B. 10 C. 80 D. 90 E. 100 4. z x y 50 37 A B C D E F G H PROHIBIDA LA VENTA
  • 46. GEOMETRÍA 46 8. En la figura se muestra un paralelepípedo rectangular. Si a = 2b y b = 2 c , ¿Cuál es el volumen en términos de c? A. 2 c2 B. 2c2 C. 3 c D. 2 c3 E. 4 c3 Como a = 2b y b = 2 c , entonces a = c. Por ser un paralelepípedo rectangular, V = a b c = c 2 c  c = 3 c 2 9. El área de la base de una pirámide es 45 y el área de una sección transversal es 20. Si la altura de la pirámide es 6 ¿a qué distancia de la sección transversal está el vértice? A. 1.5 B. 2.25 C. 4 D. 4.75 E. 5 Sea A’ = 20, el área de la sección transversal y A = 45, el área de la base de la pirámide. Sea h la distancia desde la sección transversal al vértice. Se tiene 2 A' h 20 4 h 2 h 4 A 6 45 9 6 3             10. El área de la base de una pirámide es 45 y el área de una sección transversal es 20. Si la altura de la pirámide es 6 ¿cuál es la razón entre los volúmenes de la pirámide mayor y la menor? A. 3/2 B. 2 C. 9/4 D. 3 E. 27/8 Los datos forman parte del ejercicio anterior, de manera que ya sabemos que la distancia desde la sección transversal al vértice es h = 4. Luego Si V es el volumen de la pirámide mayor y V’ el de la pirámide menor, se tiene 3 3 V 6 3 27 V' 4 2 8               11. La base de una pirámide es un triángulo equilátero cuyo perímetro es 12. Si la altura es 10, el volumen de la pirámide es A. 40 B. 3 40 C. 3 340 D. 40 3 E. 120 Como la base de la pirámide es un triángulo equilátero cuyo perímetro es 12, su lado mide 4, luego 2 b 3 A 4 4 3 4    y como h = 10, el volumen de la pirámide es b 1 1 40 3 V A h 4 3 10 3 3 3       a b c 6 h PROHIBIDA LA VENTA
  • 47. GEOMETRÍA 47 12. En un tronco de pirámide, la altura mide 10 m y las bases son cuadradas de 5 m y 9 m de lado respectivamente. Hallar la diferencia (en m 3 ) entre su volumen y la de un prisma recto de igual altura y de base igual a la sección del tronco paralela a las bases y equidistante de ellas. A. 4 B. 7 C. 40 D. 3 40 E. 70 Como cada lateral del tronco de pirámide, la sección que equidista de las bases tiene arista igual a la base media del trapecio, en este caso es 9 5 B 7 2    . El volumen del tronco de pirámide resulta  B b B b h 10 1510 V [A A A A ] 81 25 81 25 3 3 3          El volumen del prisma formado es V’ = 2 BA h 7 10 490    La diferencia resulta V – V’ = 1510 40 490 3 3   13. En una pirámide cuadrada, en la que el lado de la base mide 8 cm y la altura mide 20 cm, se traza una sección paralela a la base a 14 cm de ésta. Entonces el área de dicha sección es A. 2.14 cm 2 B. 5.76 cm 2 C. 16.32 cm 2 D. 31.36 cm 2 E. 44.08 cm 2 Sea x la longitud de la arista de la sección, se tiene entonces x 6 48 x 2.4 8 20 20     Como es un cuadrado, su área es   22 A x 2.4 5.76   14. Los diámetros de dos cilindros circulares rectos concéntricos son 12 y 6 pulgadas respectivamente y la generatriz común es de 20 pulgadas, entonces el volumen del espacio que queda entre ambos cilindros es A. 270 pulg 3 B. 270 pulg 3 C. 540 pulg 3 D. 540 pulg 3 E. 2160 pulg 3 El volumen buscado es la diferencia entre los volúmenes de los cilindros. Como los diámetros son 12 y 6, los radios son 6 y 3, luego,    2 2 2 2 V R r h 6 3 20 540          10 5 5 5 9 7 7 10 20 6 14 8 PROHIBIDA LA VENTA
  • 48. GEOMETRÍA 48 15. El volumen de una cisterna cilíndrica es 1200 m 3 y su altura es igual al diámetro, por lo tanto su área total es A. 190.98 m 2 B. 576.25 m 2 C. 600 m 2 D. 625.13 m 2 E. 712 m 2 Como el diámetro es igual a la altura se tiene h r 2  . Luego 2 3 2 3h h 4800 V r h h 1200 h 11.5176 2 4                  El área total está dada por T B LA 2 A A   El área de la base es 2 2 2 B h h A r 2 4            El area lateral es 2 L h A 2 rh 2 h h 2        Luego 2 2 2 T B L 3h A 2 A A 2 h h 4 2          Al sustituir el valor de h obtenemos 2 T 3 A h 625.13 2    16. Un cono de revolución tiene 13 cm. de generatriz y el radio de la base es de 5 cm. Se corta por un plano paralelo a la base que corta a la generatriz en un punto distante 5.2 cm. del vértice. Entonces el volumen del tronco de cono formado es A. 351.52 cm 3 B. 294.05 cm 3 C. 202.8 cm 3 D. 135.2 cm 3 E. 67.6 cm 3 Sea r el radio de la base menor del cono truncado. Sea y la altura del cono menor y H la altura del cono truncado. Al considerar la altura del cono, los radios de las bases y la generatriz obtenemos los triángulos rectángulos que se muestran en la figura. La altura del cono original es 2 2 2 2 h g r 13 5 12     Como los triángulos formados son semejantes se tiene y r 5.2 24 y , r 2 12 5 13 5      Como H + h = 12, se tiene 24 36 H 12 5 5    El volumen buscado es    2 2 36 V H R r Rr 25 4 10 294.05 3 3 5            5.2 13 13 5.2 5 12 y H r PROHIBIDA LA VENTA
  • 49. GEOMETRÍA 49 17. Dado un cono circular recto con radio 3 m y generatriz 5 m, entonces su área lateral es A. 2 B. 12 C. 15 D. 16 E. 30 El área lateral está dada por LA rg 3 5 15       18. El área lateral de un tronco de cono que se forma cuando se corta un cono recto de 6 cm. de radio y 8 cm. de altura, por medio de un plano paralelo a la base del cono y que lo corta a una altura de 4.5 cm. es A. 304.84 m 2 B. 216 m 2 C. 152.42 m 2 D. 84.82 m 2 E. 28.27 m 2 Se tiene  LA g R r   , donde g es la generatriz del tronco de cono. La generatriz del cono está dada por 2 2 2 2 g' h r 8 6 10     Como los triángulos que se forman con la altura, la generatriz y los radios son semejantes, se tiene r 3.5 r 2.625 6 8    Por el teorema de Thales, g 10 g 5.625 4.5 8    Luego    LA g R r 5.625 6 2.625 152.42        19. Dos esferas de metal de radios 2a y 3a se funden juntos para hacer una esfera mayor. El radio de la nueva esfera es A. 2.5a B. 5a C. 6.5a D. 3 35 a E.  a5 El volumen de la nueva esfera es la suma de los volúmenes de las esferas dadas. La esfera de radio 2 a tiene un volumen   33 3 1 4 4 32 V r 2a a 3 3 3       La esfera de radio 3 a tiene un volumen   33 3 2 4 4 108 V r 3a a 3 3 3       El volumen de la nueva esfera es 33 3 3 3 1 2 32 108 140 4 V V V a a a r r a 35 3 3 3 3             r 6 4.5 10 x 6 8 3.5 g r PROHIBIDA LA VENTA
  • 50. GEOMETRÍA 50 20. Un cono tiene una altura igual al doble de su radio. Una esfera tiene un radio igual al radio de la base del cono. La razón entre el volumen del cono y el volumen de la esfera es A. 1/2 B. 1 C. 3/2 D. 2 E. 4 Volumen del cono 2 2 3 C 1 1 2 V r h r 2r r 3 3 3        Volumen de la esfera 3 E 4 V r 3   Luego 3 C 3E 2 rV 13 4V 2 r 3     21. Un cono tiene una altura igual al triple de su radio. Una esfera tiene un radio igual al radio de la base del cono. La razón entre el volumen del cono y el volumen de la esfera es A. 1/2 B. 1 C. 3/2 D. 3/4 E. 3 Volumen del cono 2 2 3 C 1 1 V r h r 3r r 3 3        Volumen de la esfera 3 E 4 V r 3   Luego 3 C 3E V r 3 4V 4 r 3     22. La altura de un cono es 5 cm. Un plano a 2 cm. del vértice es paralelo a la base del cono. Si el volumen del cono más pequeño es 24 cm 3 , el volumen del cono más grande es A. 750 cm 3 B. 375 cm 3 C. 240 cm 3 D. 120 cm 3 E. 48 cm 3 Se tiene que los volúmenes de cono semejantes son proporcionales al cubo de su razón de semejanza 3 1 2 V h V H        Luego el volumen del cono más grande es 3 5 V 24 375 2         2 r r r 3 r r r 2 5 PROHIBIDA LA VENTA
  • 51. GEOMETRÍA 51 23. Un cubo está inscrito en una esfera. Si el área de la superficie total del cubo es 2 m  40 , entonces el área de la superficie de la esfera es A. 10 m 2 B. 15 m 2 C. 20 m 2 D. 30 m 2 E. 40 m 2 Tenemos que la diagonal del cubo es el diámetro de la esfera. Si x es la longitud de la arista del cubo, su área total es 2 240 20 6x x 3      En un cubo la diagonal es d 3 x , luego el radio de la esfera es 3 r x 2  El área de la superficie de la esfera es 2 S 4 r  , al sustituir los valores encontrados resulta 2 23 20 S 4 r 4 x 3 20 4 3         24. La base de una pirámide hexagonal tiene un área de 26 m 2 . Si el volumen de dicha pirámide es 78 m 3 , entonces su altura mide A. 3 m B. 4 m C. 6 m D. 9 m E. 12 m Tenemos que el volumen de una pirámide esta dado por B B 3V1 V A h h 3 A      Al sustituir los datos se obtiene B 3V 3 78 h 9 A 26     Al considerar el triángulo formado por el diámetro AB y el vértice, tenemos que mCAB = mCBA = 30 Luego la altura del cono es r h 3  . El volumen es 3 2 2 r1 1 r V r h r 3 3 3 3 3        Se tiene entonces 3 3r 1000 3 r 1000 r 10 93 3        y por tanto el diámetro es d = 20. A C B 25. h r Si el cono de la figura tiene un volumen de 31000 3 cm 9  , C es el vértice, AB un diámetro y mACB = 120°, entonces el diámetro de la base, en centímetros, es A. 5 B. 10 C. 15 D. 20 E. 30 PROHIBIDA LA VENTA
  • 52. GEOMETRÍA 52 26. El área de la superficie total de un cubo es 12 m 2 . Entonces la longitud de su diagonal es A. 2 B. 3 C. 2 D. 5 E. 6 El área de la superficie total de un cubo de lado x, está dada por 2 TA 6x y su diagonal por d 3 x Luego 2 6x 12 x 2   y d 3 x 3 2 6    27. Si la generatriz de un cono mide 25 m y el diámetro de su base es 8 m, su volumen mide A. 200 m 3 B. 400 m 3 C. 413.48 m 3 D. 418.88 m 3 E. 1587.4 m 3 El volumen está dado por 21 V r h 3   y la altura es 2 2 h g r  Tenemos que g = 25 y d = 8 o sea r = 4, luego 2 2 h 25 4 609   , por tanto 21 V 4 609 413.48 3    28. En una esfera de radio 2, se tiene inscrito un cilindro de manera que el diámetro del cilindro es igual al radio de la esfera. Entonces el área lateral del cilindro es A. 4 B. 8 C. 2 3 D. 4 3  E. 8 3 Sean A, B y C los puntos marcados en la figura. Tenemos que AB es diámetro del cilindro y BC diámetro de la esfera, luego AB = 2 y BC = 4. La altura del cilindro es AC, el cual al aplicar el Teorema de Pitágoras resulta 2 2 2 2 AC BC AB 4 2 12 2 3      El área lateral de un cilindro está dada por LA 2 rh  . Como el diámetro del cilindro mide 2, su radio mide 1. Luego LA 2 1 2 3 4 3     29. En una esfera de radio 2, se tiene inscrito un cilindro de manera que el diámetro del cilindro es igual al radio de la esfera. Entonces el volumen del cilindro es A. 4 B. 8 C. 2 3  D. 4 3 E. 8 3 Este ejercicio contiene los mismos datos del anterior, luego el cilindro tiene r = 1 y h = 2 3 Luego su volumen 2 2 V r h 1 2 3 2 3       A B C PROHIBIDA LA VENTA
  • 53. GEOMETRÍA 53 30. Si un cilindro circular tiene la altura igual que su diámetro y su área total mide 150 m 2 , entonces su volumen, en m 3 , es A. 25 B. 250 C. 250 D. 25 E. 150 Tenemos que 2 2 2 TA 2 r 2 rh 2 r 2 r 2r 6 r 150 r 5               y h = 10. Luego 2 2 V r h 5 10 250       31. Se desea construir un tubo cilíndrico de 50 cm. de largo, un diámetro externo de 12 cm. y un diámetro interno de 10 cm. ¿Qué cantidad de material se necesita? A. 550 cm 3 B. 500 cc 3 C. 375 cm 3 D. 750 cm 3 E. 1275 cm 3 . La cantidad de material es el volumen del tubo y este es la diferencia del volumen del cilindro exterior menos el volumen del cilindro interior, que representa el agujero. Tenemos: 1 2h 50, r 6, r 5   Luego V = (6) 2 (50) – (5) 2 (50) = 550  cm 3 32. Se desea construir un cono circular recto de 4’’ de altura y radio en la base de 3’’. Para formar la superficie lateral se corta un sector circular como el que se muestra en la figura. Determine el radio R y la medida del ángulo central  del sector circular necesario. Se tiene que la generatriz del cono mide 2 2 2 2 g h r 4 3 5     y que ésta es el radio del sector circular con la que se construye el cono, es decir R = 5. El área lateral del cono es el área del sector circular necesario para su construcción. Tenemos LA rg 3 5 15       . Como el área de un sector circular está dada por 2 R 360    Se tiene 2 2 R 5 1080 15 216 360 360 5                 R A. R = 7’’;  = 200º B. R = 6’’;  = 200º C. R = 6’’;  = 215º D. R = 5’’;  = 216º E. R = 5’’;  = 240º 4 g 3 r 2r PROHIBIDA LA VENTA
  • 54. GEOMETRÍA 54 33. La figura muestra dos esferas tangentes que descansan sobre una mesa plana. Si los radios de las esferas son 8 cm. y 16 cm. respectivamente, entonces la distancia en cm. entre los puntos de contacto de las esferas con la mesa es: Al considerar los centros de las esferas y los puntos de tangencia con la mesa, se forma un trapecio, como el que se muestra en la figura, en el cual OO’ = 24 ya que las esferas son tangentes. OA = 16 y O’B = 8. Si trazamos una paralela a la mes desde O’, sea D el punto donde corta al radio OA. Tenemos que AB = PO’, PA = O’B = 8, luego OP = OA – PA = 8 Como OA  AB, también se tiene OP  PO’. Luego por el teorema de Pitágoras se tiene 2 2 PO' AB 24 8 16 2    Tenemos una pirámide de base triangular. La base es un triángulo rectángulo ( __ __ BD DC ) de catetos BD = 12 y CD = 16, luego su área es b 1 A 12 16 96 2     Como __ __ AB BC y __ __ AB BD , se deduce que __ AB es perpendicular al plano de la base y por tanto es altura de la pirámide, luego b 1 1 V A h 96 9 288 3 3        x A. 8 2 B. 12 C. 16 2 D. 24 E. 24 2 A B C D 34. En la figura que se muestra, ____ BCAB  , ____ BDAB  y ____ DCBD  . Si AB = 9, BD = 12 y CD = 16, entonces el volumen del sólido formado es: A. 144 B. 288 C. 432 D. 576 E. 864 O O’P A B PROHIBIDA LA VENTA
  • 55. GEOMETRÍA 55 35. Un paralelepípedo rectangular se recorta como se muestra en la figura. El volumen del sólido obtenido es: A. 72 B. 48 C. 24 D. 16 E. 12 Basta observar que la parte recortada (la parte superior punteada) es idéntica a la parte que queda del sólido, luego el volumen buscado es la mitad del volumen del paralelepípedo original, es decir 1 V 6 4 4 48 2      1 E 8 D 15 D 22 B 29 C 2 B 9 C 16 B 23 C 30 B 3 B 10 E 17 C 24 D 31 A 4 D 11 C 18 C 25 D 32 D 5 C 12 D 19 D 26 E 33 C 6 D 13 B 20 A 27 C 34 B 7 C 14 D 21 D 28 D 35 B 4 2 4 2 6 4 PROHIBIDA LA VENTA